You are on page 1of 263

PASCUA CASE DIGESTS

CHAPTERS I to III
BY WIGMORE 2022-2023
TABLE OF CONTENTS
TABLE OF CONTENTS 2

SIMPLIFIED COURSE SYLLABUS 7


PEOPLE VS. PERFECTO 9
MACARIOLA VS. ASUNCION 10
MONTEBON VS. THE DIRECTOR OF PRISONS 11
VALENCIA VS. LOCQUIAO 12
SJS VS. DANGEROUS DRUGS BOARD 13
SABIO VS. GORDON 14
ATTY. MACALINTAL VS. COMMISSION ON ELECTIONS 15
MANILA PRINCE HOTEL VS. GSIS 16
CIR VS. SAN ROQUE POWER CORP. 17
FRANCISCO JR. VS. THE HOUSE OF REPRESENTATIVES 18
SERRANO VS. GALLANT MARITIME 19
DATU MICHAEL ABAS KIDA vs. SENATE 20
TA​ÑADA VS. ANGARA 21
MARCOS VS. MANGLAPUS 22
ESTRADA VS. DESIERTO 24
LAWYERS’ LEAGUE VS. AQUINO 25
IN RE: SATURNINO V. BERMUDEZ 26
DE LEON VS. ESGUERRA 27
TAÑADA VS. TUVERA 28
REPUBLIC VS. SANDIGANBAYAN, RAMAS AND DIMAANO 29
MANILA PRINCE HOTEL VS. GSIS 31
GAMBOA VS. TEVES 32
TA​​ÑADA VS. ANGARA 33
PAMATONG VS. COMELEC 34
REPUBLIC VS. SERENO 36
TAÑADA VS. ANGARA 37
CHIONGBIAN VS. DE LEON 38
CIVIL LIBERTIES UNION VS. EXECUTIVE SECRETARY 39
ANG BAGONG BAYANI VS. COMELEC 41
J.M. TUASON & CO., INC. VS. LAND TENURE ADMINISTRATION 43
DOMINO VS. COMELEC 45
NAVARRO, BERNAL, AND MEDINA VS. EXECUTIVE SECRETARY 47
FRIVALDO VS. COMELEC 48
ATTY. MACALINTAL VS. PET. CITED IN CHAVEZ VS. JBC 49
TONDO MEDICAL CENTER EMPLOYEES ASSOCIATION VS. CA 50
BELGICA VS. EXECUTIVE SECRETARY 51
ARAULLO VS. AQUINO III 52
ANGARA VS. ELECTORAL COMMISSION 53
COCOFED VS. REPUBLIC 54
TAWANG MULTI-PURPOSE COOPERATIVE VS. LA TRINIDAD WATER DISTRICT 55
CALLO-CLARIDAD VS. ESTEBAN 56
METROBANK VS. TOBIAS III 57
PEOPLE VS. SITON 58
PEREZ VS. PEOPLE 59
LOPEZ VS. ROXAS AND PET 63
BALANGAUAN VS. CA 64
SINGIAN, JR. VS. SANDIGANBAYAN 66
PEOPLE VS. VERA 68
ANGARA VS. ELECTORAL COMMISION 69
IMBONG VS. OCHOA, JR. 70
BAKER VS. CARR 71
PEOPLE AND HSBC VS. VERA 72
ABAKADA VS. PURISIMA 73
TELEBAP VS. COMELEC 74
PHILIPPINE ASSOCIATION OF COLLEGES AND UNIVERSITIES VS. SECRETARY OF
EDUCATION 75
JOYA VS. PCGG 76
CHAVEZ VS. JBC 77
GALICTO VS. AQUINO 78
DEL MAR VS. PHILIPPINE AMUSEMENT AND GAMING CORPORATION 79
BIRAOGO VS. PHILIPPINE TRUTH COMMISSION 80
DEMETRIA VS. ALBA 81
LAMP VS. SECRETARY OF BUDGET AND MANAGEMENT 82
DELA LLANA VS. THE CHAIRPERSON, COA, EXECUTIVE SECRETARY, AND
NATIONAL TREASURER 83
BELGICA VS. EXECUTIVE SECRETARY 84
THE CITY OF DAVAO VS. RTC 86
CHAVEZ VS. PCGG (sample case Title) 87
IDEALS, INC. VS. PSALM 88
KILOSBAYAN VS. GUINGONA 89
IDEALS, INC. VS. PSALM 91
IMBONG VS. OCHOA, JR. 92
OPOSA VS. FACTORAN 93
RESIDENT MARINE MAMMALS OF THE PROTECTED SEASCAPE OF TAÑON STRAIT
VS. REYES 94
BELGICA VS. EXECUTIVE SECRETARY 95
TADEO VS. PROVINCIAL FISCAL OF PANGASINAN 96
SOUTHERN HEMISPHERE ENGAGEMENT NETWORK, INC. VS. ANTI TERRORISM
COUNCIL 97
APEX MINING CO., VS. SOUTHEAST MINDANAO CORP 98
LOZANO VS. NOGRALES 99
IMBONG VS. OCHOA, JR. 100
CANDARI VS. DONASCO 101
BELGICA VS. EXECUTIVE SECRETARY 102
GALICTO VS. AQUINO 103
PORMENTO VS. ESTRADA AND COMELEC 104
IDEALS VS. PSALM 105
FUNA VS. THE CHAIRMAN, COA, VILLAR 106
VILANDO VS. HRET 107
GENERAL VS. URRO 108
SPS. MIRASOL VS. CA 110
CREBA VS. ROMULO 111
DUEÑAS VS. HRET 112
JAVIER VS. COMELEC 113
QUIZON VS. COMELEC 114
ABS-CBN VS. COMELEC 115
J.M. TUASON & CO. VS CA 116
YNOT VS. IAC 117
ARAULLO, ET. AL. VS. AQUINO III, ET. AL. 118
BROKENSHIRE MEMORIAL HOSPITAL, INC. VS. HON. MINISTER OF LABOR AND
EMPLOYMENT 119
ONGSUCO VS. MALONES 120
FERNANDEZ VS. CUERVA 122
ALDOVINO VS. ALUNAN 123
Francisco Serrano de Agbayani vs. Philippine National Bank 124
FLORES VS. DRILON 125
HACIENDA LUISITA INC. VS. PRESIDENTIAL AGRARIAN REFORM COUNCIL 126
COCOFED VS. REPUBLIC 127
ARAULLO, ET. AL. VS. AQUINO III, ET. AL. 128
CIR VS. RUEDA 130
SAGUISAG VS. EXECUTIVE SECRETARY 131
NICOLAS VS. ROMULO 133
MAGALLONA VS. ERMITA 134
ARIGO VS. SWIFT 135
QUA CHEE GAN VS. DEPORTATION BOARD 136
TECSON AND DESIDERIO VS. COMELEC 137
GO SR. VS. RAMOS 138
GONZALEZ VS. PENNISI 140
RE: APPLICATION FOR ADMISSION TO THE PHILIPPINE BAR, VICENTE D. CHING 142
REPUBLIC VS. LIM 143
MA VS. FERNANDEZ, JR. 144
REPUBLIC VS. SAGUN 146
KILOSBAYAN VS. EXECUTIVE SECRETARY 148
POE-LLAMANZARES VS. COMELEC 149
REPUBLIC VS. KERRY LAO ONG 151
GO VS REPUBLIC 152
REPUBLIC VS. AZUCENA 154
REPUBLIC VS. LI CHING CHUNG 155
AASJS VS. DATUMANONG 157
LOPEZ VS. COMELEC 158
JACOT VS. DAL 159
DE GUZMAN VS. COMELEC 160
SOBEJANA-CONDON VS. COMELEC 161
GUY VS. IGNACIO 163
YU VS. DEFENSOR-SANTIAGO 164
Jose B. Aznar vs. Emilio Mario Renner Osmeña 166
VILANDO VS. HRET 167
PEOPLE VS. MANAYAO 168
ANGAT VS. REPUBLIC 169
MERCADO VS. MANZANO 172
AASJS VS. DATUMANONG 173
PEDRO LEE HONG HOK VS. DAVID 174
PROFESSIONAL VIDEO, INC. VS. TESDA 176
REPUBLIC VS. VILLASOR 178
JUSMAG VS. NLRC 180
AMONOY VS. SPS. GUTIERREZ 183
PROFESSIONAL VIDEO INC. VS. TESDA 185
ATO VS. SPS. RAMOS 187
CHINA NATIONAL MACHINERY & EQUIPMENT CORP. (GROUP) VS. SANTA MARIA 189
LANSANG VS. CA 191
CALUB VS. CA 192
PHILIPPINE AGILA SATELLITE INC. VS. DE GUZMAN 193
DOH VS. PHIL. PHARMAWEALTH, INC. 196
REPUBLIC VS, PURISIMA 197
DA VS. NLRC 198
SAYSON VS. SINGSON 200
MERRITT VS, GOVERNMENT OF THE PHILIPPINE ISLANDS 201
NHA VS. HEIRS OF GUIVELONDO 202
UNITED STATES VS. RUIZ 204
MALONG VS. PNR 206
United States Vs. Hon. Eliodoro Guinto et al. 207
DEPED VS. OÑATE 208
PTA VS. PGDEI 209
EPG CONSTRUCTION CO. VS. VIGILAR 210
REPUBLIC VS. UNIMEX 211
FROILAN VS. ORIENTAL PAN SHIPPING: GR L-6060 213
THE COMMISSIONER OF PUBLIC HIGHWAYS VS. SAN DIEGO 214
Juco v. NLRC 216
PNB VS. CIR 218
REPUBLIC VS. HIDALGO 219
THE MUNICIPALITY OF SAN FERNANDO, LA UNION VS. FIRME 221
BACANI VS. NACOCO-GR. L-9657 223
NPC VS. CITY OF CABANATUAN 224
MALTO VS. PEOPLE as stated in CABALLO VS. PEOPLE 225
PEOPLE VS. BAYLON 227
VELASCO, JR. OPINION in GATUS VS. QUALITY HOUSE, INC. 229
PEOPLE VS. BABASA 231
CABANAS VS. PILAPIL 232
FERNANDO VS. CA 233
SIMPLIFIED COURSE SYLLABUS
I. IN GENERAL TONDO MEDICAL CENTER RESIDENT MARINE MAMMALS OF
PEOPLE VS. PERFECTO EMPLOYEES ASSOCIATION VS. THE PROTECTED SEASCAPE OF
MACARIOLA VS. ASUNCION CA TAÑON STRAIT VS. REYES
MONTEBON VS. THE DIRECTOR BELGICA VS. EXECUTIVE BELGICA VS. EXECUTIVE
OF PRISONS SECRETARY SECRETARY
VALENCIA VS. LOCQUIAO ARAULLO VS. AQUINO III TADEO VS. PROVINCIAL FISCAL
SJS VS. DANGEROUS DRUGS ANGARA VS. ELECTORAL OF PANGASINAN
BOARD COMMISSION SOUTHERN HEMISPHERE
SABIO VS. GORDON COCOFED VS. REPUBLIC ENGAGEMENT NETWORK, INC.
ATTY. MACALINTAL VS. TAWANG MULTI-PURPOSE VS. ANTI TERRORISM COUNCIL
COMMISSION ON ELECTIONS COOPERATIVE VS. LA TRINIDAD APEX MINING CO., VS.
MANILA PRINCE HOTEL VS. GSIS WATER DISTRICT SOUTHEAST MINDANAO CORP
CIR VS. SAN ROQUE POWER CALLO-CLARIDAD VS. ESTEBAN LOZANO VS. NOGRALES IMBONG
CORP. METROBANK VS. TOBIAS III VS. OCHOA, JR.
FRANCISCO JR. VS. THE HOUSE PEOPLE VS. SITON CANDARI VS. DONASCO
OF REPRESENTATIVES PEREZ VS. PEOPLE BELGICA VS. EXECUTIVE
SERRANO VS. GALLANT SMART COMMUNICATIONS VS. SECRETARY
MARITIME MUNICIPLATY OF MALVAR GALICTO VS. AQUINO
DATU MICHAEL ABAS KIDA VS. LOPEZ VS. ROXAS AND PET PORMENTO VS. ESTRADA AND
SENATE BALANGAUAN VS. CA COMELEC
TA​ÑADA VS. ANGARA SINGIAN, JR. VS. IDEALS VS. PSALM
MARCOS VS. MANGLAPUS SANDIGANBAYAN FUNA VS. THE CHAIRMAN, COA,
PEOPLE VS. VERA VILLAR
II. THE BACKGROUND OF THE ANGARA VS. ELECTORAL VILANDO VS. HRET
PRESENT CONSTITUTION COMMISION GENERAL VS. URRO
LAWYERS’ LEAGUE VS. AQUINO IMBONG VS. OCHOA, JR. SPS. MIRASOL VS. CA
IN RE: SATURNINO V. BERMUDEZ BAKER VS. CARR CREBA VS. ROMULO
DE LEON VS. ESGUERRA PEOPLE AND HSBC VS. VERA DUEÑAS VS. HRET
TAÑADA VS. TUVERA ABAKADA VS. PURISIMA JAVIER VS. COMELEC
REPUBLIC VS. SANDIGANBAYAN, TELEBAP VS. COMELEC QUIZON VS. COMELEC
RAMAS AND DIMAANO PHILIPPINE ASSOCIATION OF ABS-CBN VS. COMELEC
COLLEGES AND UNIVERSITIES J.M. TUASON & CO. VS CA
III. JUDICIAL ELABORATION OF VS. SECRETARY OF EDUCATION YNOT VS. IAC
THE CONSTITUTION JOYA VS. PCGG ARAULLO, ET. AL. VS. AQUINO III,
MANILA PRINCE HOTEL VS. GSIS CHAVEZ VS. JBC ET. AL.
GAMBOA VS. TEVES GALICTO VS. AQUINO BROKENSHIRE MEMORIAL
TA​​ÑADA VS. ANGARA DEL MAR VS. PHILIPPINE HOSPITAL, INC. VS. HON.
PAMATONG VS. COMELEC AMUSEMENT AND GAMING MINISTER OF LABOR AND
GUTIERREZ VS HOUSE OF CORPORATION EMPLOYMENT
REPRESENTATIVES BIRAOGO VS. PHILIPPINE TRUTH ONGSUCO VS. MALONES
REPUBLIC VS. SERENO COMMISSION FERNANDEZ VS. CUERVA
TAÑADA VS. ANGARA DEMETRIA VS. ALBA ALDOVINO VS. ALUNAN
CHIONGBIAN VS. DE LEON LAMP VS. SECRETARY OF SERRANO VS. AGBAYANI
CIVIL LIBERTIES UNION VS. BUDGET AND MANAGEMENT FLORES VS. DRILON
EXECUTIVE SECRETARY DELA LLANA VS. THE HACIENDA LUISITA INC. VS.
BOLOS VS. BOLOS CHAIRPERSON, COA, EXECUTIVE PRESIDENTIAL AGRARIAN
ANG BAGONG BAYANI VS. SECRETARY, AND NATIONAL REFORM COUNCIL
COMELEC TREASURER COCOFED VS. REPUBLIC
DATU MICHAEL ABAS KIDA VS. BELGICA VS. EXECUTIVE ARAULLO, ET. AL. VS. AQUINO III,
SENATE SECRETARY ET. AL.
J.M. TUASON & CO., INC. VS. THE CITY OF DAVAO VS. RTC
LAND TENURE ADMINISTRATION CHAVEZ VS. PCGG
DOMINO VS. COMELEC IDEALS, INC. VS. PSALM
NAVARRO, BERNAL, AND MEDINA KILOSBAYAN VS. GUINGONA
VS. EXECUTIVE SECRETARY IDEALS, INC. VS. PSALM
FRIVALDO VS. COMELEC IMBONG VS. OCHOA, JR.
ATTY. MACALINTAL VS. PET. OPOSA VS. FACTORAN
CITED IN CHAVEZ VS. JBC
I. IN GENERAL
PEOPLE VS. PERFECTO
JUEZAN, VJT
TOPIC: Defenition/Effectivity
FACTS:

The issue started when the Secretary of the Philippine Senate, Fernando Guerrero,
discovered on August 1920 that the documents regarding the testimony of the
witnesses in an investigation of oil companies had disappeared from his office. Then,
the day following the convening of the Senate, the newspaper La Nacion – edited by
herein respondent Gregorio Perfecto – published an article against the Philippine
Senate. Here, Mr. Perfecto was alleged to have violated Article 256 of the Spanish
Penal Code – provision that punishes those who insult the Ministers of the Crown.

ISSUE:

Whether or not Article 256 of the Spanish Penal Code (SPC) is still in force and can
be applied in the case at bar?

RULING:

No. The Court stated that during the Spanish Government, Article 256 of the SPC was
enacted to protect Spanish officials as representatives of the King. Also, with the
change of sovereignty over the Philippines from Spanish to American, it means that
the invoked provision of the SPC had been automatically abrogated. The Court
determined Article 256 of the SPC to be ‘political’ in nature for it is about the relation of
the State to its inhabitants, thus, the Court emphasized that ‘it is a general principle of
the public law that on acquisition of territory, the previous political relations of the
ceded region are totally abrogated.’ Hence, Article 256 of the SPC is considered no
longer in force and cannot be applied to the present case. Therefore, the respondent
was acquitted.
MACARIOLA VS. ASUNCION

TOPIC: Definition/Effectivity
Full Text: https://lawphil.net/judjuris/juri1982/may1982/am_133_j_1982.html
FACTS:
August 6, 1968: Petitioner Bernardita R. Macariola charged respondent Judge Elias B.
Asuncion of the Court of First Instance of Leyte, now Associate Justice of the Court of
Appeals, with "acts unbecoming a judge.” The petitioner alleges that the respondent
violated the Article 14, Paragraphs 1 and 5 of Code of Commerce in purchasing a
property which was litigated under his court and selling it to Traders Manufacturing
and Fishing Industries, Inc. as a stockholder and President while he was a judge of
the Court of First Instance of Leyte.

ISSUE:
Whether or not the respondent Judge Asuncion violated paragraphs 1 and 5, Article
14 of the Code of Commerce

RULING:
NO, the respondent did not violate paragraphs 1 and 5, Article 14 of the Code of
Commerce.
- The provision of the Code of Commerce states that judges in active service
cannot engage in commerce, either in person or by proxy, nor can they hold any office
or have any connection to companies in the areas in which they discharge their duties.
- However, this code of commerce although part of commercial laws takes more
of the nature of a political law (administrative law) as it regulates the relationship
between the government and certain public officers and,u employees like justices and
judges
- The present Code of Commerce is the Spanish Code of Commerce of 1885
which, being a political law, must be deemed to be abrogated because where there is
change in sovereignty the political laws of the former sovereign, whether compatible or
not with those of the new sovereign, are automatically abrogated, unless they are
expressly re-enacted by affirmative act of the new sovereign.
MONTEBON VS. THE DIRECTOR OF PRISONS
LIM, MA. THERESA
TOPIC: EFFECTIVITY
FACTS: Alfonso Montebon files a petition for habeas corpus on behalf of prisoner
Elpidio S. Cruz convicted of 3 counts of estafa. Petitioner is contesting the validity of
the recommitment decreed by the Commissioner of Justice of the Philippine Executive
Commission under date of June 3, 1943, for the unexpired portion prisoner's
maximum aggregate sentences in three cases in which he had been paroled by the
Board of Indeterminate Sentence on June 26, 1941, when he still had over five years
to serve. The commissioner of Justice's recommitment order was made by virtue of
Administrative Order No. 21, dated June 21, 1942, and approved by the Chairman of
the Executive Commission, which read: "The Board of Indeterminate Sentence and
the Board of Pardons having been abolished, the powers, duties and functions thereof
shall henceforth be assumed and exercised by the Commissioner of Justice."
Petitioner takes the position that the recommitment of which he complains was not
such an act of the belligerent occupant as should be accorded respect and recognition
by the Commonwealth Government, now Republic of the Philippines, after the
cessation of the enemy occupation.

ISSUE: Whether or not the recommitment order of the commissioner of Justice is valid
when prisoner had already been paroled before the Japanese occupation.

RULING: YES. The recommitment order of the commissioner of Justice is valid.


The authority of the Commissioner of Justice under the then existing government,
laws, and military, executive and administrative orders, to take over the powers,
functions and duties of the Board of Indeterminate Sentence, is beyond dispute. In an
international sense it matters not how the Commissioner of Justice was vested with
that authority. For most purposes the government of the occupant is likely to exercise
the lawmaking functions through decrees or regulations emanating from a military
source; and these become as effective in operation as though they were expressed in
statutory enactments.

As to whether the Indeterminate Sentence Act was in force during the occupation, the
answer is in the affirmative. A proclamation of the Commander-in-Chief of the
Japanese forces of January 2, 1942, directed that "so far as the military administration
permits, all the laws now in force in the Commonwealth, as well as executive and
judicial institutions, shall continue to be effective for the time being as in the past." This
was nothing more than a confirmation of the well-known rule of the Law of Nations that
municipal laws, as contra-distinguished from laws of political nature, are not abrogated
by a change of sovereignty. The Indeterminate Sentence Law is not a political law. It
does not affect political relations. In fact, it is a part of the Commonwealth's criminal
and penal system directly related to the punishment of crime and the maintenance of
public peace and order, which Article 43 of Section III of the Hague Regulations of
1907 compels the belligerent occupant to take all steps in his power to reestablish and
insure as far as possible. The fact that the territory which has been occupied by an
enemy comes again into the power of its legitimate government or sovereignty, does
not, except in a very few cases, wipe out the effects of acts done by the invader, which
for one reason or another it is within his competence to do.
Enforcement of the criminal law by the forces of occupation is not only valid and
binding; it is imposed on them as a high obligation by the Hague Convention.
The petition is denied without costs.

VALENCIA VS. LOCQUIAO


YU, MID (updated by Balanay, Danica)
TOPIC: Effectivity of Laws

FACTS: On May 22, 1944, Herminigildo and Raymunda Locquiao executed a deed of
donation proper nuptias written in Ilocano as Inventario Ti Sagut for their son Benito
and his wife Tomasa.
June 4, 1944: Benito and Tomasa took their marriage vows and the fact of their
marriage was inscribed at the back of O.C.T. No. 18383
May 15, 1970: Inventario Ti Sagut was registered with Office of the Register of Deeds
of Pangasinan and eventually the original title was cancelled and in lieu thereof,
Transfer Certificate of Title No. 84897 was issued in the name of Benito and Tomasa
March 18, 1973: Heirs of the Locquiao spouses, including respondent Benito and
petitioner Romana, executed a Deed of Partition with Recognition of Rights (stating
that Benito, Marciano, and the heirs Lucio Locquiao "have already received our shares
in the estates of our parents, by virtue of previous donations and conveyances,"
June 12, 1976: Deed of Compromise Agreement was issued to settle differences
concerning the distribution of 2 of the lots covered by the deed of partition (Lots 2467
and 5567)
1983: Constancia filed an action for annulment of title against the respondents

ISSUE: Whether or not the donation propter nuptias is valid in the absence of a written
acceptance of the document itself or in a separate public instrument under the New
Civil Code

RULING: Yes, it is valid. It is settled that only laws existing at the time of the execution
of a contract are applicable thereto and not later statutes, unless the latter are
specifically intended to have retroactive effect. Consequently, it is the Old Civil Code
which applies in this case since the donation propter nuptias was executed in 1944
and the New Civil Code took effect only on August 30, 1950. The fact that in 1944 the
Philippines was still under Japanese occupation is of no consequence. It is a
well-known rule of the Law of Nations that municipal laws, as contra-distinguished
from laws of political nature, are not abrogated by a change of sovereignty. This Court
specifically held that during the Japanese occupation period, the Old Civil Code was in
force. As a consequence, applying Article 1330 of the Old Civil Code in the
determination of the validity of the questioned donation, it does not matter whether or
not the donees had accepted the donation. The validity of the donation is unaffected in
either case.
In the Old Civil Code (Civil Code of Spain), donations propter nuptias must be made in
a public instrument in which the property donated must be specifically described.
However, Article 1330 of the same code provides that "acceptance is not necessary to
the validity of such gifts". In the New Civil Code, Article 129 provides that express
acceptance "is not necessary for the validity of these donations." Thus, implied
acceptance is sufficient.

With respect to whether the donation propter nuptias falls under Old Civil code of the
New Civil Code, it falls under the Old Civil Code since the donation propter nuptias
was executed in 1944 and the New Civil Code took effect only on August 30,1950.

SJS VS. DANGEROUS DRUGS BOARD


Digested by: Sophia Amor C. Bersamin

Topic: Constitutionality of Section 36 of Republic Act No. 9165, or the


Comprehensive Dangerous Drugs Act of 2002

Facts:Section 36 of Republic Act No. 9165, or the Comprehensive Dangerous Drugs Act of 2002 requires
mandatory drug testing of candidates for public office, students of secondary and tertiary schools, officers
and employees of public and private offices, and persons charged before the prosecutor's office with
certain offenses, among other personalities.

G.R. No. 158633 (Atty. Manuel J. Laserna, Jr. vs DDB and PDEA)

Petitioner Atty. Manuel J. Laserna, Jr., as citizen and taxpayer, also seeks in his petition for certiorari and
prohibition under Rule 65 that Sec. 36(c), (d), (f), and (g) of RA 9165 be struck down as unconstitutional
for infringing on the constitutional right to privacy, the right against unreasonable search and seizure, and
the right against self - incrimination, and for being contrary to the due process and equal protection
guarantees.

G.R. No. 161658 (Aquilino Q. Pimentel, Jr. v. Commission on Elections)

On Dec. 23, 2003, the COMELEC issued Resolution No. 6486, prescribing the rules and regulations for
the mandatory drug testing of candidates for public office in connection with the May 2004 elections.
Pimentel claims that Sec. 36 (g) of RA 9165 and COMELEC Resolution No. 6486 illegally impose an
additional qualification on candidates for senator. He points out that, subject to the provisions on nuisance
candidates, a candidate for senator only needs to meet the qualifications laid down in Sec. 3, Art. VI of
the Constitution, to wit: (1) citizenship, (2) voter registration, (3) literacy, (4) age, and (5) residency.
Beyond these stated qualification requirements, candidates for senator need not possess any other
qualification to run for the said position and be voted upon and elected as member of the Senate. The
Congress cannot validly amend or otherwise modify these qualification standards, as it cannot disregard,
evade, or weaken the force of a constitutional mandate, or alter or enlarge the Constitution.

Issue:

1) Are paragraphs (c), (d), (f), and (g) of Sec. 36, RA 9165 unconstitutional? Specifically, do these
paragraphs violate the right to privacy, the right against unreasonable searches and seizure, and the
equal protection clause?
2) Do Sec. 36 (g) of RA 9165 and COMELEC Resolution No. 6486 impose an additional qualification for
candidates for senator? Can Congress enact a law prescribing qualifications for candidates for senator in
addition to those laid down by the Constitution?

Ruling:

1) The Court held that paragraphs (c) and (d) are CONSTITUTIONAL; while paragraphs (f) and
(g) are UNCONSTITUTIONAL. Only paragraphs (f) and (g) violate the right to privacy, the right
against unreasonable searches and seizure, and the equal protection clause.

2) YES, Section 36 (g) of RA 9165 and COMELEC Resolution No. 6486 impose an additional
qualification for candidates for senator. NO, Congress cannot enact a law prescribing qualifications for
candidates for senator in addition to those laid down by the Constitution. The Court held that, it is basic
that if a law or an administrative rule violates any norm of the Constitution, that issuance is null and void
and has no effect. The Constitution is the basic law to which all laws must conform; no act shall be valid if
it conflicts with the Constitution.

SABIO VS. GORDON


Digested By: Alyana Mahilum

TOPIC: Doctrine of Constitutional Supremacy

FACTS:

Pursuant to Senate Resolution No. 455, Senator Gordon requested PCGG Chairman Sabio and his
Commissioners to appear as resource persons in the public meeting jointly conducted by the Committee
on Government Corporations and Public Enterprises and Committee on Public Services.

Chairman Sabio declined the invitation because of prior commitment, and at the same time invoked
Section 4(b) of EO No. 1: “No member or staff of the Commission shall be required to testify or produce
evidence in any judicial, legislative or administrative proceeding concerning matters within its official
cognizance.”

ISSUE:

Whether or not Section 4(b) of E.O. No.1 limits power of legislative inquiry by exempting all PCGG
members or staff from testifying in any judicial, legislative or administrative proceeding.

RULING:

No. Article VI, Section 21 of the 1987 Constitution grants the power of inquiry not only to the Senate and
the House of Representatives, but also to any of their respective committees. Clearly, there is a direct
conferral of investigatory power to the committees and it means that the mechanisms which the Houses
can take in order to effectively perform its investigative functions are also available to the committees.

It can be said that the Congress’ power of inquiry has gained more solid existence and expansive
construal. The Court’s high regard to such power is rendered more evident in Senate v. Ermita, where
it categorically ruled that “the power of inquiry is broad enough to cover officials of the executive branch.”
Verily, the Court reinforced the doctrine in Arnault that “the operation of government, being a legitimate
subject for legislation, is a proper subject for investigation” and that “the power of inquiry is co-extensive
with the power to legislate.”
Considering these jurisprudential instructions, Section 4(b) is directly repugnant with Article VI, Section
21. Section 4(b) exempts the PCGG members and staff from the Congress’ power of inquiry. This cannot
be countenanced. Nowhere in the Constitution is any provision granting such exemption. The Congress’
power of inquiry, being broad, encompasses everything that concerns the administration of existing laws
as well as proposed or possibly needed statutes. It even extends “to government agencies created by
Congress and officers whose positions are within the power of Congress to regulate or even abolish.”
PCGG belongs to this class.
A statute may be declared unconstitutional because it is not within the legislative power to enact; or it
creates or establishes methods or forms that infringe constitutional principles; or its purpose or effect
violates the Constitution or its basic principles.

Moreover, Sec. 4(b) of E.O. No. 1 has been repealed by the Constitution because it is inconsistent with
the constitutional provisions on the Congress’ power of inquiry (Art. VI, Sec. 21), the principle of public
accountability (Art. XI, Sec. 1), the policy of full disclosure (Art. II, Sec. 28), and the right of access to
public information (Art. III, Sec. 7).

Certainly, a mere provision of law cannot pose a limitation to the broad power of Congress, in the
absence of any constitutional basis.
ATTY. MACALINTAL VS. COMMISSION ON ELECTIONS
Digest by: Benjamin A. Misoles Jr.

TOPIC: Doctrine of Constitutional Supremacy

FACTS:

This case is a petition for certiorari and prohibition filed by Romulo B. Macalintal, a
member of the Philippine Bar, seeking a declaration that certain provisions of Republic
Act No. 9189 (The Overseas Absentee Voting Act of 2003) suffer from constitutional
infirmity.

ISSUE:

Whether or not R.A. 9189 violates the Constitution.

RULING:

It does not violate the Constitution. The Constitution is the fundamental and
paramount law of the nation to which all other laws must conform and in accordance
with which all private rights must be determined and all public authority administered.
Laws that do not conform to the Constitution shall be stricken down for being
unconstitutional. Generally, however, all laws are presumed to be constitutional. To
declare a law unconstitutional, the repugnancy of that law to the Constitution must be
clear and unequivocal, for even if a law is aimed at the attainment of some public
good, no infringement of constitutional rights is allowed. To strike down a law there
must be a clear showing that what the fundamental law condemns or prohibits, the
statute allows it to be done.

As the essence of R.A. No. 9189 is to enfranchise overseas qualified Filipinos, it


behooves the Court to take a holistic view of the pertinent provisions of both the
Constitution and R.A. No. 9189. It is a basic rule in constitutional construction that the
Constitution should be construed as a whole R.A. No. 9189 was enacted in obeisance
to the mandate of the first paragraph of Section 2, Article V of the Constitution that
Congress shall provide a system for voting by qualified Filipinos abroad. It must be
stressed that Section 2 does not provide for the parameters of the exercise of
legislative authority in enacting said law. Hence, in the absence of restrictions,
Congress is presumed to have duly exercised its function as defined in Article VI (The
Legislative Department) of the Constitution.
MANILA PRINCE HOTEL VS. GSIS
Digested by: Sophia Amor C. Bersamin

Topic: Section 10(2) of Article 12 of the 1987 Constitution - Filipino First Policy/Constitutional
Supremacy

Facts:

The Government Service Insurance System (GSIS) decided to sell through public bidding 30% to 51% of
the issued and outstanding shares of respondent Manila Hotel Corporation (MHC). The winning bidder, or
the eventual "strategic partner," is to provide management expertise and/or an international
marketing/reservation system, and financial support to strengthen the profitability and performance of the
Manila Hotel. Two (2) bidders participated: petitioner Manila Prince Hotel Corporation, a Filipino
corporation, which offered to buy 51% of the MHC or 15,300,000 shares at P41.58 per share, and
Renong Berhad, a Malaysian firm, with ITT-Sheraton as its hotel operator, which bid for the same number
of shares at P44.00 per share, or P2.42 more than the bid of petitioner. Pending the declaration of
Renong Berhad as the winning bidder/strategic partner and the execution of the necessary contracts,
petitioner sent a letter to respondent GSIS, matching the bid price of P44.00 per share tendered by
Renong Berhad. The petitioner also sent a manager’s check issued by PhilTrust Bank for thirty-three
million pesos (P33,000,000.00) as Bid Security to match the bid of the Renong Berhad. But respondent
GSIS refused to accept the petitioner’s offer to match the bid price of Renong Berhad.

GSIS continued to award the bid and sale of the share to Renong Berhad. Manila Prince Hotel
invoked Section 10(2) of Article 12 of the 1987 Constitution and argued that it must be preferred
after it matched the competitor’s bid offer. The provision states, “In the grant of rights, privileges,
and concessions covering national economy and patrimony, the State shall give preference to
qualified Filipinos.”

It submits that the Manila Hotel has been identified with the Filipino nation and has practically becomes a
historical monument, which reflects the vibrancy of Philippine heritage and culture. Therefore, it has
become a part of the national patrimony. In addition, the shares of Manila Hotel that GSIS owns is part of
the national economy. On the other hand, GSIS contended that Section 10 is merely a statement of
principle and policy as it is not a self-executing of provision. In order to become an enforceable right and
to operate, there must be existing laws to lay down conditions under which business may be done. They
also argued that non-self-executing of provision in question is implied from the tenor of the first and third
paragraph of the same section, which are not self-executing. The respondent also argued that Manila
Hotel is not part of national patrimony, and the Constitution’s mandate is not applicable to GSIS because
it possess a personality of its own separate and distinct from the Philippines as a state. In addition, the
provision is not applicable since what was being sold is 51% of the outstanding shares, not the hotel
building or land. Equity cannot be part of the national patrimony. Thus, Manila Prince Hotel has no legal
right to petition mandamus.

Issue:

1) Whether or not Manila Prince Hotel should be awarded the 51% of shares in accordance to Sec.
10(2) Art.12, of the 1987 Constitution/Filipino First Policy

Ruling:

YES. The argument of respondents that petitioner is now estopped from questioning the sale to Renong
Berhad since petitioner was well aware from the beginning that a foreigner could participate in the bidding
is meritless. Foreigners may be awarded the sale only if no Filipino qualifies, or if the qualified Filipino
fails to match the highest bid tendered by the foreign entity. Only after it had matched the bid of the
foreign firm and the apparent disregard by respondent GSIS of petitioner's matching bid did the latter
have a cause of action. The Filipino First Policy is a product of Philippine nationalism. It is embodied in
the 1987 Constitution not merely to be used as a guideline for future legislation but primarily to be
enforced; so must it be enforced.

CIR VS. SAN ROQUE POWER CORP.


NAME OF DIGESTER
TOPIC:

FACTS:

This Resolution resolves the Motion for Reconsideration and the Supplemental Motion for
Reconsideration filed by San Roque Power Corporation (San Roque) in G.R. No. 187485, the Comment to
the Motion for Reconsideration filed by the Commissioner of Internal Revenue (CIR) in G.R. No. 187485,
the Motion for Reconsideration filed by the CIR in G.R.No. 196113, and the Comment to the Motion for
Reconsideration filed by Taganito Mining Corporation (Taganito) in G.R. No. 196113.

San Roque prays that the rule established in our 12 February 2013 Decision be given only a prospective
effect, arguing that "the manner by which the Bureau of Internal Revenue (BIR) and the Court of Tax
Appeals(CTA) actually treated the 120 + 30 day periods constitutes an operative fact the effects and
1
consequences of which cannot be erased or undone."

The CIR, on the other hand, asserts that Taganito Mining Corporation's (Taganito) judicial claim for tax
credit or refund was prematurely filed before the CTA and should be disallowed because BIR Ruling No.
DA-489-03 was issued by a Deputy Commissioner, not by the Commissioner of Internal Revenue.

ISSUE:

WON an unconstitutional act, for that matter an executive order or a municipal ordinance
likewise suffering from that infirmity, cannot be the source of any legal rights or duties.

RULING:

NO

In Serrano de Agbayani v. Philippine National Bank,3 the application of the doctrine of operative fact was
discussed as follows:

As the new Civil Code puts it: "When the courts declare a law to be inconsistent with the Constitution, the
former shall be void and the latter shall govern. Administrative or executive acts, orders and regulations shall
be valid only when they are not contrary to the laws of the Constitution." It is understandable why it should
be so, the Constitution being supreme and paramount. Any legislative or executive act contrary to its terms
cannot survive.
FRANCISCO JR. VS. THE HOUSE OF
REPRESENTATIVES
NAME OF DIGESTER
TOPIC:
FACTS: There may indeed be some legitimacy to the characterization that the present controversy
subject of the instant petitions – whether the filing of the second impeachment complaint against Chief Justice
Hilario G. Davide, Jr. with the House of Representatives falls within the one year bar provided in the Constitution,
and whether the resolution thereof is a political question – has resulted in a political crisis. Perhaps even more truth
to the view that it was brought upon by a political crisis of conscience.

On July 22, 2002, the House of Representatives adopted a Resolution,2 sponsored by Representative Felix
William D. Fuentebella, which directed the Committee on Justice "to conduct an investigation, in aid of
legislation, on the manner of disbursements and expenditures by the Chief Justice of the Supreme Court of
the Judiciary Development Fund (JDF)."3

On June 2, 2003, former President Joseph E. Estrada filed an impeachment complaint4 (first impeachment
complaint) against Chief Justice Hilario G. Davide Jr. and seven Associate Justices5 of this Court for
"culpable violation of the Constitution, betrayal of the public trust and other high crimes."

The House Committee on Justice ruled on October 13, 2003 that the first impeachment complaint was
"sufficient in form,"9 but voted to dismiss the same on October 22, 2003 for being insufficient in
substance.10 To date, the Committee Report to this effect has not yet been sent to the House in plenary in
accordance with the said Section 3(2) of Article XI of the Constitution.
Four months and three weeks since the filing on June 2, 2003 of the first complaint or on October 23,
2003, a day after the House Committee on Justice voted to dismiss it, the second impeachment complaint11
was filed with the Secretary General of the House12 by Representatives Gilberto C. Teodoro, Jr. (First
District, Tarlac) and Felix William B. Fuentebella (Third District, Camarines Sur) against Chief Justice
Hilario G. Davide, Jr., founded on the alleged results of the legislative inquiry initiated by
above-mentioned House Resolution. This second impeachment complaint was accompanied by a
"Resolution of Endorsement/Impeachment" signed by at least one-third (1/3) of all the Members of the
House of Representatives.13

Thus arose the instant petitions against the House of Representatives, et. al., most of which petitions contend that
the filing of the second impeachment complaint is unconstitutional as it violates the provision of Section 5 of Article
XI of the Constitution that "[n]o impeachment proceedings shall be initiated against the same official more than
once within a period of one year.

ISSUE: WON the filing of the second impeachment complaint is unconstitutional as it violates the
provision of Section 5 of Article XI of the Constitution that "[n]o impeachment proceedings shall be initiated
against the same official more than once within a period of one year.
RULING: YES

In furthering their arguments on the proposition that impeachment proceedings are outside the scope of
judicial review, respondents Speaker De Venecia, et. al. and intervenor Senator Pimentel rely heavily on
American authorities, principally the majority opinion in the case of Nixon v. United States.
SERRANO VS. GALLANT MARITIME
Digested by: Sophia Amor C. Bersamin

Topic: Constitutionality of the 5th paragraph of Section 10 of RA 8042.

Facts:

Respondents Gallant Maritime Services, Inc. and Marlow Navigation Co., Inc. hired Petitioner Antonio
Serrano under a POEA-approved contract of employment for 12 months, as Chief Officer. Upon
departure, Serrano was forced to accept a downgraded employment, contrary to the contract, upon the
assurance and representation of respondents that he would be Chief Officer by the end of April 1998.
However, Respondent’s promise to make Serrano Chief Officer was not fulfilled. Petitioner Serrano
refused to stay on as Second Officer. He was sent back to the Philippines, serving only two months and 7
days, leaving an unexpired portion of nine months and twenty-three days. Upon complaint filed by
Petitioner Serrano before the Labor Arbiter (LA), the dismissal was declared illegal. The case was
appealed and the NLRC modified LA decision based on the provision of RA 8042. Petitioner Serrano filed
a motion questioning the constitutionality of the last clause in the 5th paragraph of Section 10 of RA 8042.

RA 8042 reads:

“In case of termination of overseas employment without just, valid or authorized cause as defined by law
or contract, the workers shall be entitled to the full reimbursement of his placement fee with interest of
twelve percent (12%) per annum, plus his salaries for the unexpired portion of his employment contract or
for three (3) months for every year of the unexpired term, whichever is less.

Issue:

1. W/N the subject clause violates Section 10, Article III of the Constitution on non-impairment of
contracts;

2. W/N the subject clause violates Section 1, Article III of the Constitution, and Section 18, Article
II and Section 3, Article XIII on labor as a protected sector.

Ruling:

1) NO, it does not violate the principle of non-impairment of contract. Petitioner’s claim that the subject
clause interferes with the stipulations in his contract on the term of his employment and the fixed salary
package he will receive is not well-founded.

It does not infringe on the impairment clause, for the law was enacted in the exercise of the police power
of the State to regulate a business, profession or calling, particularly the recruitment and deployment of
OFWs, to ensure respect for the dignity and well-being of OFWs wherever they may be employed.

2) YES, the subject clause violates the Equal Protection Clause and Right of an individual to due
Process, pursuant to Section 1, Article III of the Constitution, and Section 18, Article II and Section 3,
Article XIII. The rights under the foregoing constitutional provisions translating to economic security and
parity are guaranteed for Filipino workers. The subject clause appears neutral, for it applies to all OFWs.
However, further scrutiny reveals that the subject clause has a discriminatory intent against OFWs
because of the varying and nonuniform nature of contracts available for them. The subject clause singles
out one classification of OFWs and burdens it with a peculiar disadvantage. Thus, the subject clause in
the 5th paragraph of Section 10 of R.A. No. 8042 is violative of the right of petitioner and other OFWs to
equal protection. The subject clause “or for three months for every year of the unexpired term, whichever
is less” in the 5th paragraph of Section 10 of Republic Act No. 8042 is DECLARED
UNCONSTITUTIONAL.

DATU MICHAEL ABAS KIDA vs. SENATE


Digested by: Bajo, Ralf

Topic: Qualities of a Good Constitution: BROAD, BRIEF, DEFINITE

FACTS: These motions assail our Decision dated October 18, 2011, where we upheld
the constitutionality of Republic Act (RA) No. 10153. Pursuant to the constitutional
mandate of synchronization, RA No. 10153 postponed the regional elections in the
Autonomous Region in Muslim Mindanao (ARMM) (which were scheduled to be held
on the second Monday of August 2011) to the second Monday of May 2013 and
recognized the President’s power to appoint officers-in-charge (OICs) to temporarily
assume these positions upon the expiration of the terms of the elected officials.

ISSUE: Whether or not the ARMM elections are covered by the constitutional
mandate of synchronization of elections under Article 18, Section 5 of the Constitution.

RULING: That the ARMM elections were not expressly mentioned in the Transitory
Provisions of the Constitution on synchronization cannot be interpreted to mean that
the ARMM elections are not covered by the constitutional mandate of synchronization.
We have to consider that the ARMM, as we now know it, had not yet been officially
organized at the time the Constitution was enacted and ratified by the people.
Keeping in mind that a constitution is not intended to provide merely for the
exigencies of a few years but is to endure through generations for as long as it
remains unaltered by the people as ultimate sovereign, a constitution should be
construed in the light of what actually is a continuing instrument to govern not
only the present but also the unfolding events of the indefinite future. Although
the principles embodied in a constitution remain fixed and unchanged from the
time of its adoption, a constitution must be construed as a dynamic process
intended to stand for a great length of time, to be progressive and not static.

—--------------------------------------------

Supplement: Constitutional Issues in the Synchronization of National and Local


Elections by Sen. Miriam Defensor-Santiago
(https://legacy.senate.gov.ph/press_release/2011/0606_santiago2.asp)
The Constitution does not explicitly provide that national and local elections shall be
synchronized. Instead, the Constitution under Article 18, Transitory Provisions, Section
5, uses the phrase "for purposes of synchronization of elections." Thus, the
Constitution implicitly places constitutional value on synchronized elections. By
constitutional value, I mean that the Constitution recognizes the significance,
desirability, or utility of synchronized elections to the general public.

TA​ÑADA VS. ANGARA


Digested by:
TOPIC:

FACTS:

ISSUE:

RULING:
MARCOS VS. MANGLAPUS
Digested by: Allysson Reveche

TOPIC: Philosophical View of the Constitution (as a social contract)

FACTS: Former President Ferdinand Marcos was deposed from the presidency via
the non-violent “people power” revolution and was forced into exile. Marcos, in his
deathbed, had signified his wish to return to the Philippines to die. However, President
Cory Aquino, considering the dire consequences to the nation of his return at a time
when the stability of government is threatened from various directions and the
economy is just beginning to rise and move forward, has stood firmly on the decision
to bar the return of Marcos and his family.

Marcos filed for petition of mandamus prohibition to order the respondents to issue
them their travel documents and prevent the implementation of President Aquino’s
decision to bar them from returning to the Philippines. Petitioner questions Aquino’s
power to bar his return to the country. According to the Marcoses, such an act
deprives them of their right to life, liberty, property, and due process and equal
protection of laws. They also add that it deprives them of their right to travel which
according to Section 6, Article 3 of the 1987 Constitution, may only be imparied by a
court order.

ISSUE: Whether or not, in the exercise of executive power, the President may prohibit
the Marcoses from returning to the Philippines

RULING: Yes.

Faced with the problem of whether or not the time is right to allow the Marcoses to
return to the Philippines, the President is, under the constitution, constrained to
consider these basic principles in arriving at a decision. More than that, having sworn
to defend and uphold the Constitution, the President has the obligation under the
Constitution to protect the people, promote their welfare, and advance national
interest.

It must be borne in mind that the Constitution, aside from being an allocation of
power, is also a social contract, whereby the people have surrendered their
sovereign powers to the state for the common good. Hence, lest the officers of the
government exercising the powers delegated by the people forget, and the servants of
the people become rulers, the constitution reminds everyone that “sovereignty resides
in the people and all government authority emanates from them.”
II. BACKGROUND OF THE PRESENT
CONSTITUTION
ESTRADA VS. DESIERTO
NAME OF DIGESTER
TOPIC:
FACTS:
ISSUE:
RULING:
LAWYERS’ LEAGUE VS. AQUINO
ABRYL SYMON CULAS

TOPIC: PROVISIONAL CONSTITUTION

FACTS: ​On February 25, 1986, President Corazon Aquino issued Proclamation No. 1
announcing that she and Vice President Laurel were taking power. On March 25,
1886, Proclamation No. 3 was issued providing the basis of the Aquino government
assumption of power by stating that the “New Government” was installed through a
direct exercise of the power of the Filipino people assisted by units of the New Armed
Forces of the Philippines.

ISSUE: Whether or not the government of Corazon Aquino is legitimate.

RULING: The legitimacy of the Aquino government is not a justiciable matter. It


belongs to the realm of politics where only the people of the Philippines are the judge.
And the people have made the judgment; they have accepted the government of
President Corazon C. Aquino.

For the above-quoted reason, which are fully applicable to the petition at bar, mutatis
mutandis, there can be no question that President Corazon C. Aquino and
Vice-President Salvador H. Laurel are the incumbent and legitimate President and
Vice-President of the Republic of the Philippines. or the above-quoted reasons, which
are fully applicable to the petition at bar.
IN RE: SATURNINO V. BERMUDEZ
ABRYL SYMON D. CULAS

TOPIC: PROVISIONAL CONSTITUTION

FACTS: NO FACTS

ISSUE:

RULING: Petitioners have no personality to sue and their petitions state no cause of
action. For the legitimacy of the Aquino government is not a justiciable matter. It
belongs to the realm of politics where only the people of the Philippines are the judge.
And the people have made the judgment; they have accepted the government of
President Corazon C. Aquino which is in effective control of the entire country so that
it is not merely a de facto government but in fact and law a de jure government.
Moreover, the community of nations has recognized the legitimacy of tlie present
government. All the eleven members of this Court, as reorganized, have sworn to
uphold the fundamental law of the Republic under her government.
DE LEON VS. ESGUERRA
ABRYL SYMON D. CULAS

TOPIC:

FACTS: ​Petitioner Alfredo M. De Leon was elected Barangay Captain and the other
petitioners as Barangay Councilmen under Batas Pambansa Blg. 222, otherwise
known as the Barangay Election Act of 1982.

Respondent OIC Governor signed a Memorandum ordering the replacement of all the
barangay officials of all the barangay(s) in the Municipality of Taytay, Rizal
—Designating respondents as members of the Barangay Council,

Petitioners maintain that pursuant to Section 3 of the Barangay Election Act of 1982
(BP Blg. 222), their terms of office "shall be six (6) years. Thus petitioners pray that
subject memoranda be declared null and void and that respondents be prohibited from
taking over their positions.

Respondents contend that the terms of office of elective and appointive officials were
abolished and that the provision in the Barangay Election Act fixing the term of office
of Barangay officials to six (6) years must be deemed to have been repealed for being
inconsistent with the Section 2, Article III of the Provisional Constitution.

SECTION 2. All elective and appointive officials and employees under the 1973
Constitution shall continue in office until otherwise provided by proclamation or
executive order or upon the designation or appointment and qualification of their
successors, if such appointment is made within a period of one year from February
25,1986.

ISSUE: Whether or not the designation of respondents to replace petitioners was


validly made.

RULING: NO. Having become inoperative, respondent OIC Governor could no longer
rely on Section 2, Article III of the Provisional Constitution, thereof to designate
respondents to the elective positions occupied by petitioners.

The Memoranda issued by respondent OIC Governor are both declared to be of no


legal force and effect.
TAÑADA VS. TUVERA
ABRYL SYMON D. CULAS

TOPIC:

FACTS: Petitioners seek to compel respondent public officials to publish, and/or


cause the publication in the Official Gazette of various presidential decrees, letters of
instructions, general orders, proclamations, executive orders, letter of implementation
and administrative orders.

Respondents contend that publication in the Official Gazette is not a sine qua non
requirement for the effectivity of laws where the laws themselves provide for their own
effectivity dates.

ISSUE: Whether or not such presidential issuances require publication for their
effectivity

RULING: YES

The clear object of the Section 1 of Commonwealth Act 638 is to give the general
public adequate notice of the various laws which are to regulate their actions and
conduct as citizens. Without such notice and publication, there would be no basis for
the application of the maxim "ignorantia legis non excusat." It would be the height of
injustice to punish or otherwise burden a citizen for the transgression of a law of which
he had no notice whatsoever
REPUBLIC VS. SANDIGANBAYAN, RAMAS AND
DIMAANO
ABRYL SYMON D. CULAS

TOPIC:

FACTS: Dimaano claimed ownership of the monies, communications equipment,


jewelry and land titles taken from her house by the Philippine Constabulary raiding
team. AFP anti-graft Board mandated this and in lieu of the task of the PCGG to
recover all ill-gotten wealth of former President Ferdinand E. Marcos, his immediate
family, relatives, subordinates and close associates.

ISSUE: Whether the revolutionary government was bound by the Bill of Rights of the
1973 Constitution interregnum, that is, after the actual and effective take-over of
power by the revolutionary government?

RULING: YES. The Provisional Constitution adopted verbatim the Bill of Rights of the
1973 Constitution. We hold that the Bill of Rights under the 1973 Constitution was not
operative during the interregnum. However, we rule that the protection accorded to
individuals under the Covenant and the Declaration remained in effect during the
interregnum.

The seizure of these items was therefore void, and unless these items are contraband
per se, and they are not, they must be returned to the person from whom the raiding
seized them. However, we do not declare that such person is the lawful owner of
these items, merely that the search and seizure warrant could not be used as basis to
seize and withhold these items from the possessor. We thus hold that these items
should be returned immediately to Dimaano.
III. JUDICIAL ELABORATION OF THE
CONSTITUTION
MANILA PRINCE HOTEL VS. GSIS
TOPIC: THE CONSTITUTION SHOULD BE CONSIDERED SELF-EXECUTING RATHER
THAN NON-SELF-EXECUTING

FACTS: The Government Service Insurance System (GSIS) decided to sell through public
bidding 30% to 51% of the issued and outstanding shares of the Manila Hotel (MHC).

In a close bidding, two bidders participated: Manila Prince Hotel Corporation (MPHC), a
Filipino corporation, which offered to buy 51% of the MHC at P41.58 per share, and Renong
Berhad, a Malaysian firm, with ITT-Sheraton as its hotel operator, which bid for the same
number of shares at P44.00 per share, or P2.42 more than the bid of petitioner.

Pending the declaration of Renong Berhard as the winning bidder and the execution of the
contracts, the MPHC matched the bid price in a letter to GSIS. MPHC sent a manager’s check
to the GSIS in a subsequent letter, which GSIS refused to accept. On 17 October 1995, perhaps
apprehensive that GSIS has disregarded the tender of the matching bid, MPHC came to the
Court on prohibition and mandamus.

Petitioner invokes Sec. 10, second par., Art. XII, of the 1987 Constitution and submits that the
Manila Hotel has been identified with the Filipino nation and has practically become a
historical monument which reflects the vibrancy of Philippine heritage and culture.

Respondents assert that Sec. 10, second par., Art. XII, of the 1987 Constitution is merely a
statement of principle and policy since it is not a self-executing provision and requires
implementing legislation(s).

ISSUE: Whether the provisions of the Constitution, particularly Article XII Section 10, are
self-executing.

RULING: Yes. Sec 10, Art. XII of the 1987 Constitution is a self-executing provision.

A provision which lays down a general principle, such as those found in Article II of the 1987
Constitution, is usually not self-executing. But a provision which is complete in itself and
becomes operative without the aid of supplementary or enabling legislation, or that which
supplies sufficient rule by means of which the right it grants may be enjoyed or protected, is
self-executing.

Hence, unless it is expressly provided that a legislative act is necessary to enforce a


constitutional mandate, the presumption now is that all provisions of the constitution are
self-executing. If the constitutional provisions are treated as requiring legislation instead of
self-executing, the legislature would have the power to ignore and practically nullify the
mandate of the fundamental law.

In fine, Section 10, second paragraph, Art. XII of the 1987 Constitution is a mandatory,
positive command which is complete in itself and which needs no further guidelines or
implementing laws or rules for its enforcement. From its very words the provision does not
require any legislation to put it in operation.
GAMBOA VS. TEVES
ABRYL SYMON D. CULAS

TOPIC:

FACTS: This case involves the alleged unconstitutional current ownership structure of
PLDT. In the case the term "capital" is to be decided if it involves both preferred and
common stocks as Petitioner Gamboa prays before the courts that ownership of
common or voting shares is the sole basis in determining foreign equity in a public
utility and that any other government rulings, opinions, and regulations inconsistent
with such be declared unconstitutional and a violation of the intent and spirit of the
1987 Constitution

ISSUE: Whether Section 11, Article XII of the 1987 Constitution is self-executing

RULING: No. Section 11 Article XII of the 1987 Constitution is one of the constitutional
provisions that are not self-executing and need sufficient details for a meaningful
implementation. While the provision states that no franchise for the operation of a
public utility shall be granted to a corporation organized under Philippine laws unless
at least 60% of its capital is owned by Filipino citizens, it does not provide for the
meaning of the term "capital."

The meaning of "capital," must then be interpreted to encompass the entirety of a


corporation’s outstanding capital stock (both common and preferred shares, voting or
non-voting).

The Constitution expressly declares as State policy the development of an economy


"effectively controlled" by Filipinos.
TA​​ÑADA VS. ANGARA
NAME OF DIGESTER
TOPIC:
FACTS:
ISSUE:
RULING:
PAMATONG VS. COMELEC

FACTS: When the petitioner, Rev. Elly Velez Pamatong, filed his Certificate of Candidacy for
Presidency, the Commision on Elections (COMELEC) refused to give the petition its due course.
Pamatong requested a case for reconsideration. However, the COMELEC again denied his
request. The COMELEC declared Pamatong, along with 35 other people, as nuisance
candidates, as stated in the Omnibus Election Code. The COMELEC noted that such candidates
“could not wage a nationwide campaign and/or are either not nominated by a political party
or not supported by a registered political party with national constituency.” Pamatong argued
that this was against his right to “equal access to opportunities for public service,” citing
Article 2, Section 26 of the Constitution, and that the COMELEC was indirectly amending the
Constitution in this manner. Pamatong also stated that he is the “most qualified among all the
presidential candidates” and supported the statement with his legal qualifications, his alleged
capacity to wage national and international campaigns, and his government platform.

ISSUE: WON COMELEC’s refusal of Pamatong’s request for presidential candidacy, along with
the grounds for such refusal, violate the right to equal access to opportunities for public
service.

RULING: NO. The Court noted that the provisions under Article II are generally considered
not-self executing. As such, the provision in section 26, along with the other policies in the
article, does not convey any judicially enforceable rights. Article 2 “merely specifies a
guideline for legislative or executive action” by presenting ideals/standards through the
policies presented. Article 2, Section 26 recognizes a privilege to run for public office, one that
is subject to limitations provided by law. As long as these limitations are enforced without
discrimination, then the equal access clause is not violated. The Court justified the COMELEC’s
need for limitations on electoral candidates given the interest of ensuring rational, objective,
and orderly elections. In the absence of any limitations, the election process becomes a
“mockery” if anyone, including those who are clearly unqualified to hold a government
position, is allowed to run. Note: Pamatong presented other evidence that he claims makes
him eligible for candidacy. The Court however stated that it is not within their power to make
such assessments.
*Bernas’ Annotations
Case: Petitioner questions his being declared a nuisance candidate
Rule: Implicit in the petitioner’s invocation of the constitutional provision ensuring "equal access to
opportunities for public office" is the claim that there is a constitutional right to run for or hold public office
and, particularly in his case, to seek the presidency. There is none. What is recognized is merely a
privilege subject to limitations imposed by law. Section 26, Article II of the Constitution neither bestows
such a right nor elevates the privilege to the level of an enforceable right. There is nothing in the plain
language of the provision which suggests such a thrust or justifies an interpretation of the sort.
The "equal access" provision is a subsumed part of Article II of the Constitution, entitled "Declaration of
Principles and State Policies." The provisions under the Article are generally considered not self-
executing, and there is no plausible reason for according a different treatment to the "equal access"
provision. Like the rest of the policies enumerated in Article II, the provision does not contain any judicially
enforceable constitutional right but merely specifies a guideline for legislative or executive action. The
disregard of the provision does not give rise to any cause of action before the courts.
As earlier noted, the privilege of equal access to opportunities to public office may be subjected to
limitations. Some valid limitations specifically on the privilege to seek elective office are found in the
provisions of the Omnibus Election Code on "Nuisance Candidates" and COMELEC Resolution No. 6452
dated December 10, 2002 outlining the instances wherein the COMELEC may motu proprio refuse to give
due course to or cancel a Certificate of Candidacy. G.R. No. 161872 April 13, 2004 PAMATONG vs.
COMMISSION ON ELECTIONS

GUTIERREZ VS HOUSE OF REPRESENTATIVES


Digested by Eden Maizelle Salialam

TOPIC: Judicial Elaboration of the Constitution: Construction (in case of doubt, the Constitution
should be considered self-executing rather than non-self-executing)

FACTS:

● Private respondents Risa Hontiveros-Baraquel, Danilo Lim, and spouses Felipe and Evelyn
Pestaño (Baraquel group) filed an impeachment complaint1 against petitioner, upon the
endorsement of Party-List Representatives Arlene Bag-ao and Walden Bello.2
● On August 3, 2010, private respondents Renato Reyes, Jr., Mother Mary John Mananzan,
Danilo Ramos, Edre Olalia, Ferdinand Gaite and James Terry Ridon (Reyes group) filed
another impeachment complaint5 against petitioner.
● The Rules of Procedure in Impeachment Proceedings of the 15th Congress was
published on September 2, 2010.
● After hearing, public respondent found the two complaints, which both alleged culpable
violation of the Constitution and betrayal of public trust sufficient in substance, which
assumed hypothetically the truth of their allegations, hinged on the issue of whether
valid judgment to impeach could be rendered thereon.

ISSUE: W/N impeachment provisions in Section 3(8), Article XI of the Constitution self-executing, in relation to
petitioner’s contention that she was denied due process of the law.

RULING:

The impeachment provisions are not self-executing. Section 3(8) does not, in any circumstance,
operate to suspend the entire impeachment mechanism which the Constitutional Commission took
pains in designing even its details.

As against constitutions of the past, modern constitutions have been generally drafted upon a
different principle and have often become in effect extensive codes of laws intended to operate
directly upon the people in a manner similar to that of statutory enactments, and the function of
constitutional conventions has evolved into one more like that of a legislative body. Hence, unless
it is expressly provided that a legislative act is necessary to enforce a constitutional
mandate, the presumption now is that all provisions of the constitution are self-executing. If
the constitutional provisions are treated as requiring legislation instead of self-executing, the
legislature would have the power to ignore and practically nullify the mandate of the fundamental
law. This can be cataclysmic. That is why the prevailing view is, as it has always been, that —
. . . in case of doubt, the Constitution should be considered self-executing rather than
non-self-executing . . . . Unless the contrary is clearly intended, the provisions of the Constitution
should be considered self-executing, as a contrary rule would give the legislature discretion to
determine when, or whether, they shall be effective.
REPUBLIC VS. SERENO
ABRYL SYMON D. CULAS
TOPIC:

FACTS: This is a quo warranto proceeding which involves a judicial determination of


the right to use or exercise the office of the Chief Justice of the Supreme Court
occupied by Maria Lourdes Sereno.

There is an alleged failure to regularly disclose her assets, liabilities and net worth
prior to appointment as Associate Justice. Respondent insists that she could not be
removed from office except through impeachment.

ISSUE: W/NSection 2 Art XI of the Constitution limits to an action of impeachment


only.

RULING: No. Sec 2 Art XI does not preclude a quo warranto action questioning an
impeachable officer’s qualification to assume office.

The Constitution allows different provisions to stand together and to give effect to the
clear intent of the Constitution to address not only the impeachable offense but also
the issue of the qualifications of public officers.
TAÑADA VS. ANGARA
ABRYL SYMON D. CULAS
TOPIC:
FACTS: Secretary Navarro of DTI, representing the Philippines, signed the Final Act
of submitting the World Trade Organization agreement and adopting its Ministerial
Declarations and Decisions.

Petitioners assail the WTO Agreement for violating the mandate of the Constitution “to
develop a self-reliant and independent national economy effectively controlled by
Filipinos and give preference to qualified Filipinos promoting labor and domestic
goods.

ISSUE: W/N provisions of the Agreement establishing the WTO contravene provisions
of the Constitution.

RULING: No. The Constitution did not intend to pursue an isolationist policy and did
not shut out foreign investments, goods, and services. The Constitution does not
encourage the unlimited entry of foreign goods, services, and investments into the
country, it does not prohibit them either
CHIONGBIAN VS. DE LEON
Digested by: Nalcot, MPAA

FACTS: William Chiongbian (Petitioner) is a son of a Chinese citizen, Victoriano


Chiongbian, who was elected into office before the adoption of the Constitution, when
William Chiongbian was still a minor.

William’s Father became a Filipino citizen by virtue of Article IV, section 1, subsection
2 of the Constitution. William Chiongbian, the herein petitioner, who was then a minor,
also became a Filipino citizen by reason of subsection 3 (Article IV) of the
Constitution, his father having become a Filipino citizen upon the adoption of said
Constitution. This is also in conformity with the settled rule of our jurisprudence that a
legitimate minor child follows the citizenship of his father.

Respondents seek to cancel petitioner’s registration certificates of vessels and rescind


the sale of vessels from the same on the ground that the latter is allegedly not a
Filipino citizen and therefore not qualified to operate and own vessels of Philippine
registry.

ISSUE: WON the petitioner is a Filipino citizen.

HELD: Yes, because the petitioner, aside from the fact that he was a minor at the time
of the adoption of the Constitution, follows the citizenship of his father who, having
been elected to public office before the adoption of the said Constitution, became a
Filipino citizen as provided by the same (Art. IV, 1987 Constitution).

The Constitutional Convention could not have dedicated a provision of our


Constitution merely for the benefit of one person without considering that it could also
affect others. When they adopted subsection 2, they permitted, if not willed, that said
provision should function to the full extent of its substance and its terms, not by itself
alone, but in conjunction with all other provisions of that great document.
CIVIL LIBERTIES UNION VS. EXECUTIVE SECRETARY
Bontilao, Hanna Lyka

Art. 7, Sec 13 – "(T)he President, Vice-President, the Members of the Cabinet, and their
deputies or assistants shall not, unless otherwise provided in this Constitution, hold any other
office or employment during their tenure. They shall not, during said tenure, directly or
indirectly, practice any other profession, participate in any business, or be financially interested
in any contract with, or in any franchise, or special privilege granted by the Government or any
subdivision, agency or instrumentality thereof, including government-owned or controlled
corporations or their subsidiaries.”

Art. 9, Sec. 7 par. (2) – “Unless otherwise allowed by law or by the primary functions of his
position, no appointive official shall hold any other office or employment in the government or
any subdivision, agency or instrumentality thereof, including government-owned or controlled
corporations or their subsidiaries.”

TOPIC: RATIO LEGIS EST ANIMA & UT MAGIS VALEAT QUAM PEREAT
FACTS: Petitioners seek a declaration of the unconstitutionality of Executive Order No. 284
issued by President Corazon C. Aquino on July 25, 1987. The pertinent provisions of the
assailed Executive Order are:
Sec. 1. Even if allowed by law or by the ordinary functions of his position, a member of the
Cabinet, undersecretary or assistant secretary or other appointive officials of the Executive
Department may, in addition to his primary position, hold not more than two positions in the
government and government corporations and receive the corresponding compensation
therefor; Provided, that this limitation shall not apply to ad hoc bodies or committees, or to
boards, councils or bodies of which the President is the Chairman.

ISSUE: Whether or not Executive Order No. 284 is unconstitutional.


RULING: Yes. (Under Ratio legis et enima) The intent of the framers of the Constitution was to
impose a stricter prohibition on the President and his official family in so far as holding other
offices or employment in the government or elsewhere is concerned. Article 9-B, Sec. 7 is
meant to lay down the general rule applicable to all elective and appointive public officials and
employees, while Art. 7, Sec. 13 is meant to be the exception applicable only to the President,
the Vice-President, Members of the Cabinet, their deputies and assistants.

This being the case, the qualifying phrase "unless otherwise provided in this Constitution" in
Art. 7, Sec. 13 cannot possibly refer to the broad exceptions provided under Article 9-B, Sec. 7
of the 1987 Constitution because it would render nugatory and meaningless the manifest intent
and purpose of the framers of the Constitution to impose a stricter prohibition on the President,
Vice-President, Members of the Cabinet, their deputies and assistants with respect to holding
other offices or employment in the government during their tenure.

(Under Ut Magis Valeat Quam Pereat) Respondents' reading of the provisions in question
would render certain parts of the Constitution inoperative. For instance, if the phrase "unless
otherwise provided in this Constitution" found in Art. 7, Sec. 13 has reference to Article 9-B,
Sec. 7(1), it would render meaningless the specific provisions of the Constitution authorizing
the Vice-President to become a member of the Cabinet, and to act as President without
relinquishing the Vice-Presidency where the President shall not have been chosen or fails to
qualify.

BOLOS VS. BOLOS


Name of Digester: Eden Maizelle T. Salialam

TOPIC: PRINCIPLES OF CONSTITUTIONAL CONSTRUCTION: VERBA LEGIS

FACTS:
On January 16, 2007, the RTC issued the order declaring its August 2, 2006 decision
final and executory and granting the Motion for Entry of Judgment filed by Cynthia. Not
in conformity, Danilo filed with the CA a petition for certiorari under Rule 65 seeking to
annul the orders of the RTC as they were rendered with grave abuse of discretion
amounting to lack or in excess of jurisdiction, to wit: 1) the September 19, 2006 Order
which denied due course to Danilo’s appeal; 2) the November 23, 2006 Order which
denied the motion to reconsider the September 19, 2006 Order; and 3) the January
16, 2007 Order which declared the August 2, 2006 decision as final and executory.
Danilo also prayed that he be declared psychologically capacitated to render the
essential marital obligations to Cynthia, who should be declared guilty of abandoning
him, the family home and their children.

The CA granted the petition and reversed and set aside the assailed orders of the
RTC. The appellate court stated that the requirement of a motion for reconsideration
as a prerequisite to appeal under A.M. No. 02-11-10-SC did not apply in this case as
the marriage between Cynthia and Danilo was solemnized on February 14, 1980
before the Family Code took effect. It relied on the ruling of this Court in Enrico v.
Heirs of Sps. Medinaceli3 to the effect that the "coverage [of A.M. No. 02-11-10-SC]
extends only to those marriages entered into during the effectivity of the Family Code
which took effect on August 3, 1988."

ISSUE: Whether or not petitioner’s interpretation that the phrase "under the Family
Code" in A.M. No. 02-11-10-SC refers to the word "petitions" rather than to the word
"marriages."

RULING:
The categorical language of A.M. No. 02-11-10-SC leaves no room for doubt. The
coverage extends only to those marriages entered into during the effectivity of the
Family Code which took effect on August 3, 1988.7 The rule sets a demarcation line
between marriages covered by the Family Code and those solemnized under the Civil
Code.8The Court finds Itself unable to subscribe to petitioner’s interpretation that the
phrase "under the Family Code" in A.M. No. 02-11-10-SC refers to the word "petitions"
rather than to the word "marriages."
A cardinal rule in statutory construction is that when the law is clear and free from any
doubt or ambiguity, there is no room for construction or interpretation. There is only
room for application.9 As the statute is clear, plain, and free from ambiguity, it must be
given its literal meaning and applied without attempted interpretation. This is what is
known as the plain-meaning rule or verba legis. It is expressed in the maxim, index
animi sermo, or "speech is the index of intention." Furthermore, there is the maxim
verba legis non est recedendum, or "from the words of a statute there should be no
departure."10
ANG BAGONG BAYANI VS. COMELEC
NAME OF DIGESTER
TOPIC:
FACTS:
ISSUE:
RULING:
DATU MICHAEL ABAS KIDA VS. SENATE
NAME OF DIGESTER

TOPIC:

FACTS:

ISSUE:

RULING:
J.M. TUASON & CO., INC. VS. LAND TENURE
ADMINISTRATION
Louise Francis P. Ouano

TOPIC: Verba Legis non est recedendum

FACTS:
On August 3, 1959, Republic Act no. 2616 took effect. The act states that the Tatalon
Estate jointly owned by J.M. Tuason and Co. Inc, Gregorio Araneta and Co. Inc., and
Florencio Deudor et al was authorized to be expropriated. More than a year later Land
Tenure Administration was directed by the executive secretary to institute the
expropriation of the aforesaid property. Appellee thereupon filed a prohibition with a
preliminary injunction to prevent respondents from instituting the expropriation. The
lower court decided that the said act was unconstitutional and a writ of prohibition was
granted to the appellee. Respondent appealed to C.A.

ISSUE:
Whether the RA 2616 as amended by 3454 is constitutional? YES

RULING:
The decision of the lower court of January 10, 1963, holding that RA 2616 is
unconstitutional as amended by RA 3454 is reversed i.e. the expropriation is
constitutional
Rationale (as was stated in each:
Judicial Review is granted, if not expressly, at least by clear implication from
constitutional provisions. It may be exercised if an affected part files the appropriate
suit to test the validity of a legislative act, executive act, or municipal ordinance for that
matter.¬ The constitution is the supreme law and is binding on all governmental
agencies. Failure to comply provides a ground to nullify a governmental measure.

The question is one of constitutional construction (of interpreting the constitution). The
task is to ascertain the realization of the purpose of the framers and of the people in
adopting the Constitution. It is assumed that the words in the constitutional provisions
express the objectivity sought to be attained. They are to be given their ordinary
meaning except when technical terms are employed in which case the significance
attached to them prevails. This case is such a case and is therefore one of minimal
construction. The congress has the legislative will to expropriate and subdivide lands it
deems to be fit for sale. Moreover, it cannot be denied that congress has the capacity
to exercise such authority. The language employed is not swathed in obscurity
(because congress has the legislative power as stated in the constitution). It is
presumed that the constitution suffices to govern the life of the people not only at the
present time but also in the indefinite future. The constitution though does not give
rigid answers but is flexible and accommodates the problems the future may pose.
The constitution is dynamic in nature and not static. It reflects the social political
environment of the times. It adapts and changes. Although looking at the historical
reasons for why an act was passed would be helpful, it is not enough.
DOMINO VS. COMELEC
Digested by: Jadz Felix Alicer
TOPIC: CONSTRUCTION: Others- Verba Legis, Ratio Legis Est Anima, & Ut Magis
Valeat Quam Pereat
Full Text: https://lawphil.net/judjuris/juri1999/jul1999/gr_134015_1999.html
FACTS:
On 25 March 1998, DOMINO filed his certificate of candidacy for the position of
Representative of the Lone Legislative District of the Province of Sarangani.

On 30 March 1998, private respondents Narciso Ra. Grafilo, Jr., Eddy B. Java, Juan P.
Bayonito, Jr., Rosario Samson and Dionisio P. Lim, Sr., filed with the COMELEC a
Petition to Deny Due Course to or Cancel Certificate of Candidacy

Private respondents alleged that DOMINO, contrary to his declaration in the certificate
of candidacy, is not a resident, much less a registered voter, of the province of
Sarangani where he seeks election.

DOMINO maintains that he had complied with the one-year residence requirement
and that he has been residing in Sarangani since January 1997

Allegedly, counting, from the day after June 22, 1997 when respondent registered at
Precinct No. 4400-A, up to and until the day of the elections on May 11, 1998,
respondent lacks the one (1) year residency requirement provided for candidates for
Member of the House of Representatives under Section 6, Article VI of the
Constitution.

ISSUE: : Whether or not petitioner herein has resided in the subject congressional
district for at least one (1) year immediately preceding the May 11, 1998 elections.

RULING: Records show that petitioner's domicile of origin was Candon, Ilocos Sur
and that sometime in 1991, he acquired a new domicile of choice at 24 Bonifacio St.
Ayala Heights, Old Balara, Quezon City, as shown by his certificate of candidacy for
the position of representative of the 3rd District of Quezon City in the May 1995
election. Petitioner is now claiming that he had effectively abandoned his "residence"
in Quezon City and has established a new "domicile" of choice at the Province of
Sarangani.

It is doctrinally settled that the term "residence," as used in the law prescribing the
qualifications for suffrage and for elective office, means the same thing as "domicile,"
which imports not only an intention to reside in a fixed place but also personal
presence in that place, coupled with conduct indicative of such intention. "Domicile"
denotes a fixed permanent residence to which, whenever absent for business,
pleasure, or some other reasons, one intends to return. "Domicile" is a question of
intention and circumstances. In the consideration of circumstances, three rules must
be borne in mind, namely: (1) that a man must have a residence or domicile
somewhere; (2) when once established it remains until a new one is acquired; and (3)
a man can have but one residence or domicile at a time.

As a general rule, the principal elements of domicile, physical presence in the locality
involved and intention to adopt it as a domicile, must concur in order to establish a
new domicile. No change of domicile will result if either of these elements is absent.
Intention to acquire a domicile without actual residence in the locality does not result in
acquisition of domicile, nor does the fact of physical presence without intention.

The lease contract entered into sometime in January 1997, does not adequately
support a change of domicile. The lease contract may be indicative of DOMINO's
intention to reside in Sarangani but it does not engender the kind of permanency
required to prove abandonment of one's original domicile. The mere absence of
individual from his permanent residence, no matter how long, without the intention to
abandon it does not result in loss or change of domicile. Thus, the date of the contract
of lease of a house and lot located in the province of Sarangani, i.e., 15 January 1997,
cannot be used, in the absence of other circumstances, as the reckoning period of the
one-year residence requirement.

Further, Domino's lack of intention to abandon his residence in Quezon City is


further strengthened by his act of registering as voter in one of the precincts in
Quezon City. While voting is not conclusive of residence, it does give rise to a strong
presumption of residence especially in this case where DOMINO registered in his
former barangay.

Exercising the right of election franchise is a deliberate public assertion of the fact of
residence, and is said to have decided preponderance in a doubtful case upon the
place the elector claims as, or believes to be, his residence.
NAVARRO, BERNAL, AND MEDINA VS. EXECUTIVE
SECRETARY
NAME OF DIGESTER
TOPIC:
FACTS:
ISSUE:
RULING:
FRIVALDO VS. COMELEC
KRISTINA DANICA BALANAY

TOPIC: Judicial elaboration of the constitution and construction

FACTS: Petitioner Frivaldo filed his certificate for Candidacy for Governor about 3 weeks before the election.
Private Respondent Lee, another candidate for the said position, filed a petition to disqualify Frivaldo by reason
of not being a citizen of the Philippines. A week before the election, Second Division of COMELEC promulgated
a resolution granting the petition of Lee. A motion for Reconsideration was filed by Frivaldo which remained
unacted until after the elections. Thus, his candidacy continued and he was voted in during the elections.

Few days after the Election, the COMELEC En Banc affirmed the promulgated resolution of the Second
Division. Frivaldo garnered the highest number of votes in the said election. Lee filed a petition praying for his
proclamation as Governor and was then proclaimed as such. Frivaldo filed a petition for annulment of the
proclamation of Lee and for his own proclamation alleging that he had already taken his oath of allegiance as a
citizen of the Philippines which he filed a couple of months ago before the election.

Frivaldo’s motion was recognized and was then proclaimed as Governor. Lee filed a motion for reconsideration
which was denied by COMELEC En Banc.

ISSUE: Whether or not Frivaldo’s repatriation was valid and may be given retroactive effect.

RULING: Yes. Under Philippine law, citizenship may be reacquired by direct act of Congress, by naturalization
or by repatriation. Frivaldo said that in G.R. No. 104654 and during the oral argument in this case that he tried
to resume his citizenship by direct act of Congress, but that the bill allowing him to do so "failed to materialize,
notwithstanding the endorsement of several members of the House of Representatives" due, according to him,
to the "maneuvers of his political rivals." In the same case, his attempt at naturalization was rejected by this
Court because of jurisdictional, substantial and procedural defects.

Lee said that P.D. No. 725 had "been effectively repealed," asserting that "then President Corazon Aquino
exercising legislative powers under the Transitory Provisions of the 1987 Constitution, forbade the grant of
citizenship by Presidential Decree or Executive Issuances as the same poses a serious and contentious issue
of policy which the present government, in the exercise of prudence and sound discretion, should best leave to
the judgment of the first Congress under the 1987 Constitution," adding that in her memorandum dated March
27,1987 to the members of the Special Committee on Naturalization constituted for purposes of Presidential
Decree No. 725, President Aquino directed them "to cease and desist from undertaking any and all proceedings
within your functional area of responsibility as defined under Letter of Instructions (LOI) No. 270 dated April 11,
1975, as amended."

This memorandum dated March 27, 198724 cannot by any stretch of legal hermeneutics be construed as a law
sanctioning or authorizing a repeal of P.D. No. 725. Laws are repealed only by subsequent ones and a repeal
may be express or implied. It is obvious that no express repeal was made because then President Aquino in her
memorandum-- based on the copy furnished us by Lee-- did not categorically and/or impliedly state that P.D.
725 was being repealed or was being rendered without any legal effect. In fact, she did not even mention it
specifically by its number or text. On the other hand, it is a basic rule of statutory construction that repeals by
implication are not favored. An implied repeal will not be allowed "unless it is convincingly and unambiguously
demonstrated that the two laws are clearly repugnant and patently inconsistent that they cannot co-exist."

But to remove all doubts on this important issue, we also hold that the repatriation of Frivaldo RETRO ACTED
to the date of the filing of his application on August 17,1994. It is true that under the Civil Code of the
Philippines, "(l)aws shall have no retroactive effect, unless the contrary is provided." But there are settled
exceptions to this general rule, such as when the statute is CURATIVE or REMEDIAL in nature or when it
CREATES NEW RIGHTS.
ATTY. MACALINTAL VS. PET. CITED IN CHAVEZ VS. JBC
Digested by: Sophia Amor C. Bersamin

TOPIC: VERBA LEGIS, RATIO LEGIS EST ANIMA, & UT MAGIS VALEAT QUAM PEREAT

FACTS: Atty. Romulo Macalintal questions the constitutionality of the Presidential Electoral Tribunal
(PET) as an illegal and unauthorized progeny of Section 4, Article VII of the Constitution which prohibits
the designation of Members of the Supreme Court and of other courts established by law to any agency
performing quasi-judicial or administrative functions.

ISSUE: W/N the creation of the Presidential Electoral Tribunal is unconstitutional for being a violation of
paragraph 7, Section 4 of Article VII of the 1987 Constitution.

RULING: If the logic of petitioner is to be followed, all Members of the Court, sitting in the Senate and
House Electoral Tribunals would violate the constitutional proscription found in Section 12, Article VIII.
Surely, the petitioner will be among the first to acknowledge that this is not so. The Constitution which, in
Section 17, Article VI, explicitly provides that three Supreme Court Justices shall sit in the Senate and
House Electoral Tribunals, respectively, effectively exempts the Justices-Members thereof from the
prohibition in Section 12, Article VIII. In the same vein, it is the Constitution itself, in Section 4, Article VII,
which exempts the Members of the Court, constituting the PET, from the same prohibition.

We have previously declared that the PET is not simply an agency to which Members of the Court were
designated. Once again, the PET, as intended by the framers of the Constitution, is to be an institution
independent, but not separate, from the judicial department, i.e., the Supreme Court. McCulloch v. State
of Maryland49 proclaimed that "[a] power without the means to use it, is a nullity." The vehicle for the
exercise of this power, as intended by the Constitution and specifically mentioned by the Constitutional
Commissioners during the discussions on the grant of power to this Court, is the PET. Thus, a
microscopic view, like the petitioner’s, should not constrict an absolute and constitutional grant of judicial
power.

Verba legis dictates that wherever possible, the words used in the Constitution must be given their
ordinary meaning except where technical terms are employed, in which case the significance thus
attached to them prevails. As the Constitution is not primarily a lawyer’s document, it being essential for
the rule of law to obtain that it should ever be present in the people’s consciousness, its language as
much as possible should be understood in the sense they have in common use. However, where there is
ambiguity or doubt, the words of the Constitution should be interpreted in accordance with the intent of its
framers or ratio legis et anima. A doubtful provision must be examined in light of the history of the times,
and the condition and circumstances surrounding the framing of the Constitution. Last, ut magis valeat
quam pereat – the Constitution is to be interpreted as a whole. It is a well-established rule in constitutional
construction that no one provision of the Constitution is to be separated from all the others, to be
considered alone, but that all the provisions bearing upon a particular subject are to be brought into view
and to be so interpreted as to effectuate the great purposes of the instrument. Sections bearing on a
particular subject should be considered and interpreted together as to effectuate the whole purpose of the
Constitution and one section is not to be allowed to defeat another, if by any reasonable construction, the
two can be made to stand together. In other words, the court must harmonize them, if practicable, and
must lean in favor of a construction which will render every word operative, rather than one which may
make the words idle and nugatory.
TONDO MEDICAL CENTER EMPLOYEES ASSOCIATION
VS. CA
NAME OF DIGESTER
TOPIC:
FACTS:
ISSUE:
RULING:
BELGICA VS. EXECUTIVE SECRETARY
Digested by: Jannah Ajoc

FACTS:

Belgica sought for the Issuance of Temporary Restraining Order or Writ of Preliminary
Injunction seeking that the annual "Pork Barrel System," presently embodied in the
provisions of the GAA of 2013 which provided for the 2013 PDAF, and the Executive‘s
lump-sum, discretionary funds, such as the Malampaya Funds and the Presidential
Social Fund, be declared unconstitutional and null and void for being acts constituting
grave abuse of discretion. Also, they pray that the Court issues a TRO against
respondents.

ISSUE:

Whether or not the 2013 PDAF Article and all other Congressional Pork Barrel Laws
similar thereto are unconstitutional considering that they violate the principles of
constitutional provisions on (a) separation of powers; (b) non-delegability of legislative.

RULING:

Yes, the PDAF article is unconstitutional. The post-enactment measures which govern
the areas of project identification, fund release and fund realignment are not related to
functions of congressional oversight and, hence, allow legislators to intervene and/or
assume duties that properly belong to the sphere of budget execution. This violates
the principle of separation of powers. Congress ‘role must be confined to mere
oversight that must be confined to: (1) scrutiny and (2) investigation and monitoring of
the implementation of laws. Any action or step beyond that will undermine the
separation of powers guaranteed by the constitution. Thus, the court declares the
2013 PDAF article as well as all other provisions of law which similarly allow
legislators to wield any form of post-enactment authority in the implementation or
enforcement of the budget, unrelated to congressional oversight, as violative of the
separation of powers principle and thus unconstitutional. Wherefore, the petitions are
PARTLY GRANTED.
ARAULLO VS. AQUINO III
LIM, MA. THERESA
TOPIC: Effects of the declaration of unconstitutionality
FACTS:
-Sen. Jinggoy Ejercito Estrada delivered a privilege speech in the Senate of the Philippines to reveal that
some Senators, including himself, had been allotted an additional ₱50 Million each as "incentive" for
voting in favor of the impeachment of Chief Justice Renato C. Corona.
-Responding to Sen. Estrada’s revelation, Secretary Florencio Abad of the DBM issued a public
statement entitled Abad: Releases to Senators Part of Spending Acceleration Program, explaining that
the funds released to the Senators had been part of the DAP, a program designed by the DBM to ramp up
spending to accelerate economic expansion. He clarified that the funds had been released to the
Senators based on their letters of request for funding.
- Hence, the petition of Araullo assailing the constitutionality of the DAP and the issuances relating to the
DAP.
ISSUE: Whether or not the Disbursement Acceleration Program is unconstitutional and if so what are its
effects.

RULING: Yes. However, the Court ruled only some parts of DAP are unconstitutional for being in violation
of Section 25(5), Article VI of the 1987 Constitution and the doctrine of separation of powers.
A legislative or executive act that is declared void for being unconstitutional cannot give rise to any right
or obligation. However, rigidly applying the rule may at times be impracticable or wasteful. Should we not
recognize the need to except from the rigid application of the rule the instances in which the void law or
executive act produced an almost irreversible result?
The need is answered by the doctrine of operative fact. Iin De Agbayani v. Philippine National Bank:
An unconstitutional act, for that matter an executive order or a municipal ordinance likewise suffering from
that infirmity, cannot be the source of any legal rights or duties. Nor can it justify any official act taken
under it. Its repugnancy to the fundamental law once judicially declared results in its being to all intents
and purposes a mere scrap of paper. As the new Civil Code puts it: ‘When the courts declare a law to be
inconsistent with the Constitution, the former shall be void and the latter shall govern.’ Administrative or
executive acts, orders and regulations shall be valid only when they are not contrary to the laws of the
Constitution. The Constitution being supreme and paramount, any legislative or executive act contrary to
its terms cannot survive.
Such a view has support in logic and possesses the merit of simplicity. It may not however be sufficiently
realistic. It does not admit of doubt that prior to the declaration of nullity such challenged legislative or
executive act must have been in force and had to be complied with. This is so as until after the judiciary,
in an appropriate case, declares its invalidity, it is entitled to obedience and respect. Parties may have
acted under it and may have changed their positions. What could be more fitting than that in a
subsequent litigation regard be had to what has been done while such legislative or executive act was in
operation and presumed to be valid in all respects. It is now accepted as a doctrine that prior to its being
nullified, its existence as a fact must be reckoned with. This is merely to reflect awareness that precisely
because the judiciary is the governmental organ which has the final say on whether or not a legislative or
executive measure is valid, a period of time may have elapsed before it can exercise the power of judicial
review that may lead to a declaration of nullity. It would be to deprive the law of its quality of fairness and
justice then, if there be no recognition of what had transpired prior to such adjudication.
In the language of an American Supreme Court decision: ‘The actual existence of a statute, prior to such
a determination [of unconstitutionality], is an operative fact and may have consequences which cannot
justly be ignored. The past cannot always be erased by a new judicial declaration. The effect of the
subsequent ruling as to invalidity may have to be considered in various aspects, with respect to particular
relations, individual and corporate, and particular conduct, private and official.’"
The doctrine of operative fact recognizes the existence of the law or executive act prior to the
determination of its unconstitutionality as an operative fact that produced consequences that cannot
always be erased, ignored or disregarded. In short, it nullifies the void law or executive act but sustains its
effects. It provides an exception to the general rule that a void or unconstitutional law produces no effect.
But its use must be subjected to great scrutiny and circumspection, and it cannot be invoked to validate
an unconstitutional law or executive act, but is resorted to only as a matter of equity and fair play. It
applies only to cases where extraordinary circumstances exist, and only when the extraordinary
circumstances have met the stringent conditions that will permit its application.
We find the doctrine of operative fact applicable to the adoption and implementation of the DAP. Its
application to the DAP proceeds from equity and fair play. The consequences resulting from the DAP and
its related issuances could not be ignored or could no longer be undone.
ANGARA VS. ELECTORAL COMMISSION
RYAN JUANITO GARCIA

TOPIC: Theory of Judicial Review

FACTS:
​ In the elections of September 17, 1935, petitioner Jose A. Angara and the
respondents Pedro Ynsua, Miguel Castillo, and Dionisio Mayor were candidates for
the position of members of the National Assembly for the first district of Tayabas. The
provincial board of canvassers proclaimed Angara as member-elect of the National
Assembly and on November 15, 1935, he took his oath of office. On December 3,
1935, the National Assembly passed Resolution No. 8, which in effect, fixed the last
date to file election protests. On December 8, 1935, Ynsua filed before the Electoral
Commission a "Motion of Protest" against Angara and prayed, among other things,
that Ynsua be declared an elected Member of the National Assembly or that the
election of said position be nullified. On Dec. 9, 1935, the Electoral Commission
adopted Resolution No. 6 stating that the last day for filing of protests is on December
9 of the same year. Angara contended that the Constitution confers exclusive
jurisdiction upon the Electoral Commission solely as regards the merits of contested
elections to the National Assembly and the Supreme Court therefore has no
jurisdiction to hear the case

ISSUE:
Whether or not the Electoral Commission acted without or in excess of its
jurisdiction

RULING:
All the agencies of the government were designed by the Constitution to
achieve specific purposes, and each constitutional organ working within its own
particular sphere of discretionary action must be deemed to be animated with the
same zeal and honesty in accomplishing the great ends for which they were created
by the sovereign will. However, The separation of powers is a fundamental principle in
our system of government. It obtains not through express provision but by actual
division in our constitution. Each department of the government has exclusive
cognizance of matters within its jurisdiction, and is supreme within its own sphere. But
it does not follow from the fact that three powers are to be kept separate and distinct
that the constitution intended them to be absolutely unrestrained and independent to
each other. The constitution has provided for an elaborate system of checks and
balances to secure coordination in the workings of the various departments of the
government. The Electoral Commission was acting within the legitimate exercise of its
constitutional prerogative in assuming to take cognizance of the protest filed by the
respondent Pedro Ynsua against the election of the herein petitioner Jose A. Angara,
and that the resolution of the National Assembly of December 3, 1935 can not in any
manner toll the time for filing protests against the elections, returns and qualifications
of members of the National Assembly, nor prevent the filing of a protest within such
time as the rules of the Electoral Commission might prescribe.

The petition for a writ of prohibition against the Electoral Commission is hereby
denied.
G.R. No. 177857-58 | March 22, 2011
COCOFED VS. REPUBLIC
TOPIC: Theory of Judicial Review: Judicial Review and Separation of Powers

FACTS:

Philippine Coconut Producers Federation, Inc.(COCOFED) seeks the Court's approval


of the conversion of 753,848,312 Class "A" and Class "B" common shares of San
Miguel Corporation (SMC) registered in the names of 14 Holding Companies (CIIF
companies) into 753,848,312 SMC Series 1 Preferred Shares.

COCOFED proposes to constitute a trust fund to be known as the Coconut Industry


Trust Fund (CITF) for the benefit of the coconut farmers, with the Republic, acting
through the Philippine Coconut Authority (PCA), as trustee. As proposed, the
constitution of the CITF shall be subject to terms and conditions which, for the most
part, reiterate the features of SMC's conversion offer.

The Republic filed its Comment questioning COCOFED's personality to seek the
Court's approval of the desired conversion. Republic maintains that the CIIF SMC
common shares are sequestered assets and are in custodia legis under Presidential
Commission on Good Government's (PCGG's) administration. Hence, only PCGG has
the authority to approve the proposed conversion and seek the necessary Court
approval. Republic cites Republic v. Sandiganbayan where the coconut levy funds
were declared as prima facie public funds, thus reinforcing its position that only
PCGG, a government agency, can ask for approval of the conversion.

ISSUE:

W/N the Courts may exercise its power of judicial review in the case at bar.

RULING:

No. Jurisprudence is well-established that the courts cannot intervene or interfere with
executive or legislative discretion exercised within constitutional limits.

Corollary to the principle of separation of powers is the doctrine of primary jurisdiction


that the courts will DEFER to the decisions of the administrative offices and agencies
by reason of their expertise and experience in the matters assigned to them.
Administrative decisions on matters within the jurisdiction of administrative bodies are
to be respected and can only be set aside on proof of grave abuse of discretion, fraud,
or error of law.

The only instance when the Courts ought to interfere is when a department or an
agency has acted with grave abuse of discretion or violated a law.
TAWANG MULTI-PURPOSE COOPERATIVE VS. LA
TRINIDAD WATER DISTRICT
RYAN JUANITO GARCIA
FACTS:
Tawang Multi-Purpose Cooperative (TMPC) is a cooperative, registered with
the Cooperative Development Authority, and organized to provide domestic water
services in Barangay Tawang, La Trinidad, Benguet. On the other hand, La Trinidad
Water District (LTWD) is a local water utility created under Presidential Decree (PD)
No. 198, as amended. It is authorized to supply water for domestic, industrial and
commercial purposes within the municipality of La Trinidad, Benguet. TMPC filed with
the National Water Resources Board (NWRB) an application for a certificate of public
convenience (CPC) to operate and maintain a waterworks system in Barangay Tawang.
LTWD opposed TMPC’s application. LTWD claimed that, under Section 47 of PD No.
198, as amended, its franchise is exclusive which no franchise shall be granted to any
other person or agency for domestic, industrial or commercial water service within
the district or any portion thereof unless and except to the extent that the board of
directors of said district consents. NWRB held that LTWD’s franchise cannot be
exclusive since exclusive franchises are unconstitutional and found that TMPC is legally
and financially qualified to operate and maintain a waterworks system. LTWD filed a
motion for reconsideration but the NWRB denied the motion. LTWD appealed to the
RTC which the RTC cancelled TMPC’s CPC and held that Section 47 is valid because it is
for the state, through its authorized agencies or instrumentalities, to be able to keep
and maintain ultimate control and supervision over the operation of public utilities.

ISSUE:
Whether or not Section 47 of PD No. 198, as amended, is valid which LTWD be
exclusive in character.

RULING:
The court held that plain words do not require explanation. The 1935, 1973
and 1987 Constitutions are clear that franchises for the operation of a public utility
cannot be exclusive in character. These Constitutions expressly and clearly state that,
"nor shall such franchise x xx be exclusive in character." There is no exception. When
the law is clear, there is nothing for the courts to do but to apply it. The duty of the
Court is to apply the law the way it is worded. Section 47 gives the board of directors
the authority to make an exception to the absolute prohibition in the Constitution. In
short, the board of directors are given the discretion to create franchises that are
exclusive in character. If a law or contract violates any norm of the constitution that
law or contract whether promulgated by the legislative or by the executive branch or
entered into by private persons for private purposes is null and void and without any
force and effect. Therefore, this cannot be relied upon by LTWD in support of its
opposition against TMPC’s application for CPC and the subsequent grant thereof by
the NWRB.
CALLO-CLARIDAD VS. ESTEBAN
NAME OF DIGESTER
TOPIC:
FACTS:
ISSUE:
RULING:
METROBANK VS. TOBIAS III
NAME OF DIGESTER
TOPIC:
FACTS:
ISSUE:
RULING:
PEOPLE VS. SITON
NAME OF DIGESTER
TOPIC:
FACTS:
ISSUE:
RULING:
PEREZ VS. PEOPLE
NAME OF DIGESTER
TOPIC:
FACTS:
ISSUE:
RULING:
SMART COMMUNICATIONS, INC., Petitioner vs.MUNICIPALITY OF MALVAR,
BATANGAS, Respondent. G.R. No. 204429 February 18, 2014

NATURE OF THE CASE: PETITION for review on certiorari of the decision


and resolution of the Court of Tax Appeals En Banc.

SC RULING: Petition was DENIED

FACTS:

· Smart constructed a telecommunications tower within the territorial


jurisdiction of the Municipality of Malvar.
· The construction of the tower was for the purpose of receiving and
transmitting cellular communications within the covered area.
· On 30 July 2003, the Municipality passed Ordinance No. 18, series
of 2003, entitled "An Ordinance Regulating the Establishment of Special
Projects."
· On 24 August 2004, Smart received from the Permit and
Licensing Division of the Office of the Mayor of the Municipality an
assessment letter with a schedule of payment for the total amount of
₱389,950.00 for Smart’s telecommunications tower. The letter reads
as follows:

This is to formally submit to your good office your schedule of payments in


the Municipal Treasury of the Local Government Unit of Malvar, province of
Batangas which corresponds to the tower of your company built in the
premises of the municipality, to wit

:TOTAL PROJECT COST: PHP 11,000,000.00


For the Year 2001-2003
50% of 1% of the total project cost Php55,000.00
Add: 45% surcharge 24,750.00Php
79,750.00
Multiply by 3 yrs. (2001, 2002, 2003). Php239,250.00
For the year 2004
1% of the total project cost Php110,000.00
37% surcharge 40,700.00
Php150,700.00
TOTAL Php389,950.00
· Due to the alleged arrears in the payment of the assessment, the
Municipality also caused the posting of a closure notice on the
telecommunications tower.
· On 9 September 2004, Smart filed a protest, claiminglack of due
process in the issuance of the assessment and closure notice. In the
same protest, Smart challenged the validity of Ordinance No. 18 on
which the assessmentwas based.
· Municipality of Malvar denied Smart’s protest.
· Smart filed with RTC an "Appeal/Petition" assailing the validity of
Ordinance No. 18.
· RTC granted Smart’s Appeal/Petition.
· The trial court held that the assessment covering the period from
2001 to July 2003 was void since Ordinance No.18 was
approved only on 30 July 2003. However, the trial court declared
valid the assessment starting 1 October2003, citing Article 4 of the
Civil Code of the Philippines, in relation to the provisions of
Ordinance No. 18 and Section 166 of Republic Act No. 7160 or
the Local Government Code of 1991 (LGC).
· On 8 July 2009, Smart filed a petition for review with the CTA First
Division, denied the petition for review.
· CTA En Banc, affirmed the CTA First Division’s decision and
resolution.

ISSUES:
· Whether or not the fees imposed under Ordinance No. 18 are
taxes.
· Whether or not Ordinance No. 18 is constitutional

RULING:

The Court finds that the fees imposed under Ordinance No. 18 are not taxes.
Section 5, Article X of the 1987 Constitution provides that "each local
government unit shall have the power to create its own sources of revenues
and to levy taxes, fees, and charges subject to such guidelines and limitations
as the Congress may provide, consistent with the basic policy of local
autonomy. Such taxes, fees, and charges shall accrue exclusively to the local
government. "Consistent with this constitutional mandate, the LGC grants
the taxing powers to each local government unit. Specifically, Section
142 of the LGC grants municipalities the power to levy taxes, fees,
and charges not otherwise levied by provinces. Section 143 of the LGC
provides for the scale of taxes on business that may be imposed by
municipalities Section 147of the same law provides for the fees and
charges that may be imposed by municipalities on business and
occupation .The LGC defines the term "charges" as referring to pecuniary
liability, as rents or fees against persons or property, while the term
"fee" means "a charge fixed by law or ordinance for the regulation or
inspection of a business or activity. "The primary purpose of Ordinance No.
18 is to regulate the "placing, stringing, attaching, installing, repair and
construction of all gas mains, electric, telegraph and telephone wires,
conduits, meters and other apparatus" listed therein, which included
Smart’s telecommunications tower. Clearly, the purpose of the assailed
Ordinance is to regulate the enumerated activities particularly related to the
construction and maintenance of various structures. The fees in Ordinance
No. 18 are not impositions on the building or structure itself; rather, they
are impositions on the activity subject of government regulation, such as
the installation and construction of the structures. Since the main purpose of
Ordinance No. 18 is to regulate certain construction activities of the identified
special projects, which included "cell sites" or telecommunications towers, the
fees imposed in Ordinance No. 18 are primarily regulatory in nature, and not
primarily revenue-raising. While the fees may contribute to the revenues of the
Municipality, this effect is merely incidental. Thus, the fees imposed in
Ordinance No. 18 are not taxes. In Progressive Development Corporation v.
Quezon City, the Court declared that "if the generating of revenue is the
primary purpose and regulation is merely incidental, the imposition is a tax;
but if regulation is the primary purpose, the fact that incidentally revenue
is also obtained does not make the imposition a tax.
LOPEZ VS. ROXAS AND PET
NAME OF DIGESTER

Lopez vs Roxas and PET

By Eden Maagad

FACTS

Fernando Lopez and Gerardo Roxas were the main contenders for the office of the VP in the 1965 elections. By Resolution No. 2,
the 2 houses of Congress proclaimed Lopez elected as VP with a 26,724 votes difference. Roxas then filed with the Presidential
Electoral Tribunal, Election Protest No. 2, contesting the election of Lopez as VP, upon the ground that it was not he but Roxas
who had obtained the largest number of votes for VP. Lopez then instituted in the Supreme Court an action for prohibition with
preliminary injunction against Roxas to prevent PET from hearing and deciding the election contest, upon the ground that RA
1793, creating PET is unconstitutional, and that all proceedings taken by it are a nullity. Lopez' contention is predicated upon the
ground:

· That Congress may not, by law, authorize an election contest for Pres and VP, the Constitution being silent thereon

· that such contest tends to nullify the constitutional authority of Congress to procaim the candidates elected for Pres and VP

· that the recount of votes by the PET is inconsistent with the exclusive power of Congress to canvass the election returns for
the Pres and VP

· that no amendment to the Consti providing for an election protest involving Pres and VP office has been adopted

· that the tenure of the Pres and VP is fixed by the Consti and cannot be abridged by an Act of Congress like RA 1793

· that said act has an effect of amending the Consti that it permits PET to review the congressional proclamation of Pres and
VP

· that it is illegal for Justices of the SC to sit as members of the PET since the decisions thereof are appealable to the SC on
questions of law

· that the PET is a court inferior to the SC, and - that Congress by legislation appoint in effect the members of the PET.
ISSUE

WON Lopez is correct

RULING

Pursuant to the Constitution, the Judicial Power shall be vested in one Supreme Court and in such inferior courts as may be
established by law." This provision vests in the judicial branch of the government, not merely some specified or limited judicial
power, but "the" judicial power under our political system, and accordingly the entirety or all of said power, EXCEPT only so
much as the Constitution confers upon some other agency, such as the power to judge all contests relating to the election, returns
and qualifications of members of the Senate and those of the HOR which is vested by the fundamental law solely in the Senate
Electoral Tribunal and the House Electoral Tribunal. JUDICIAL POWER is the authority to settle justiciable controversies or
disputes involving rights that are enforceable and demandable before the courts of justice OR the redress of wrongs for violations
of such rights. The proper exercise of said authority requires legislative action: 1. Defining such enforceable and demandable rights
and/or prescribing remedies for violations thereof; and 2. Determining the court with jurisdiction to hear and decide said
controversies or disputes, in the first instance and/or on appeal. For this reason, the Constitution ordains that the Congress shall
have the power to define, prescribe, and apportion the jurisdiction of the various courts, subject to Constitutional limitations. Prior
to RA 1793, a defeated candidate for Pres or VP had NO LEGAL RIGHT to demand by election protest a recount of the votes cast.
As a consequence, controversies or disputes on this matter were not justiciable. RA 1793 has the effect of giving the candidate the
legal right to contest judicially the election of the Pres-elect or VP-elect and to demand a recount of the votes. And by providing,
further, that the PET shall be composed of the Chief Justice and other 10 members of the SC, said legislation has conferred upon
such Court an additional original jurisdiction of an exclusive character. RA 1793 has not created a new or separate court. It has
merely conferred upon the SC the functions of a PET. The Court is only one, although the functions may be distinct and separate.

PET is not inferior to the SEC since it is the same court although the functions peculiar to PET are more limited in scope than those
of the SC. Hence, the enactment of RA 1793, does not entail an assumption by Congress of the power of appointment vested by the
Consti in the President. It merely connotes the imposition of additional duties upon the members of the SC. Moreover, the power to
be the judge of contests relation to election, returns and qualification is essentially judicial. It belongs exclusively to the judicial
dept except insofar as the Constitution provides otherwise. It follows therefore that not only that RA 1793 is not inconsistent with
the Constitution or with principle of separation of powers but it is in harmony with the grant of judicial power to said courts.

BALANGAUAN VS. CA
Digested By: Alyana Mahilum
TOPIC: Grave Abuse of Discretion

FACTS:

Petition for Certiorari assailing the Decision and Resolution of the Court of Appeals w/c annulled and set
aside the Resolution of DOJ in Hong Kong and Shanghai Banking Corporation (HSBC) vs Balanguan
which dismissed the criminal complaint for Estafa filed against the Balanguan. In this Petition,
Balanguans urge the Supreme Court to reverse and set aside the decision of the Court of Appeals and
accordingly, dismiss the complaint against them in view of the absence of probable cause. HSBC filed an
estafa case against Balanguans which was dismissed by the Prosecutor in its Resolution, finding no
probable cause. HSBC appealed to the Secretary of DOJ by means of a Petition for Review which was
also dismissed. Motion for Reconsideration was also denied. HSBC then went to the Court of Appeals by
means of Petition for Certiorari thereby annulling and setting aside the resolutions of the DOJ.
Balanguans filed a Motion for Reconsideration before the Court of Appeals but was denied.
ISSUE:

Whether or not the Court of Appeals’ decision and resolution are tainted with grave abuse of discretion in
finding probable cause.

RULING:

The Prosecutor exceeded his authority and gravely abused his discretion. It must be remembered that a
finding of probable cause does not require an inquiry into whether there is sufficient evidence to procure a
conviction. It is enough that it is believed that the act or omission complained of constitutes the offense
charged.

The Court of Appeals found fault in the DOJ's failure to identify and discuss the issues raised by HSBC in
its Petition for Review. And, in support thereof, HSBC maintains that it is incorrect to argue that "it was not
necessary for the Secretary of Justice to have his resolution recite the facts and the law on which it was
based," because courts and quasi-judicial bodies should faithfully comply with Section 14, Article VIII of
the Constitution requiring that decisions rendered by them should state clearly and distinctly the facts of
the case and the law on which the decision is based.

It must be remembered that a preliminary investigation is not a quasi-judicial proceeding, and that the
DOJ is not a quasi-judicial agency exercising a quasi-judicial function when it reviews the findings of a
public prosecutor regarding the presence of probable cause

Though some cases describe the public prosecutor's power to conduct a preliminary investigation as
quasi-judicial in nature, this is true only to the extent that, like quasi-judicial bodies18, the prosecutor is an
officer of the executive department exercising powers akin to those of a court, and the similarity ends at
this point.

The alleged circumstances of the case at bar make up the elements of abuse of confidence, deceit or
fraudulent means, and damage under Art. 315 of the Revised Penal Code on estafa and/or qualified
estafa. They give rise to the presumption of reasonable belief that the offense of estafa has been
committed; and, thus, the filing of an Information against petitioners Bernyl and Katherene is warranted.
SINGIAN, JR. VS. SANDIGANBAYAN
ALERIC MONDANO
TOPIC: Grave Abuse of Discretion

FACTS:

Atty. Orlando L. Salvador was PCGG Consultant on detail with the Presidential Ad
Hoc Committee on Behest Loans (Committee). He was also the coordinator of the
Technical Working Group composed of officers and employees of government
financing institutions to examine and study the reports and recommendations of the
Asset Privatization Trust relating to loan accounts in all government financing
institutions. Among the accounts acted upon by the Committee were the loans granted
to Integrated Shoe, Inc. (ISI) by the Philippine National Bank (PNB).

In 1972, the PNB approved the loan, subject to certain stipulations. The said letter of
credit was to be secured by the following collaterals: a) a second mortgage on a lot
with improvements, machinery and equipment; b) machinery and equipment to be
imported under the subject letter of credit; and c) assignment of US$0.50 per pair of
shoes of ISI’s export sales. It was further subjected to the several conditions. ISI was
then further extended subsequent loan accommodations. The Committee found that
the loans extended to ISI bore characteristics of behest loans specifically for not
having been secured with sufficient collaterals and obtained with undue haste.

As a result, Atty. Orlando Salvador filed with the Office of the Ombudsman a sworn
complaint dated 20 March 1996, for violation of Section 3, paragraphs (e) and (g), of
Republic Act No. 3019, as amended. Hence, the corresponding eighteen (18)
Informations against petitioner and his co-accused for violation of Section 3(e) and (g)
of Rep. Act No. 3019 were filed before the Sandiganbayan. The eighteen (18)
Informations correspond to the nine (9) loan accommodations granted to ISI, each
loan being the subject of two informations alleging violations of both paragraphs of
Section 3 of Rep. Act No. 3019.

Thus, herein petitioner was charged with nine counts of violation of Section 3(e),7 and
another nine counts of violation of Section 3(g),8 of Republic Act No.3019 (RA 3019),
or the Anti-Graft and Corrupt Practices Act. Docketed as Criminal Case Nos.
26297-26314, the cases involved the purported granting of behest loans by the
government’s Philippine National Bank (PNB) to Integrated Shoes, Inc. (ISI), in
various amounts and on different dates as above-enumerated.
Some of the accused passed away, hence the dismissal of the case against them.
Nonetheless, trial ensued with respect to the remaining cases.
Prosecution presented their testimonial evidence and documentary evidence. After the
presentation of its testimonial and documentary evidence, the prosecution rested its
case and filed its Formal Offer of Exhibits. The respondent court admitted in toto the
State’s documentary exhibits.

Petitioner, with prior leave, filed a Demurrer to Evidence anchored on the following
grounds: (1) lack of proof of conspiracy with any PNB official; (2) the contracts with
PNB contained provisions that are beneficial, and not manifestly and grossly
disadvantageous, to the government; (3)the loans could not be characterized as
behest loans because they were secured by sufficient collaterals and ISI increased its
capitalization; and (4) assuming the loans are behest loans, petitioner could not be
held liable for lack of any participation. However, the Sandiganbayan denied the
demurrer.

ISSUE: Whether Sandiganbayan acted grave abuse of discretion in denying Singian’s


Demurrer of Evidence

RULING:

NO. There is no grave abuse of discretion on the part of the Sandiganbayan in


denying petitioner’s Demurrer to Evidence.

At the outset, we emphasize that "the resolution of a demurrer to evidence should be


left to the exercise of sound judicial discretion. A lower court’s order of denial shall not
be disturbed, that is, the appellate courts will not review the prosecution’s evidence
and precipitately decide whether such evidence has established the guilt of the
accused beyond a reasonable doubt, unless accused has established that such
judicial discretion has been gravely abused,
there by amounting to a lack or excess of jurisdiction. Mere allegations of such abuse
will not suffice."

"Grave abuse of discretion is the capricious and whimsical exercise of judgment on


the part of the public officer concerned which is equivalent to an excess or lack of
jurisdiction. The abuse of discretion must be so patent and gross as to amount to an
evasion of a positive duty or a virtual refusal to perform a duty enjoined by law, or to
act at all in contemplation of law as where the power is exercised in an arbitrary and
despotic manner by reason of
passion or hostility."

In this case, petitioner miserably failed to present an iota of evidence to show that the
Sandiganbayan abused, much more, gravely abused, its discretion in denying
petitioner’s Demurrer to Evidence.
PEOPLE VS. VERA
NAME OF DIGESTER
TOPIC:
FACTS:
ISSUE:
RULING:
ANGARA VS. ELECTORAL COMMISION
NAME OF DIGESTER
TOPIC:
FACTS:
ISSUE:
RULING:
IMBONG VS. OCHOA, JR.
LIM, MA. THERESA
TOPIC: Actual case or controversy
FACTS: Republic Act (R.A.) No. 10354, otherwise known as the Responsible
Parenthood and Reproductive Health Act of 2012 (RH Law), was enacted by
Congress on December 21, 2012. Following the enactment, petitioners and
challengers of the law questioned its constitutionality. One among the number of
issues they raised with the said law is that it threatens conscientious objectors of
criminal prosecution, imprisonment and other forms of punishment, as it compels
medical practitioners 1] to refer patients who seek advice on reproductive health
programs to other doctors; and 2] to provide full and correct information on
reproductive health programs and service, although it is against their religious beliefs
and convictions.

The respondents, aside from traversing the substantive arguments of the petitioners,
pray for the dismissal of the petitions for the principal reasons that 1] there is no actual
case or controversy and, therefore, the issues are not yet ripe for judicial
determination.; 2] some petitioners lack standing to question the RH Law; and 3] the
petitions are essentially petitions for declaratory relief over which the Court has no
original jurisdiction.

ISSUE: Whether or not the constitutionality of the RH Law may be challenged when
said law has yet been enforced.

RULING: YES. The petition holds merit.


In this case, the Court is of the view that an actual case or controversy exists and that
the same is ripe for judicial determination. Considering that the RH Law and its
implementing rules have already taken effect and that budgetary measures to carry
out the law have already been passed, it is evident that the subject petitions present a
justiciable controversy. As stated earlier, when an action of the legislative branch is
seriously alleged to have infringed the Constitution, it not only becomes a right, but
also a duty of the Judiciary to settle the dispute.

Moreover, the petitioners have shown that the case is so because medical
practitioners or medical providers are in danger of being criminally prosecuted under
the RH Law for vague violations thereof, particularly public health officers who are
threatened to be dismissed from the service with forfeiture of retirement and other
benefits. They must, at least, be heard on the matter NOW.

Lest it be misunderstood, it bears emphasizing that the Court does not have the
unbridled authority to rule on just any and every claim of constitutional violation.
Jurisprudence is replete with the rule that the power of judicial review is limited by four
exacting requisites, viz : (a) there must be an actual case or controversy; (b) the
petitioners must possess locus standi; (c) the question of constitutionality must be
raised at the earliest opportunity; and (d) the issue of constitutionality must be the lis
mota of the case.

In this case the court declared the RH LAW constitutional except for some parts of it
which are found unconstitutional including:
Section 23(a)(3) and the corresponding provision in the RH-IRR, particularly Section
5.24 thereof, insofar as they punish any healthcare service provider who fails and/or
refuses to refer a patient not in an emergency or life-threatening case, as defined
under Republic Act No. 8344, to another health care service provider within the same
facility or one which is conveniently accessible regardless of his or her religious
beliefs;

BAKER VS. CARR


NAME OF DIGESTER
TOPIC:
FACTS:
ISSUE:
RULING:
PEOPLE AND HSBC VS. VERA
NAME OF DIGESTER
TOPIC:
FACTS:
ISSUE:
RULING:
ABAKADA VS. PURISIMA
NAME OF DIGESTER
TOPIC:
FACTS:
ISSUE:
RULING:
TELEBAP VS. COMELEC
NAME OF DIGESTER
TOPIC:
FACTS:
ISSUE:
RULING:
PHILIPPINE ASSOCIATION OF COLLEGES AND
UNIVERSITIES VS. SECRETARY OF EDUCATION
Kyra Valentin
TOPIC: Conditions for the Exercise of Judicial Review: The Elements of Standing

FACTS: Philippine Association of Colleges and Universities assailed the constitutionality of Act No.
2706, known as the “Act making the Inspection and Recognition of private schools and colleges
obligatory for the Secretary of Public Instruction.”
As contended by PACU, the Act is unconstitutional because of the following reasons: 1) The act
deprives the owner of the school and colleges as well as teachers and parents of liberty and property
without due process of law; 2) it will also deprive the parents of their natural rights and duty to rear
their children for civic efficiency; and 3) its provisions conferred on the Secretary of Education unlimited
powers and discretion to prescribe rules and standards constitute towards unlawful delegation of
legislative powers.

Additionally, the association contended that the Constitution guaranteed every citizen the right to own
and operate a school and any law requiring previous governmental approval or permit before such
person could exercise the said right.

On the contrary, the Department of Education maintained that 1) the matters does not contain
justiciable controversy and thus does not need court decision or intervention; 2) petitioners are
inestoppels to challenge the validity of the said act; and 3) the Act is constitutionally valid.

Section 1 of Act No. 2706 provides that “It shall be the duty of the Secretary of Public Instruction to
maintain a general standard of efficiency in all private schools and colleges of the Philippines so that the
same shall furnish adequate instruction to the public, in accordance with the class and grade of
instruction given in them, and for this purpose said Secretary or his duly-authorized representative shall
have authority to advise, inspect, and regulate said schools and colleges in order to determine the
efficiency of instruction given in the same.”

ISSUE: Whether or not Act No. 2706 is unconstitutional.

RULING:

No, Act No. 2706 is constitutional.

The organic law provides that the state has the power to regulate private schools for the development
of morals, civic efficiency, and scientific aptitude of students. The court found no justiciable controversy.
The power of the courts to declare a law unconstitutional arises only when the interest of litigant
require the use of judicial authority for their protection against actual interference. As such, judicial
power is limited to the decision of actual cases and controversies. Thus, the court does not sit to
adjudicate a mere academic question, such as that provided by the petitioner. On this phase of the
litigation, the court conclude that there has been no undue delegation of legislative power even if the
petitioners appended a list of circulars and memoranda issued by the Department of Education.
JOYA VS. PCGG
Digested by: Nalcot, MPAA

FACTS: Petitioners are Filipino citizens, taxpayers and artists who claim to be deeply
concerned with the preservation and protection of the country’s artistic wealth.

This petition concerns old Masters Paintings and antique silverware alleged to be part
of the ill-gotten wealth of the late Pres. Marcos and his cronies. These were seized
from Malacañang and the Metropolitan Museum of Manila, and were consigned for
sale at public auction by the Govt through respondent PCGG.

Believing the items to be historical relics of cultural significance, petitioners filed the
instant petition for prohibition and mandamus to enjoin the PCGG to proceed with an
auction sale with prayer for preliminary injunction. The application for preliminary
injunction was denied and the auction proceeded as scheduled.

ISSUE: WON the instant petition complies with the legal requisites for this Court to
exercise its power of judicial review over this case.

HELD: No. The petitioners have no legal standing to file this petition. The confiscation
of the subject items by the Aquino administration should not be understood to mean
that its ownership has automatically passed on to the government without complying
with constitutional or statutory requirements.

Any dispute on the statutory defects in the acquisition and their subsequent disposition
must be raised only by the proper parties—the true owners thereof—whose authority
to recover emanates from their proprietary rights. Having failed to show that they are
the legal owners of the artworks or that these have become publicly owned, petitioners
do not possess any legal right to question their alleged unauthorized disposition.
CHAVEZ VS. JBC
NAME OF DIGESTER
TOPIC:
FACTS:
ISSUE:
RULING:
GALICTO VS. AQUINO
NAME OF DIGESTER
TOPIC:
FACTS:
ISSUE:
RULING:
DEL MAR VS. PHILIPPINE AMUSEMENT AND GAMING
CORPORATION
Digested by: Jadz Felix Alicer
TOPIC:
FACTS:
ISSUE:
RULING:
BIRAOGO VS. PHILIPPINE TRUTH COMMISSION
NAME OF DIGESTER
TOPIC:
FACTS:
ISSUE:
RULING:
DEMETRIA VS. ALBA
NAME OF DIGESTER: Jadz Felix Alicer
TOPIC: Standing of Taxpayers
Full Text: https://lawphil.net/judjuris/juri1987/feb1987/gr_71977_1987.html
FACTS:
Assailed in this petition for prohibition with prayer for a writ of preliminary injunction is
the constitutionality of the first paragraph of Section 44 of Presidential Decree No.
1177, otherwise known as the "Budget Reform Decree of 1977."

Petitioners, who filed the instant petition as concerned citizens of this country, as
members of the National Assembly/Batasan Pambansa representing their millions of
constituents, as parties with general interest common to all the people of the
Philippines, and as taxpayers whose vital interests may be affected by the outcome of
the reliefs prayed for"

The Solicitor General, for the public respondents, questioned the legal standing of
petitioners, who were allegedly merely begging an advisory opinion from the Court,
there being no justiciable controversy fit for resolution or determination.

ISSUE: Whether or not the petitioners have legal standing to sue as taxpayers

RULING:

YES. It is well-settled that the validity of a statute may be contested only by one who
will sustain a direct injury in consequence of its enforcement. Yet, there are many
decisions nullifying, at the instance of taxpayers, laws providing for the disbursement
of public funds, upon the theory that "the expenditure of public funds by an officer of
the State for the purpose of administering an unconstitutional act constitutes a
misapplication of such funds," which may be enjoined at the request of a taxpayer.
LAMP VS. SECRETARY OF BUDGET AND
MANAGEMENT
NAME OF DIGESTER
TOPIC:
FACTS:
ISSUE:
RULING:
DELA LLANA VS. THE CHAIRPERSON, COA,
EXECUTIVE SECRETARY, AND NATIONAL TREASURER
NAME OF DIGESTER
TOPIC:
FACTS:
ISSUE:
RULING:
BELGICA VS. EXECUTIVE SECRETARY
GR NO. 208566

NAME OF DIGESTER: Kyle Ferdausi G. Cerna

TOPIC: On the Standing of Taxpayers for Judicial Review.

FACTS:
Spurred in large part by the findings contained in the CoA Report and the Napoles
controversy, several petitions were lodged before the Court similarly seeking that the
"Pork Barrel System" be declared unconstitutional.

Petitioner Greco Antonious Beda B. Belgica, and many others filed an Urgent Petition
For Certiorari and Prohibition With Prayer For The Immediate Issuance of Temporary
Restraining Order (TRO) and/or Writ of Preliminary Injunction dated August 27, 2013
under Rule 65 of the Rules of Court seeking that the annual "Pork Barrel System,"
presently embodied in the provisions of the GAA of 2013 which provided for the 2013
PDAF, and the Executive‘s lump-sum, discretionary funds, such as the Malampaya
Funds and the Presidential Social Fund, be declared unconstitutional and null and
void for being acts constituting grave abuse of discretion.

Also, they pray that the Court issue a TRO against respondents Paquito N. Ochoa, Jr.,
Florencio B. Abad (Secretary Abad) and Rosalia V. De Leon, in their respective
capacities as the incumbent Executive Secretary, Secretary of the Department of
Budget and Management (DBM), and National Treasurer, or their agents, for them to
immediately cease any expenditure under the aforesaid funds.

ISSUE:

Whether or not the petitioner has a legal standing to seek for judicial review.

RULING: Yes. Petitioners have come before the Court in their respective capacities as
citizen-taxpayers and accordingly, assert that they "dutifully contribute to the coffers of
the National Treasury."

Clearly, as taxpayers, they possess the requisite standing to question the validity of
the existing "Pork Barrel System" under which the taxes they pay have been and
continue to be utilized. It is undeniable that petitioners, as taxpayers, are bound to
suffer from the unconstitutional usage of public funds, if the Court so rules. Invariably,
taxpayers have been allowed to sue where there is a claim that public funds are
illegally disbursed or that public money is being deflected to any improper purpose, or
that public funds are wasted through the enforcement of an invalid or unconstitutional
law, as in these cases.
Moreover, as citizens, petitioners have equally fulfilled the standing requirement given
that the issues they have raised may be classified as matters "of transcendental
importance, of overreaching significance to society, or of paramount public interest."
The CoA Chairperson ‘s statement during the Oral Arguments that the present
controversy involves "not merely a systems failure" but a "complete breakdown of
controls" amplifies, in addition to the matters above-discussed, the seriousness of the
issues involved herein. Indeed, of greater import than the damage caused by the
illegal expenditure of public funds is the mortal wound inflicted upon the fundamental
law by the enforcement of an invalid statute. All told, petitioners have sufficient locus
standi to file the instant cases.
THE CITY OF DAVAO VS. RTC
NAME OF DIGESTER
TOPIC:
FACTS:
ISSUE:
RULING:
CHAVEZ VS. PCGG (sample case Title)
NAME OF DIGESTER: Kyle Ferdausi G. Cerna
TOPIC: Legal Standing

FACTS: Petitioner, invoking his constitutional right to information and the correlative
duty of the state to disclose publicly all its transactions involving the national interest,
demands that respondents make public any and all negotiations and agreements
pertaining to PCGG's task of recovering the Marcoses' ill-gotten wealth. He claims that
any compromise on the alleged billions of ill-gotten wealth involves an issue of
"paramount public interest," since it has a "debilitating effect on the country's
economy" that would be greatly prejudicial to the national interest of the Filipino
people. Hence, the people in general have a right to know the transactions or deals
being contrived and effected by the government.

Respondents counter argue that they do not deny forging a compromise agreement
with the Marcos heirs. They claim, though, that petitioner's action is premature,
because there is no showing that he has asked the PCGG to disclose the negotiations
and the Agreements. And even if he has, PCGG may not yet be compelled to make
any disclosure, since the proposed terms and conditions of the Agreements have not
become effective and binding.

ISSUE: Whether or not the petitioner has a standing to file for a petition the disclosure
of documents from PCGG.

RULING: Yes. The instant petition is anchored on the right of the people to information
and access to official records, documents and papers — a right guaranteed under
Section 7, Article III of the 1987 Constitution. Petitioner, a former solicitor general, is a
Filipino citizen. Because of the satisfaction of the two basic requisites laid down by
decisional law to sustain petitioner's legal standing, i.e. (1) the enforcement of a public
right (2) espoused by a Filipino citizen, we rule that the petition at bar should be
allowed.

The high court agreed with the petitioner, when he emphasized that, the ordinary
taxpayers have a right to initiate and prosecute actions questioning the validity of acts
or orders of government agencies or instrumentalities, if the issues raised are "of
paramount public interest;" and if they "immeasurably affect the social, economic, and
moral well-being of the people."
IDEALS, INC. VS. PSALM
NAME OF DIGESTER
TOPIC:
FACTS:
ISSUE:
RULING:
KILOSBAYAN VS. GUINGONA
G.R. No. 113375
Aimee E. Demontano

TOPIC: Standing to sue as taxpayers


Transcendental Importance” and “Paramount Public Interest”

FACTS:
This is a special civil action for prohibition and injunction, with a prayer for a temporary
restraining order and preliminary injunction, which seeks to prohibit and restrain the
implementation of the "Contract of Lease" executed by the Philippine Charity
Sweepstakes Office (PCSO) and the Philippine Gaming Management Corporation
(PGMC) in connection with the on-line lottery system “lotto”.

Petitioner Kilosbayan, Incorporated (KILOSBAYAN) avers that it is a non-stock


domestic corporation composed of civic-spirited citizens, pastors, priests, nuns, and
lay leaders who are committed to the cause of truth, justice, and national renewal. The
rest of the petitioners, except Senators Freddie Webb and Wigberto Tañada and
Representative Joker P. Arroyo, are suing in their capacities as members of the Board
of Trustees of KILOSBAYAN and as taxpayers and concerned citizens. Senators
Webb and Tañada and Representative Arroyo are suing in their capacities as
members of Congress and as taxpayers and concerned citizens of the Philippines.

PCSO formally issued a Request for Proposal (RFP) for the Lease Contract of an
on-line lottery system, the terms of which require that the lessor shall be a domestic
corporation, with at least 60% of its shares owned by Filipino shareholders.

One of the bidders was Philippine Gaming Management Corporation (PGMC), a


domestic corporation incorporated by Berjaya Group Berhad, a Malaysian company,
through its individual nominees, together with some Filipino investors.

An agreement denominated as Contract of Lease was executed between PCSO and


PGMC.

Petitioner Kilosbayan filed an injunction petition with the court to restrain the
implementation of the Contract of Lease, on the ground, among others, that under the
PCSO Charter, PCSO is prohibited from holding and conducting lotteries 'in
collaboration, association or joint venture with any person, association, company or
entity.”

The government responded that it is actually PCSO which will operate the lottery while
the winning corporate bidders are merely "lessors."
ISSUE:
WoN KILOSBAYAN was allowed to initiate and prosecute actions

RULING: YES
In line with the liberal policy of this Court on locus standi, ordinary taxpayers,
members of Congress, and even association of planters, and non-profit civic
organizations were allowed to initiate and prosecute actions before this Court to
question the constitutionality or validity of laws, acts, decisions, rulings, or orders of
various government agencies or instrumentalities.

We find the instant petition to be of transcendental importance to the public. The


issues it raised are of paramount public interest and of a category even higher than
those involved in many of the aforecited cases. The ramifications of such issues
immeasurably affect the social, economic, and moral well-being of the people even in
the remotest barangays of the country and the counter- productive and retrogressive
effects of the envisioned on-line lottery system are as staggering as the billions in
pesos it is expected to raise. The legal standing then of the petitioners deserves
recognition and, in the exercise of its sound discretion, this Court hereby brushes
aside the procedural barrier which the respondents tried to take advantage of.
IDEALS, INC. VS. PSALM
NAME OF DIGESTER
TOPIC:
FACTS:
ISSUE:
RULING:
IMBONG VS. OCHOA, JR.
LIM, MA. THERESA
TOPIC: Instances on standing
FACTS: Republic Act (R.A.) No. 10354, otherwise known as the Responsible
Parenthood and Reproductive Health Act of 2012 (RH Law), was enacted by
Congress on December 21, 2012. Following the enactment, petitioners and
challengers of the law questioned its constitutionality. One among the number of
issues they raised with the said law is that it threatens conscientious objectors of
criminal prosecution, imprisonment and other forms of punishment, as it compels
medical practitioners 1] to refer patients who seek advice on reproductive health
programs to other doctors; and 2] to provide full and correct information on
reproductive health programs and service, although it is against their religious beliefs
and convictions.

The OSG attacks the legal personality of the petitioners to file their respective
petitions. The petitioners, for their part, invariably invoke the "transcendental
importance" doctrine and their status as citizens and taxpayers in establishing the
requisite locus standi.

ISSUE: Whether or not the petitioners lack legal standing to question the RH Law..

RULING: NO. In this case, the Court has accorded certain individuals standing to sue,
not otherwise directly injured or with material interest affected by a Government act,
provided a constitutional issue of transcendental importance is invoked.

Locus standi or legal standing is defined as a personal and substantial interest in a


case such that the party has sustained or will sustain direct injury as a result of the
challenged governmental act. The Court leans on the doctrine that "the rule on
standing is a matter of procedure, hence, can be relaxed for non-traditional plaintiffs
like ordinary citizens, taxpayers, and legislators when the public interest so requires,
such as when the matter is of transcendental importance, of overreaching significance
to society, or of paramount public interest.”

The transcendental importance of the issues involved in this case warrants that we set
aside the technical defects and take primary jurisdiction over the petition at bar. One
cannot deny that the issues raised herein have potentially pervasive influence on the
social and moral well being of this nation, specially the youth; hence, their proper and
just determination is an imperative need. This is in accordance with the
well-entrenched principle that rules of procedure are not inflexible tools designed to
hinder or delay, but to facilitate and promote the administration of justice. Their strict
and rigid application, which would result in technicalities that tend to frustrate, rather
than promote substantial justice, must always be eschewed.

After all, the RH Law drastically affects the constitutional provisions on the right to life
and health, the freedom of religion and expression and other constitutional rights.
Mindful of all these and the fact that the issues of contraception and reproductive
health have already caused deep division among a broad spectrum of society, the
Court entertains no doubt that the petitions raise issues of transcendental importance
warranting immediate court adjudication. More importantly, considering that it is the
right to life of the mother and the unborn which is primarily at issue, the Court need not
wait for a life to be taken away before taking action. The Court cannot, and should not,
exercise judicial restraint at this time when rights enshrined in the Constitution are
being imperiled to be violated. To do so, when the life of either the mother or her child
is at stake, would lead to irreparable consequences.

OPOSA VS. FACTORAN


RYAN JUANITO GARCIA

FACTS: The principal petitioners are all minors duly represented and joined by their
respective parents. Impleaded as an additional plaintiff is the Philippine Ecological
Network, Inc. (PENI), a domestic, non-stock and non-profit corporation organized for
the purpose of, inter alia, engaging in concerted action geared for the protection of
our environment and natural resources. The original defendant was the Honorable
Fulgencio S. Factoran, Jr., then Secretary of the Department of Environment and
Natural Resources (DENR). His substitution in this petition by the new Secretary, the
Honorable Angel C. Alcala, was subsequently ordered upon proper motion by the
petitioners. The complaint was instituted as a taxpayers’ class suit and alleges that the
plaintiffs are all citizens of the Republic of the Philippines, taxpayers, and entitled to
the full benefit, use and enjoyment of the natural resource treasure that is the
country’s virgin tropical forests. The same was filed for themselves and others who are
equally concerned about the preservation of said resource but are so numerous that it
is impracticable to bring them all before the Court. The petitioners are minors who
claimed their right to a balanced and healthful ecology. It speaks of the responsibilities
of each generation as trustees and guardian of the environment for future
generations. The minors further assert that they represent their generation as well as
generations yet unborn. Consequently, it is prayed for that judgment be rendered
ordering defendant, his agents, representatives and other persons acting on his behalf
to cancel all existing timber license agreements in the country, cease and desist from
receiving, accepting, processing, renewing or approving new timber license
agreements and granting the plaintiffs such other reliefs just and equitable under the
premises.

ISSUE: Whether or not the petitioners have locus standi.

RULING: The petitioners have legal standing. Their personality to sue in behalf of the
succeeding generations can only be based on the concept of intergenerational
responsibility insofar as the right to a balanced and healthful ecology is concerned.
RESIDENT MARINE MAMMALS OF THE PROTECTED
SEASCAPE OF TAÑON STRAIT VS. REYES
NAME OF DIGESTER
TOPIC:
FACTS:
ISSUE:
RULING:
BELGICA VS. EXECUTIVE SECRETARY
NAME OF DIGESTER
TOPIC:
FACTS:
ISSUE:
RULING:
TADEO VS. PROVINCIAL FISCAL OF PANGASINAN
NAME OF DIGESTER
TOPIC:
FACTS:
ISSUE:
RULING:
SOUTHERN HEMISPHERE ENGAGEMENT NETWORK,
INC. VS. ANTI TERRORISM COUNCIL
ABRYL SYMON D. CULAS

TOPIC: Actual Case and Controversy; Advisory Opinion vs. Declaratory Relief (Page
6 of syllabus)

FACTS: This involves six petitions by different groups and individuals challenging the
constitutionality of Republic Act No. 9372 (RA 9372), "An Act to Secure the State and
Protect our People from Terrorism," otherwise known as the Human Security Act of
2007

Petitioner-organizations assert locus standi on the basis of being suspected


"communist fronts" by the government, especially the military; whereas individual
petitioners invariably invoke the "transcendental importance" doctrine and their status
as citizens and taxpayers.

No ground was properly established by petitioners for the taking of judicial notice. no
specific charge or proscription under RA 9372 has been filed against them, three
years after its effectivity, belies any claim of imminence of their perceived threat
emanating from the so-called tagging.

ISSUE: W/N case involves an actual case and controversy to satisfy the exercise of
judicial review?

RULING: No. Petitioners have not presented any personal stake in the outcome of the
controversy.

None of them faces any charge under RA 9372 It bears to stress that generalized
interests, albeit accompanied by the assertion of a public right, do not establish locus
standi. Evidence of a direct and personal interest is key.

Petitioners fail to present an actual case or controversy

By constitutional fiat, judicial power operates only when there is an actual case or
controversy.
APEX MINING CO., VS. SOUTHEAST MINDANAO CORP
ABRYL SYMON D. CULAS

TOPIC: A Constitutional Question raised at the earliest possible time (Page 6 of


syllabus)

FACTS: This involves three cases of different groups concerning the disputed area of
Diwalwal gold rush area and the seeking of exploration permits. Proclamation No. 297
declares the disputed area as mineral reservation and an environmentally critical area
which affects the claims of the different groups.

In its last-ditch effort to salvage its case, Southeast Mindanao Gold Mining
Corporation (SEM) contends that Proclamation No. 297, issued by President Gloria
Macapagal-Arroyo and declaring the Diwalwal Gold Rush Area as a mineral
reservation, is invalid on the ground that it lacks the concurrence of Congress.

It is well-settled that when questions of constitutionality are raised, the court can
exercise its power of judicial review only if the following requisites are present: (1) an
actual and appropriate case exists; (2) there is a personal and substantial interest of
the party raising the constitutional question; (3) the exercise of judicial review is
pleaded at the earliest opportunity; and (4) the constitutional question is the lis mota of
the case.

ISSUE: W/N The third Requisite of Judicial Review is Present. (that it be pleaded at
the earliest opportunity)

RULING: No. Taking into consideration the foregoing requisites of judicial review, it is
readily clear that the third requisite is absent. The general rule is that the question of
constitutionality must be raised at the earliest opportunity, so that if it is not raised in
the pleadings, ordinarily it may not be raised at the trial; and if not raised in the trial
court, it will not be considered on appeal.

In the instant case, the Reply to Respondent SEM’s Consolidated Comment filed on
20 May 2003, Mines Adjudication Board (MAB) mentioned Proclamation No. 297,
which was issued on 25 November 2002. This proclamation, according to the MAB,
has rendered SEM’s claim over the contested area moot, as the President has already
declared the same as a mineral reservation and as an environmentally critical area.
SEM did not put to issue the validity of said proclamation in any of its pleadings
despite numerous opportunities to question the same. It was only after the assailed
Decision was promulgated.

​Certainly, posing the question on the constitutionality of Proclamation No. 297 for the
first time in its Motion for Reconsideration is, indeed, too late.
LOZANO VS. NOGRALES
ABRYL SYMON D. CULAS

TOPIC: A Constitutional Question raised at the earliest possible time (Page 6 of


syllabus)

FACTS: The two petitions, filed by petitioners in their capacities as concerned citizens
and taxpayers, prayed for the nullification of House Resolution No. 1109 entitled "A
Resolution Calling upon the Members of Congress to Convene for the Purpose of
Considering Proposals to Amend or Revise the Constitution, Upon a Three-fourths
Vote of All the Members of Congress."

ISSUE: W/N This was raised at the earliest possible time

RULING: No. In the present case, the fitness of petitioners’ case for the exercise of
judicial review is grossly lacking.

House Resolution No. 1109 only resolved that the House of Representatives shall
convene at a future time for the purpose of proposing amendments or revisions to the
Constitution. No actual convention has yet transpired and no rules of procedure have
yet been adopted. More importantly, no proposal has yet been made, and hence, no
usurpation of power or gross abuse of discretion has yet taken place. In short, House
Resolution No. 1109 involves a quintessential example of an uncertain contingent
future event that may not occur as anticipated, or indeed may not occur at all. The
House has not yet performed a positive act that would warrant an intervention from
this Court.
IMBONG VS. OCHOA, JR.
ABRYL SYMON D. CULAS

TOPIC: A Constitutional Question raised at the earliest possible time (Page 6 of


syllabus)

FACTS: ​Responsible Parenthood and Reproductive Health Act of 2012 (RH Law),
was enacted by Congress on December 21, 2012.

The petitioners pray that the entire RH Law be declared unconstitutional.

The Office of the Solicitor General (OSG) submits that as an "as applied challenge," it
cannot prosper considering that the assailed law has yet to be enforced and applied to
the petitioners, and that the government has yet to distribute reproductive health
devices that are abortive.

ISSUE: W/N The Constitutional Question is raised at the earliest possible time

RULING: In this case, the Court is of the view that an actual case or controversy
exists and that the same is ripe for judicial determination. Considering that the RH Law
and its implementing rules have already taken effect and that budgetary measures to
carry out the law have already been passed, it is evident that the subject petitions
present a justiciable controversy. As stated earlier, when an action of the legislative
branch is seriously alleged to have infringed the Constitution, it not only becomes a
right, but also a duty of the Judiciary to settle the dispute.

Moreover, the petitioners have shown that the case is so because medical
practitioners or medical providers are in danger of being criminally prosecuted under
the RH Law for vague violations thereof, particularly public health officers who are
threatened to be dismissed from the service with forfeiture of retirement and other
benefits. They must, at least, be heard on the matter NOW.
CANDARI VS. DONASCO
ABRYL SYMON D. CULAS

TOPIC: A Constitutional Question raised at the earliest possible time (Page 6 of


syllabus)

FACTS: plaintiffs were incumbent BOD members of DARBCI and did not consent or
sanctioned (sic) the 26 November 2005 BOD election

Respondents instituted Civil Case to enjoin petitioners Candari, et al from holding a


special general assembly (GA) and an election of officers alleging that the process by
which the GA had been called was not in accordance with Sec. 35 of Republic Act No.
6938, otherwise known as the Cooperative Code of the Philippines.

Despite the TRO, but without the participation of petitioners, 5,910 members – or
78.68% of the total membership of the cooperative – went through with the GA on 26
November 2005

RTC, finding the provisional remedy of preliminary injunction to be moot, issued a


Resolution denying respondents’ prayer for the issuance of a writ of preliminary
injunction and quashing the TRO previously issued.

ISSUE: W/N The Question was raised at the earliest possible time

RULING: No. petitioners cannot, however, be restored to their positions. Their terms
expired in 1989, thereby rendering their prayer for reinstatement moot and academic.

If there were genuine grievances against petitioners, the affected members should
have timely raise (sic) these issues in the annual general assembly or in a special
general assembly. Or, if such a remedy would be futile for some reason or another,
judicial recourse was available.
BELGICA VS. EXECUTIVE SECRETARY
NAME OF DIGESTER

TOPIC:
FACTS:
ISSUE:
RULING:
GALICTO VS. AQUINO
Lig Andre S. Nuesca

TOPIC: A Constitutional Question Raised at the Earliest Possible Time


FACTS: On July 26, 2010, Pres. Aquino made public in his first State of the Nation Address the alleged
excessive allowances, bonuses and other benefits of Officers and Members of the Board of Directors of
the Manila Waterworks and Sewerage System – a government owned and controlled corporation (GOCC)
which has been unable to meet its standing obligations. Subsequently, the Senate of the Philippines
(Senate), through the Senate Committee on Government Corporations and Public Enterprises, conducted
an inquiry in aid of legislation on the reported excessive salaries, allowances, and other benefits of
GOCCs and government financial institutions (GFIs).

Heeding the call of Congress, Pres. Aquino, on September 8, 2010, issued EO 7, entitled "Directing the
Rationalization of the Compensation and Position Classification System in the [GOCCs] and [GFIs], and
for Other Purposes." EO 7 provided for the guiding principles and framework to establish a fixed
compensation and position classification system for GOCCs and GFIs. EO 7 ordered (1) a moratorium on
the increases in the salaries and other forms of compensation, except salary adjustments under EO 8011
and EO 900, of all GOCC and GFI employees for an indefinite period to be set by the President, and (2) a
suspension of all allowances, bonuses and incentives of members of the Board of Directors/Trustees until
December 31, 2010.

On June 6, 2011, Congress enacted Republic Act (R.A.) No. 10149, otherwise known as the "GOCC
Governance Act of 2011." Section 11 of RA 10149 expressly authorizes the President to fix the
compensation framework of GOCCs and GFIs.

Before us is a Petition for Certiorari and Prohibition with Application for Writ of Preliminary Injunction
and/or Temporary Restraining Order, seeking to nullify and enjoin the implementation of Executive Order
No. (EO) 7 issued by the Office of the President on September 8, 2010. Petitioner Jelbert B. Galicto
asserts that EO 7 is unconstitutional for having been issued beyond the powers of the President and for
being in breach of existing laws.

ISSUE: Whether or not that the petition is considered mooted

RULING: YES. The petition has been mooted by supervening events. Because of the transitory
nature of EO 7, it has been pointed out that the present case has already been rendered moot by these
supervening events: (1) the lapse on December 31, 2010 of Section 10 of EO 7 that suspended the
allowances and bonuses of the directors and trustees of GOCCs and GFIs; and (2) the enactment of R.A.
No. 10149 amending the provisions in the charters of GOCCs and GFIs empowering their board of
directors/trustees to determine their own compensation system, in favor of the grant of authority to the
President to perform this act.

A moot case is "one that ceases to present a justiciable controversy by virtue of


supervening events, so that a declaration thereon would be of no practical use or value.”[A]n
action is considered ‘moot’ when it no longer presents a justiciable controversy because the
issues involved have become academic or dead[,] or when the matter in dispute has already
been resolved and hence, one is not entitled to judicial intervention unless the issue is likely to
be raised again between the parties x x x. Simply stated, there is nothing for the x x x court to
resolve as [its] determination x x x has been overtaken by subsequent events.”
All told, in view of the supervening events rendering the petition moot, as well as its patent formal and
procedural infirmities, we no longer see any reason for the Court to resolve the other issues raised in the
certiorari petition.

PORMENTO VS. ESTRADA AND COMELEC


NALCOT, MPAA

TOPIC: A Constitutional Question Raised at the Earliest Possible Time

FACTS: Petitioner Atty. Evillo C. Pimento files petition asking whether private
respondent Joseph Ejercito Estrada is covered by the ban on the President from “any
reelection.”

Private respondent was elected President of the Philippines in the general elections
held on May 11, 1998 and sought presidency again in the general elections held on
May 10, 2010. Petitioner Atty. Evillo C. Pimento opposed private respondent’s
candidacy and filed a petition for disqualification. On his second run for presidential
office, Private respondent did not win.

ISSUE: Whether or not the case presents an actual case or controversy.

RULING: No. Since the issue on the proper interpretation of the phrase "any
reelection" will be premised on a person’s second (whether immediate or not) election
as President, there is no case or controversy to be resolved in this case. No live
conflict of legal rights exists. There is in this case no definite, concrete, real or
substantial controversy that touches on the legal relations of parties having adverse
legal interests. No specific relief may conclusively be decreed upon by this Court in
this case that will benefit any of the parties herein. As such, one of the essential
requisites for the exercise of the power of judicial review, the existence of an actual
case or controversy, is sorely lacking in this case.

As a rule, this Court may only adjudicate actual, ongoing controversies. The Court is
not empowered to decide moot questions or abstract propositions, or to declare
principles or rules of law which cannot affect the result as to the thing in issue in the
case before it. In other words, when a case is moot, it becomes non-justiciable.

An action is considered "moot" when it no longer presents a justiciable controversy


because the issues involved have become academic or dead or when the matter in
dispute has already been resolved and hence, one is not entitled to judicial
intervention unless the issue is likely to be raised again between the parties. There is
nothing for the court to resolve as the determination thereof has been overtaken by
subsequent events. [Excerpt from Atty. Pascua’s Notes]
IDEALS VS. PSALM
NAME OF DIGESTER
TOPIC:
FACTS:
ISSUE:
RULING:
FUNA VS. THE CHAIRMAN, COA, VILLAR
NAME OF DIGESTER
TOPIC:
FACTS:
ISSUE:
RULING:
VILANDO VS. HRET
RYAN JUANITO GARCIA
TOPIC:

FACTS:
In the May 2017 elections Jocelyn Limkaichong ran as a representative in the 1st
District of Negros Oriental. She won over the other contender, Olivia Paras. Petitions involving
the disqualification of Limkaichong and questions about her citizenship were filed. They
alleged that Limkaichong was not a natural born citizen of the Philippines because when she
was born her father was still a Chinese and that her mother lost her Filipino citizenship by
virtue of her marriage to Limkaichong’s father.

ISSUES:
(1) Whether or not the House of Representatives Electoral Tribunal already acquired
jurisdiction over the case; and ,

(2) Whether or not Limkaichong is qualified to hold an office in the Republic of the
Philippines.

RULING:
1. No. The House of Representatives Electoral Tribunal has no authority to delve into
the legality of the judgment of naturalization. To rule otherwise would operate as a collateral
attack on the citizenship of Limkalchong’s father which is not permissible. In our jurisdiction,
an attack on a person’s citizenship may only be done through a direct action for its nullity.

2. Yes. Records disclose that Limkaichong was born in Dumaguete City on November
9, 1959. The governing law is the citizenship provision of the 1935 Constitution. The HRET,
therefore, correctly relied on the presumption of validity of the July 9, 1957 and September
21, 1959 Orders of the Court of First Instance (CFI) Negros Oriental, which granted the petition
and declared Julio Sy a naturalized Filipino absent any evidence to the contrary. Respondent
Limkaichong falls under the category of those persons whose fathers are citizens of the
Philippines. (Section 1(3), Article IV, 1935 Constitution) It matters not whether the father
acquired citizenship by birth or by naturalization. Therefore, following the line of transmission
through the father under the 1935 Constitution, the respondent has satisfactorily complied
with the requirement for candidacy and for holding office, as she is a natural-born Filipino
citizen.

Respondent participated in the barangay elections as a young voter in 1976,


accomplished voter's affidavit as of 1984, and ran as a candidate and was elected as Mayor of
La Libertad, Negros Oriental in 2004. These are positive acts of election of Philippine
citizenship.

The case of In re: Florencio Mallare, elucidates how election of citizenship is


manifested in actions indubitably showing a definite choice. It was noted that respondents
had informally elected citizenship after January 17, 1973 during which time the 1973
Constitution considered as citizens of the Philippines all those who elect citizenship in
accordance with the 1935 Constitution.
GENERAL VS. URRO
Kyra Valentin
TOPIC: Conditions for the Exercise of Judicial Review: (4) A Constitutional Question
that is the very lis mota of the case i.e. an unavoidable question

FACTS: When Roces, a former NAPOLCOM Commissioner, died in September 2007, PGMA appointed the
petitioner on July 21, 2008 as acting NAPOLCOM Commissioner in place of Roces. On the same date, PGMA
appointed Eduardo U. Escueta (Escueta) as acting NAPOLCOM Commissioner and designated him as NAPOLCOM
Vice Chairman.

Later, PGMA appointed Alejandro S. Urro(Urro) in place of the petitioner, Constancia P.de Guzman in place of Celia
Leones, and Escuetaas permanent NAPOLCOM Commissioners. In a letter dated March 19, 2010, DILG Head
Executive Assistant/Chief-of-Staff Pascual V. Veron Cruz, Jr. issued separate congratulatory letters to the
respondents, for being appointed as NAPOLCOM Commissioners. The petitioner then filed the present quo
warranto petition questioning the validity of the respondents appointments mainly on the ground that it violates
the constitutional prohibition against midnight appointments. On July 30, 2010, Pres. Benigno S. Aquino III, issued
Executive Order No. 2 (E.O. No. 2) "Recalling, Withdrawing, and Revoking Appointments Issued by the Previous
Administration in Violation of the Constitutional Ban on Midnight Appointments."

The petitioner argues that the appointment issued to him was really a "regular" appointment, and as such, he
cannot be removed from office except for cause. Since the appointment paper of respondent Urro, while bearing a
date prior to the effectivity of the constitutional ban on appointments,was officially released (perthe
congratulatory letter dated March 19, 2010 issued to Urro) when the appointment ban was already in effect, then
the petitioners appointment, though temporary in nature, should remain effective as no new and valid
appointment was effectively made. The petitioner assails the validity of the appointments of respondents De
Guzman and Escueta on the same grounds. Both parties dwelt lengthily on the issue of constitutionality of the
respondents appointments in light of E.O. No. 2.
ISSUE: Whether or not the Court can exercise its power of judicial review
RULING: ​No, The petition lacks merit.

POLITICAL LAW: Judicial power; kinds of appointments.


When questions of constitutional significance are raised, the Court can exercise its power of judicial review only if the following
requisites are present: (1) the existence of an actual and appropriate case; (2) the existence of personal and substantial interest
on the part of the party raising the constitutional question; (3)recourse to judicial review is made at the earliest opportunity; and
(4) the constitutional question is the lis mota of the case. Lis mota literally means "the cause of the suit or action. In the present
case, the constitutionality of the respondents appointments is not the lis mota of the case. From the submitted pleadings, what is
decisive is the determination of whether the petitioner has a cause of action to institute and maintain this present petition: a quo
warranto against respondent Urro.

The Court already held that for a petition for quo warranto to be successful, the suing private individual must show a clear right to
the contested office. Since the petitioner merely holds an acting appointment (and an expired one at that), he clearly does not
have a cause of action to maintain the present petition. The essence of an acting appointment is its temporariness and its
consequent revocability at any time by the appointing authority.

Generally, the power to appoint vested in the President includes the power to make temporary (acting) appointments, unless he
is otherwise specifically prohibited by the Constitution or by the law, or where an acting appointment is repugnant to the nature
of the office involved. Here, nothing in the enumeration of functions of the members of the NAPOLCOM that would be subverted
or defeated by the President's appointment of an acting NAPOLCOM Commissioner pending the selection and qualification of a
permanent appointee. Viewed as an institution, a survey of pertinent laws and executive issuances will show that the NAPOLCOM
has always remained as an office under or within the Executive Department. Clearly, there is nothing repugnant between the
petitioners acting appointment, on one hand, and the nature of the functions of the NAPOLCOM Commissioners or of the
NAPOLCOM as an institution, on the other.

Estoppel also clearly militates against the petitioner. From the time he was appointed until apprised of the appointment of Urro,
the petitioner discharged the functions of his office without expressing any misgivings on his appointment. He cannot later on be
heard to say that the appointment was really a permanent one so that he could not be removed except for cause. Denied.
SPS. MIRASOL VS. CA
NAME OF DIGESTER
TOPIC:
FACTS:
ISSUE:
RULING:
CREBA VS. ROMULO
NAME OF DIGESTER
TOPIC:
FACTS:
ISSUE:
RULING:
DUEÑAS VS. HRET
NAME OF DIGESTER
TOPIC:
FACTS:
ISSUE:
RULING:
JAVIER VS. COMELEC
NAME OF DIGESTER
TOPIC:
FACTS:
ISSUE:
RULING:
QUIZON VS. COMELEC
NAME OF DIGESTER
TOPIC:
FACTS:
ISSUE:
RULING:
ABS-CBN VS. COMELEC
Digested by: Jadz Felix Alicer
TOPIC: Functions of Judicial Review: Symbolic
Full Text: https://lawphil.net/judjuris/juri2000/jan2000/gr_133486_2000.html
FACTS:

The Resolution was issued by the Comelec allegedly upon "information from [a]
reliable source that ABS-CBN (Lopez Group) has prepared a project, with PR groups,
to conduct radio-TV coverage of the elections . . . and to make [an] exit survey of the .
. . vote during the elections for national officials particularly for President and Vice
President, results of which shall be [broadcast] immediately." The electoral body
believed that such project might conflict with the official Comelec count, as well as the
unofficial quick count of the National Movement for Free Elections (Namfrel). It also
noted that it had not authorized or deputized Petitioner ABS-CBN to undertake the exit
survey.

The solicitor general contends that the petition is moot and academic, because the
May 11, 1998 election has already been held and done with. Allegedly, there is no
longer any actual controversy before us.

ISSUE: Whether or not the case is moot

RULING:

The issue is not totally moot. While the assailed Resolution referred specifically to
the May 11, 1998 election, its implications on the people's fundamental freedom of
expression transcend the past election. The holding of periodic elections is a basic
feature of our democratic government. By its very nature, exit polling is tied up with
elections.

To set aside the resolution of the issue now will only postpone a task that could well
crop up again in future elections in any event, in Salonga v. Cruz Paño, the Court had
occasion to reiterate that it "also has the duty to formulate guiding and controlling
constitutional principles, precepts, doctrines, or rules. It has the symbolic function of
educating bench and bar on the extent of protection given by constitutional
guarantees."51Since the fundamental freedoms of speech and of the press are being
invoked here, we have resolved to settle, for the guidance of posterity, whether they
likewise protect the holding of exit polls and the dissemination of data derived
therefrom.
J.M. TUASON & CO. VS CA
NAME OF DIGESTER
TOPIC:
FACTS:
ISSUE:
RULING:
YNOT VS. IAC
Kristina Danica Balanay
TOPIC: Power of Judicial Review

FACTS:
On January 13, 1984, petitioner Restituto Ynot had transported six carabaos in a pump boat from Masbate to Iloilo
when they were confiscated by the police station commander of Barotac Nuevo, Iloilo, for violation of the above
measure. The petitioner sued for recovery, and the Regional Trial Court of Iloilo City issued a writ of replevin upon his
filing of a supersedeas bond of P12,000.00. After considering the merits of the case, the court sustained the
confiscation of the carabaos and, since they could no longer be produced, ordered the confiscation of the bond. The
court also declined to rule on the constitutionality of the executive order, as raised by the petitioner, for lack of authority
and also for its presumed validity. The petitioner appealed the decision to the Intermediate Appellate Court, which
upheld the trial court, and now Ynot comes before the SC through a petition for review on certiorari.

Ynot’s petition assails that the questioned EO is unconstitutional insofar as it authorizes outright confiscation of the
carabao or carabeef being transported across provincial boundaries. Petitioner claims that the penalty is invalid
because it is imposed without according the owner a right to be heard before a competent and impartial court as
guaranteed by due process. He complains that the measure should not have been presumed, and so sustained, as
constitutional.

This case challenges the constitutionality of Executive Order 626-A which amends EO 626 and states that:
SECTION 1. Executive Order No. 626 is hereby amended such that henceforth, no carabao regardless of age, sex,
physical condition or purpose and no carabeef shall be transported from one province to another. The carabao or
carabeef transported in violation of this Executive Order as amended shall be subject to confiscation and forfeiture by
the government, to be distributed to charitable institutions and other similar institutions as the Chairman of the National
Meat Inspection Commission may ay see fit, in the case of carabeef, and to deserving farmers through dispersal as the
Director of Animal Industry may see fit, in the case of carabaos.

ISSUE: Whether EO 626-A is not unconstitutional.

RULING: NO. In the instant case, the carabaos were arbitrarily confiscated by the police station commander, were
returned to the petitioner only after he had filed a complaint for recovery and given a supersedeas bond of P12,000.00,
which was ordered confiscated upon his failure to produce the carabaos when ordered by the trial court. The executive
order defined the prohibition, convicted the petitioner and immediately imposed punishment, which was carried out
forthright. The measure struck at once and pounced upon the petitioner without giving him a chance to be heard, thus
denying him the centuries-old guarantee of elementary fair play.

It has already been remarked that there are occasions when notice and hearing may be validly dispensed with
notwithstanding the usual requirement for these minimum guarantees of due process. It is also conceded that
summary action may be validly taken in administrative proceedings as procedural due process is not necessarily
judicial only. In the exceptional cases accepted, however there is a justification for the omission of the right to a
previous hearing, to wit, the immediacy of the problem sought to be corrected and the urgency of the need to correct it.

In the case at bar, there was no such pressure of time or action calling for the petitioner's peremptory treatment. The
properties involved were not even inimical per se as to require their instant destruction. There certainly was no reason
why the offense prohibited by the executive order should not have been proved first in a court of justice, with the
accused being accorded all the rights safeguarded to him under the Constitution. Considering that, as the Court held in
Pesigan v. Angeles, Executive Order No. 626-A is penal in nature, the violation thereof should have been pronounced
not by the police only but by a court of justice, which alone would have had the authority to impose the prescribed
penalty, and only after trial and conviction of the accused.

To sum up, the SC finds that the challenged measure is an invalid exercise of the police power because the method
employed to conserve the carabaos is not reasonably necessary to the purpose of the law and, worse, is unduly
oppressive. Due process is violated because the owner of the property confiscated is denied the right to be heard in his
defense and is immediately condemned and punished. The conferment on the administrative authorities of the power
to adjudge the guilt of the supposed offender is a clear encroachment on judicial functions and militates against the
doctrine of separation of powers. There is, finally, also an invalid delegation of legislative powers to the officers
mentioned therein who are granted unlimited discretion in the distribution of the properties arbitrarily taken. For these
reasons, Executive Order No. 626-A is unconstitutional.
ARAULLO, ET. AL. VS. AQUINO III, ET. AL.
Digested by: Sophia Amor C. Bersamin

TOPIC: Functions of Judicial Review - Symbolic

FACTS: On September 25, 2013, Sen. Jinggoy Ejercito Estrada delivered a privilege speech in the
Senate of the Philippines to reveal that some Senators, including himself, had been allotted an additional
₱50 Million each as "incentive" for voting in favor of the impeachment of Chief Justice Renato C. Corona.

Responding to Sen. Estrada’s revelation, Secretary Florencio Abad of the DBM issued a public statement
entitled Abad: Releases to Senators Part of Spending Acceleration Program, explaining that the funds
released to the Senators had been part of the DAP, a program designed by the DBM to ramp up spending
to accelerate economic expansion. He clarified that the funds had been released to the Senators based
on their letters of request for funding; and that it was not the first time that releases from the DAP had
been made because the DAP had already been instituted in 2011 to ramp up spending after sluggish
disbursements had caused the growth of the gross domestic product (GDP) to slow down. He explained
that the funds under the DAP were usually taken from (1) unreleased appropriations under Personnel
Services;2 (2) unprogrammed funds; (3) carry-over appropriations unreleased from the previous year;
and (4) budgets for slow-moving items or projects that had been realigned to support faster-disbursing
projects.

ISSUE: W/N the court should exercise the power of Judicial Review on the executive acts implemented
by DBM with regard to the disbursement of funds

RULING: The requisites for the exercise of the power of judicial review are the following, namely: (1)
there must be an actual case or justiciable controversy before the Court; (2) the question before the Court
must be ripe for adjudication; (3) the person challenging the act must be a proper party; and (4) the issue
of constitutionality must be raised at the earliest opportunity and must be the very litis mota of the case.

An actual and justiciable controversy exists in these consolidated cases. The incompatibility of the
perspectives of the parties on the constitutionality of the DAP and its relevant issuances satisfy the
requirement for a conflict between legal rights. The issues being raised herein meet the requisite
ripeness considering that the challenged executive acts were already being implemented by the DBM,
and there are averments by the petitioners that such implementation was repugnant to the letter and spirit
of the Constitution. Moreover, the implementation of the DAP entailed the allocation and expenditure of
huge sums of public funds. The fact that public funds have been allocated, disbursed or utilized by reason
or on account of such challenged executive acts gave rise, therefore, to an actual controversy that is
ripe for adjudication by the Court.

Quite often, as here, the petitioner in a public action sues as a citizen or taxpayer to gain locus standi.
Although such petitioner may not be as adversely affected by the action complained against as are
others, it is enough that he sufficiently demonstrates in his petition that he is entitled to protection or relief
from the Court in the vindication of a public right. Under their respective circumstances, each of the
petitioners has established sufficient interest in the outcome of the controversy as to confer locus standi
on each of them. In addition, considering that the issues center on the extent of the power of the Chief
Executive to disburse and allocate public funds, whether appropriated by Congress or not, these cases
pose issues that are of transcendental importance to the entire Nation, the petitioners included.
As such, the determination of such important issues call for the Court’s exercise of its broad and wise
discretion "to waive the requirement and so remove the impediment to its addressing and resolving the
serious constitutional questions raised.
BROKENSHIRE MEMORIAL HOSPITAL, INC. VS. HON.
MINISTER OF LABOR AND EMPLOYMENT
NAME OF DIGESTER
TOPIC:
FACTS:
ISSUE:
RULING:
ONGSUCO VS. MALONES
NAME OF DIGESTER: Kyle Ferdausi G. Cerna
TOPIC: Judicial Discretion and the Power to Review

FACTS:
Due to the imposition of Municipal Ordinance No. 98-01, Respondent wrote a letter to
petitioners informing them that they were occupying stalls in the newly renovated
municipal public market without any lease contract, as a consequence of which, the
stalls were considered vacant and open for qualified and interested applicants.

This increased rentals for the stalls and the imposition of goodwill fees for stalls. The
same Code authorized respondent to enter into lease contracts over the said market
stalls, and incorporated a standard contract of lease for the stall holders at the
municipal public market.

This prompted petitioners to file before the RTC a Petition for Prohibition/Mandamus,
with Prayer for Issuance of Temporary Restraining Order and/or Writ of Preliminary
Injunction, against respondent. Arguing mainly, that the Municipal Ordinance is not
enforceable because it did not go through a public hearing. The respondents also
claimed that such a petition for the court’s intervention cannot be done due to the lack
of exhaustion of administrative remedies. The RTC and the CA ruled for the
respondents.

ISSUE: Whether or not the petitioners erred in filing for a petition seeking for the
court’s intervention.

RULING: No.

The rule on the exhaustion of administrative remedies is intended to preclude a court


from taking unto itself the authority to resolve a controversy, the jurisdiction over which
is initially lodged with an administrative body of special competence. Thus, a case
where the issue raised is a purely legal question, well within the competence; and the
jurisdiction of the court and not the administrative agency, would clearly constitute an
exception. Resolving questions of law, which involve the interpretation and application
of laws, constitutes essentially an exercise of judicial power that is exclusively
allocated to the Supreme Court and such lower courts the Legislature may establish.

In this case, the parties are not disputing any factual matter on which they still need to
present evidence. The sole issue petitioners raised before the RTC in Civil Case No.
25843 was whether Municipal Ordinance No. 98-01 was valid and enforceable despite
the absence, prior to its enactment, of a public hearing held in accordance with Article
276 of the Implementing Rules and Regulations of the Local Government Code. This
is undoubtedly a pure question of law, within the competence and jurisdiction of the
RTC to resolve.
Under the Constitution, the Supreme Court recognizes the original jurisdiction of lower
courts over cases involving the constitutionality or validity of an ordinance. It provides,
that, the Supreme Court has the power to Review, revise, reverse, modify or affirm on
appeal or certiorari, as the law or the Rules of Court may provide, final judgments and
orders of lower courts in:

(a) All cases in which the constitutionality or validity of any treaty, international or
executive agreement, law, presidential decree, proclamation, order, instruction,
ordinance, or regulation is in question.
FERNANDEZ VS. CUERVA
NAME OF DIGESTER
TOPIC:
FACTS:
ISSUE:
RULING:
ALDOVINO VS. ALUNAN
NAME OF DIGESTER
TOPIC:
FACTS:
ISSUE:
RULING:
G. R. No. L-23127

Francisco Serrano de Agbayani vs. Philippine National Bank

Digested by: Rynn Judd C. Escaño

Judicial Elaboration of the Constitution

Effect of a Declaration of Unconstitutionality

Facts:

On July 19, 1939, Francisco Serrano de Agbayani secured a loan of ₱450 from
Philippine National Bank (PNB) secured with a real estate mortgage. The loan matured
on July 19, 1944. However, the bank was unable to collect because it was war time. In
1945, President Osmeña issued Executive Order no. 32: Debt Moratorium Law which
suspended the payments of loans for four (4) years because of the effects of the war. In
1948, RA 342 extended the EO no. 32 for another eight (8) years making it effective
until 1956. However, the Supreme Court declared RA 342 unconstitutional. In 1959,
PNB filed a suit for the payment of the loan. Plaintiff countered with his suit against
both defendants on August 10, 1959, her main allegation being that the mortgage
sought to be foreclosed had long been prescribed, fifteen years having elapsed from the
date of maturity, July 19, 1944.

Issue:

W/N the PNB can still collect from de Agbayani?

Ruling:

Yes. Due to the declaration of unconstitutionality of RA 342, all its effects scratch and
are deemed non – existent. There might be cases wherein not all the effect of an
unconstitutional law might be erased but in the case at hand the extension that de
Agbayani got from RA no. 342 extending Debt Moratorium were deemed invalid.

The orthodox view is that an unconstitutional act, for that matter an executive order or
a municipal ordinance likewise suffering from that infirmity, cannot be the source of
any legal rights or duties. Nor can it justify any official act taken under it. Its
repugnancy to the fundamental law once judicially declared results in its being to all
intents and purposes a mere scrap of paper.
FLORES VS. DRILON
RYAN JUANITO GARCIA

FACTS:
The constitutionality of Sec. 13, par. (d), of R.A. 7227, under which respondent Mayor
Richard J. Gordon of Olongapo City was appointed Chairman and Chief Executive Officer of the
Subic Bay Metropolitan Authority (SBMA), was challenged in the petition with prayer for
prohibition, preliminary injunction and temporary restraining order "to prevent useless and
unnecessary expenditures of public funds by way of salaries and other operational expenses
attached to the office. Petitioners maintain that the proviso in par. (d) of Sec. 13 infringed on
the following constitutional and statutory provisions: (a) Sec. 7, first par., Art. IX-B, of the
Constitution, which states that "no elective official shall be eligible for appointment or
designation in any capacity to any public officer or position during his tenure," because the
City Mayor of Olongapo City is an elective official and the subject posts are public offices; (b)
Sec. 16, Art. VII, of the Constitution, which provides that "the President shall . . . . appoint all
other officers of the Government whose appointments are not otherwise provided for by law,
and those whom he may be authorized by law to appoint", since it was Congress through the
questioned proviso and not the President who appointed the Mayor to the subject posts; and,
(c) Sec. 261, par. (g), of the Omnibus Election Code. The principal question is whether the
proviso in Sec. 13, par. (d), of R.A. 7227 which states, "Provided, however, that for the first
year of its operations from the effectivity of this Act, the mayor of the City of Olongapo shall
be appointed as the chairman and chief executive officer of the Subic Authority," violates the
constitutional proscription against appointment or designation of elective officials to other
government posts.

ISSUE:
Whether or not the proviso violates the constitutional proscription against
appointment or designation of elective officials to other government posts.

RULING:
YES, the subject proviso directs the President to appoint an elective official to other
government posts. It seeks to prevent a situation where a local elective official will work for
his appointment in an executive position in government, and thus neglect his constituents.
This is precisely what the Constitution prohibits. Since this is precisely what the constitutional
proscription seeks to deter, it needs no stretching of the imagination to conclude that the
proviso contravenes Sec. 7, first par., Art. IX-B, of the Constitution, which provides: “No
elective official shall be eligible for appointment or designation in any capacity to any public
office or position during his tenure. Unless otherwise allowed by law or by the primary
functions of his position, no appointive official shall hold any other office or employment in
the Government or any subdivision, agency or instrumentality thereof, including
government-owned or controlled corporations or their subsidiaries.
HACIENDA LUISITA INC. VS. PRESIDENTIAL
AGRARIAN REFORM COUNCIL
NAME OF DIGESTER
TOPIC:
FACTS:
ISSUE:
RULING:
COCOFED VS. REPUBLIC
NAME OF DIGESTER
TOPIC:
FACTS:
ISSUE:
RULING:
ARAULLO, ET. AL. VS. AQUINO III, ET. AL.
NAME OF DIGESTER
TOPIC:
FACTS:
ISSUE:
RULING:
IV. THE PHILIPPINES AS A STATE
CIR VS. RUEDA
ABRYL SYMON D. CULAS
(slight revisions by Adrian Lo)

TOPIC: ELEMENTS OF A STATE: TERRITORY, PEOPLE, SOVEREIGNTY,


GOVERNMENT (Page 7 of Syllabus)

FACTS: Antonio Campos Rueda is the administrator of the estate of the deceased
Doña Maria de la Estrella Soriano Vda. de Cerdeira. Collector of Internal Revenue
(CIR) issued a decision assessing against and demanding from the Rueda the sum
P161,874.95 as deficiency estate and inheritance taxes, including interest and
penalties, on the transfer of intangible personal properties situated in the Philippines
and belonging to said Maria de Cerdeira who is a Spanish national of Tangier,
Morocco. Intangible Personal Properties were claimed as exempt from taxes. CIR
insists that the law of Tangier is not reciprocal to Section 122 of the National Internal
Revenue Code.

CIR insists that Tangier was not a foreign country within the meaning of Section 122 of
the National Internal Revenue Code, but a mere principality.

The Provison in Section 122 of the National Internal Revenue Code reads: "That no tax
shall be collected under this Title in respect of intangible personal property (a) if the
decedent at the time of his death was a resident of a foreign country which at the time of
his death did not impose a transfer tax or death tax of any character in respect of
intangible person property of the Philippines not residing in that foreign country, or (b) if
the laws of the foreign country of which the decedent was a resident at the time of his
death allow a similar exemption from transfer taxes or death taxes of every character
in respect of intangible personal property owned by citizens of the Philippines not
residing in that foreign country."

ISSUE: W/N Tangier is bereft of international personality to be excluded from Section
122 of NIRC

RULING: No. The Court did commit itself to the doctrine that even a tiny principality,
that of Liechtenstein, hardly an international personality in the sense, did fall under this
exempt category.

​In the case of Collector of Internal Revenue v. De Lara, there can be no doubt that
California as a state in the American Union was in the alleged requisite of international
personality. Nonetheless, it was held to be a foreign country within the meaning of
Section 122 of the National Internal Revenue Code.
SAGUISAG VS. EXECUTIVE SECRETARY
ABRYL SYMON D. CULAS
TOPIC: NATIONAL TERRITORY (Page 7 of Syllabus)
FACTS: Petitioners question the constitutionality of the Enhanced Defense
Cooperation Agreement (EDCA) between the Republic of the Philippines and the
United States of America claiming that the instrument violated multiple constitutional
provisions.

ISSUE: W/N the provisions under EDCA are consistent with the Constitution, as well
as with existing laws and treaties

RULING: Yes. Under EDCA, an immovable property is owned by the Philippines, even
if built completely on the back of U.S. funding. This is consistent with the constitutional
prohibition on foreign land ownership.

First, The U.S. under EDCA does not enjoy any right of jurisdiction over any part of
the Philippines in which its forces or equipment may be found;
Second, In EDCA, U.S. access is purely at the invitation of the Philippines;
Third, in EDCA, Philippines is guaranteed access over the entire area of the Agreed
Locations;
Fourth, under EDCA, there is no such grant of power or authority over the
establishment, use, operation, defense, and control of military bases, including the
limits of territorial waters and air space adjacent to or in the vicinity of those bases. It
merely allows the U.S. to exercise operational control over the construction of
Philippine-owned structures and facilities;
Fifth, No right to staging areas, bombing and gunnery ranges is given under EDCA;
Sixth, The U.S. does not have any right, power, or authority to control and prohibit the
movement and operation of all types of vehicles within the vicinity of the bases;
Seventh, under EDCA, the U.S. is merely given temporary access to land and
facilities (including roads, ports, and airfields);
Eighth, it merely extends to U.S. forces temporary access to public land and facilities
when requested;
Ninth,under EDCA, the U.S. no longer has the right, power, and authority to construct,
install, maintain, and employ any type of facility, weapon, substance, device, vessel or
vehicle, or system unlike in the old treaty. EDCA merely grants the U.S., through
bilateral security mechanisms, the authority to undertake construction, alteration, or
improvements on the Philippine-owned Agreed Locations;
Tenth, EDCA does not allow the U.S. to acquire, by condemnation or expropriation
proceedings, real property belonging to any private person;
Eleventh, EDCA does not allow the U.S. to unilaterally bring into the country
non-Philippine nationals who are under its employ, together with their families, in
connection with the construction, maintenance, or operation of the bases. EDCA
strictly adheres to the limits under the VFA;
Twelfth, EDCA does not allow the U.S. to exercise jurisdiction over any offense
committed by any person within the Agreed Locations; and
Thirteenth, EDCA does not allow the U.S. to operate military post exchange (PX)
facilities, which is free of customs duties and taxes.
NICOLAS VS. ROMULO
ABRYL SYMON D. CULAS
TOPIC:

FACTS: Respondent Lance Corporal Daniel Smith is a member of the US Armed Forces.
He was charged with the crime of rape committed against a 22 year old Filipina, herein
petitioner Suzette Nicolas. Taking advantage of her intoxication, Nicolas was raped inside
a Starex Van.

Pursuant to the Visiting Forces Agreement (VFA) between the Republic of the Philippines
and the United States, entered into on February 10, 1998, the United States, at its
request, was granted custody of defendant Smith pending the proceedings.

Smith was taken out of the Makati Jail by enforcement agents purportedly acting under
orders of the Department of the Interior and Local Government, and brought to a facility for
detention under the control of the United States government.

Petitioners contend that the Philippines should have custody of defendant L/CPL Smith
because, first of all, the VFA is void and unconstitutional.

ISSUE: W/N Corporal Smith is immune from the local jurisdiction of our COurts

RULING: The rule in international law is that a foreign armed forces allowed to enter
one’s territory is immune from local jurisdiction, except to the extent agreed upon. The
Status of Forces Agreements involving foreign military units around the world vary in terms
and conditions, according to the situation of the parties involved, and reflect their
bargaining power. But the principle remains, i.e., the receiving State can exercise
jurisdiction over the forces of the sending State only to the extent agreed upon by the
parties.

To prevent a recurrence of this experience, Sec. 25 of ARTICLE XVIII was adopted in the
1987 Constitution. The provision is thus designed to ensure that any agreement allowing
the presence of foreign military bases, troops or facilities in Philippine territory shall be
equally binding on the Philippines and the foreign sovereign State involved. The idea is to
prevent a recurrence of the situation in which the terms and conditions governing the
presence of foreign armed forces in our territory were binding upon us but not upon the
foreign State.
MAGALLONA VS. ERMITA
Digested By: Alyana A. Mahilum
TOPIC: Method of Determining the Baselines

FACTS:

RA 3046 was passed in 1961 which provides among others the demarcation lines of the
baselines of the Philippines as an archipelago. This is in consonance with UNCLOS I.

RA 5446 amended RA 3046 in terms of typographical errors and included Section 2 in which
the government reserved the drawing of baselines in Sabah in North Borneo.

RA 9522 took effect in March 2009 amending RA 5446. The amendments, which are in
compliance with UNCLOS III in which the Philippines is one of the signatory, shortening one
baseline while optimizing the other and classifying Kalayaan Group of Island and Scarborough
Shoal as Regimes of Island.

Petitioners in their capacity as taxpayer, citizen and legislator assailed the constitutionality of
RA 9522:- it reduces the territory of the Philippines in violation to the Constitution and it opens
the country to maritime passage of vessels and aircrafts of other states to the detriment of the
economy, sovereignty, national security and of the Constitution as well. They added that the
classification of Regime of Islands would be prejudicial to the lives of the fishermen.

ISSUE:

1. Whether or not the petitioners have locus standi to bring the suit; and
2. Whether or not RA 9522 is unconstitutional

RULING:

Petition is dismissed.

1. The SC ruled the suit is not a taxpayer or legislator, but as a citizen suit, since it is the
citizens who will be directly injured and benefitted in affording relief over the remedy sought.

2. The SC upheld the constitutionality of RA 9522.

RA 9522 did not delineate the territory of the Philippines but is merely a statutory tool to
demarcate the country’s maritime zone and continental shelf under UNCLOS III. Also, the
classification of KGI and Scarborough Shoal as Regime of Islands is consistent with the
Philippines’ sovereignty. Had RA 9522 enclosed the islands as part of the archipelago, the
country would be violating UNCLOS III since it categorically stated that the length of the
baseline shall not exceed 125 nautical miles. Thirdly, the new base line introduced by RA 9522
is without prejudice with delineation of the baselines of the territorial sea around the territory of
Sabah, situated in North Borneo, over which the Republic of the Philippines has acquired
dominion and sovereignty. The enactment of UNCLOS III compliant baselines law for the
Philippine archipelago and adjacent areas, as embodied in RA 9522, allows an
internationally-recognized delimitation of the breadth of the Philippines’ maritime zones and
continental shelf. RA 9522 is therefore a most vital step on the part of the Philippines in
safeguarding its maritime zones, consistent with the Constitution and our national interest.
ARIGO VS. SWIFT
ADRIAN WILLIAM LO

TOPIC: TERRITORY - EXCLUSIVE ECONOMIC ZONE

FACTS:

On the way to Indonesia from Subic Bay, the USS Guardian (an American
warship)
Ran aground in the TUBBATAHA REEF. There were no injuries, no leaking fuel or
oil.

Petitioners Arigo et al claim that the grounding, salvaging, and post-salvaging


operations are damaging the environment.

ISSUE: WoN the respondents may be held liable for damages caused by USS
Guardian

RULING: Senior Associate Justice Antonio T. Carpio took the position that the conduct of
the US in this case, when its warship entered a restricted area in violation of R.A. No.
10067 and caused damage to the TRNP reef system, brings the matter within the ambit
of Article 31 of the United Nations Convention on the Law of the Sea (UNCLOS).

He explained that while historically, warships enjoy sovereign immunity from suit as
extensions of their flag State, Art. 31 of the UNCLOS creates an exception to this rule.

Article 31 - Responsibility of the flag State for damage caused by a warship or other
government ship operated for non-commercial purposes: The flag State shall bear
international responsibility for any loss or damage to the coastal State resulting from
the non-compliance by a warship or other government ship operated for non-commercial
purposes with the laws and regulations of the coastal State concerning passage through
the territorial sea or with the provisions of this Convention or other rules of international
law.

We fully concur with Justice Carpio's view that non-membership in the UNCLOS does
not mean that the US will disregard the rights of the Philippines as a Coastal State
over its internal waters and territorial sea. We thus expect the US to bear "international
responsibility" under Art. 31 in connection with the USS Guardian grounding which
adversely affected the Tubbataha reefs.
QUA CHEE GAN VS. DEPORTATION BOARD
ADRIAN WILLIAM LO
TOPIC: PEOPLE

FACTS:

The Court of First Instance denied the petition for writs of habeas corpus, mandamus
and certiorari by the petitioners (who are aliens). On May 12, 1952, Special
Prosecutor Emilio L. Galang charged petitioner before the respondent Deportation
Board for these crimes:

- Purchasing $130,000 with license from Central Bank and remitting the same t to
Hong Kong
- Attempted bribery of PH and US officials.

A member of the Deportation Board issued a warrant of arrest for the petitioners.

ISSUE: WoN aliens residing in the Philippines are accorded the same protection
against unlawful searches and seizures as citizens are.

RULING: Yes, they are also protected from unlawful searches and seizures.

Article 3, Section 2. The right of the people to be secure in their persons, houses,
papers, and effects against unreasonable searches and seizures of whatever nature
and for any purpose shall be inviolable, and no search warrant or warrant of arrest
shall issue except upon probable cause to be determined personally by the judge
after examination under oath or affirmation of the complainant and the witnesses he
may produce, and particularly describing the place to be searched and the persons or
things to be seized.

PEOPLE = Refer to inhabitants of the country. This includes FOREIGNERS, not just
citizens.

WARRANT OF ARREST = shall issue only when determined personally by the


JUDGE, so the member of the Deportation Board should not have issued the warrant.

IN VIEW OF THE FOREGOING, Executive Order No. 398, series of 1951, insofar as it
empowers the Deportation Board to issue warrant of arrest upon the filing of formal
charges against an alien or aliens and to fix bond and prescribe the conditions for the
temporary release of said aliens, is declared illegal. As a consequence, the order of
arrest issued by the respondent Deportation Board is declared null and void and the
bonds filed pursuant to such order of arrest, decreed cancelled. With the foregoing
modification, the decision appealed from is hereby affirmed. No costs. So ordered.
TECSON AND DESIDERIO VS. COMELEC
Lig Andre S. Nuesca

TOPIC: Citizenship (Historical Background)


FACTS: On 31 December 2003, respondent Ronald Allan Kelly Poe, also known as Fernando
Poe, Jr. (hereinafter "FPJ"), filed his certificate of candidacy for the position of President of the
Republic of the Philippines.

A petition docketed SPA No. 04-003 before the Commission on Elections ("COMELEC") to
disqualify FPJ and to deny due course or to cancel his certificate of candidacy upon the thesis
that FPJ made a material misrepresentation in his certificate of candidacy by claiming to
be a natural-born Filipino citizen when in truth, according to Fornier, his parents were
foreigners; his mother, Bessie Kelley Poe, was an American, and his father, Allan Poe,
was a Spanish national, being the son of Lorenzo Pou, a Spanish subject. Granting,
petitioner asseverated, that Allan F. Poe was a Filipino citizen, he could not have transmitted
his Filipino citizenship to FPJ, the latter being an illegitimate child of an alien mother. Petitioner
based the allegation of the illegitimate birth of respondent on two assertions - first, Allan F. Poe
contracted a prior marriage to a certain Paulita Gomez before his marriage to Bessie Kelley
and, second, even if no such prior marriage had existed, Allan F. Poe, married Bessie Kelly
only a year after the birth of respondent.

ISSUE: Whether or not FPJ is natural-born Filipino citizen

RULING: The documentary evidence introduced by no less than respondent himself,


consisting of a birth certificate of respondent and a marriage certificate of his parents showed
that FPJ was born on 20 August 1939 to a Filipino father and an American mother who were
married to each other a year later, or on 16 September 1940. Birth to unmarried parents would
make FPJ an illegitimate child. Petitioner contended that as an illegitimate child, FPJ so
followed the citizenship of his mother, Bessie Kelley, an American citizen.

Where jurisprudence regarded an illegitimate child as taking after the citizenship of its mother,
it did so for the benefit of the child. It was to ensure a Filipino nationality for the illegitimate child
of an alien father in line with the assumption that the mother had custody, would exercise
parental authority and had the duty to support her illegitimate child. It was to help the child, not
to prejudice or discriminate against him.

While tracing respondents’ paternal lineage, the Filipino citizenship of Lorenzo Pou could only
be drawn from the presumption that having died in 1954 at 84 years old, Lorenzo would have
been born sometime in the year 1870, when the Philippines was under Spanish rule, and that
San Carlos, Pangasinan, his place of residence upon his death in 1954, in the absence of any
other evidence, could have well been his place of residence before death, such that Lorenzo
Pou would have benefited from the "en masse Filipinization" that the Philippine Bill had
effected in 1902.

That citizenship (of Lorenzo Pou), if acquired, would thereby extend to his son, Allan F. Poe,
father of respondent FPJ. The 1935 Constitution, during which regime respondent FPJ has
seen first light, confers citizenship to all persons whose fathers are Filipino citizens regardless
of whether such children are legitimate or illegitimate.
GO SR. VS. RAMOS
Louise Francis P. Ouano

TOPIC:
Citizenship Proceedings and Res Adjudicata, Exceptions

FACTS:
These petitions stemmed from the complaint-affidavit for deportation initiated by Luis
T. Ramos before the Bureau of Immigration and Deportation against Jimmy T. Go
alleging that the latter is an illegal and undesirable alien.

Luis alleged that while Jimmy represents himself as a Filipino citizen, Jimmy’s
personal circumstances and other records indicate that he is not so. To prove his
contention, Luis presented the birth certificate of Jimmy, issued by the Office of
the Civil Registrar of Iloilo City, which indicated Jimmy’s citizenship as
"Chinese."

Luis argued that although it appears from Jimmy’s birth certificate that his parents,
Carlos and Rosario Tan, are Filipinos, the document seems to be tampered, because
only the citizenship of Carlos appears to be handwritten while all the other entries
were typewritten. He also averred that in September 1989 or thereabout, Jimmy,
through stealth, machination and scheming managed to cover up his true citizenship,
and with the use of falsified documents and untruthful declarations, was able to
procure a Philippine passport from the Department of Foreign Affairs.

ISSUE:
Whether or Not res judicata applies to decisions on citizenship. NO

RULING:
Carlos and Jimmy’s claim that the cause of action of the Bureau has prescribed
is untenable. Cases involving issues on citizenship are sui generis. Once the
citizenship of an individual is put into question, it necessarily has to be threshed out
and decided upon. In the case of Frivaldo v. Commission on Elections, we said
that decisions declaring the acquisition or denial of citizenship cannot govern a
person’s future status with finality. This is because a person may subsequently
reacquire, or for that matter, lose his citizenship under any of the modes recognized by
law for the purpose. Indeed, if the issue of one’s citizenship, after it has been passed
upon by the courts, leaves it still open to future adjudication, then there is more reason
why the government should not be precluded from questioning one’s claim to
Philippine citizenship, especially so when the same has never been threshed out by
any tribunal.
Citizenship proceedings, are a class of its own, in that, unlike other cases, res judicata
does not obtain as a matter of course. In a long line of decisions, this Court said that
every time the citizenship of a person is material or indispensable in a judicial
or administrative case, whatever the corresponding court or administrative
authority decides therein as to such citizenship is generally not considered as
res judicata; hence, it has to be threshed out again and again as the occasion
may demand.

Res judicata may be applied in cases of citizenship only if the following concur:
1. a person’s citizenship must be raised as a material issue in a controversy
where said person is a party;
2. the Solicitor General or his authorized representative took active part in the
resolution thereof; and
3. the finding or citizenship is affirmed by the Supreme Court.

One of the arguments raised to sustain Carlos’ claim to Philippine citizenship is the
doctrine of jus soli, or the doctrine or principle of citizenship by place of birth. To recall,
both the trial court and the Court of Appeals ruled that the doctrine of jus soli was
never extended to the Philippines.

We agree. The doctrine of jus soli was for a time the prevailing rule in the acquisition
of one’s citizenship. However, the Supreme Court abandoned the principle of jus soli
in the case of Tan Chong v. Secretary of Labor. It is a settled rule that only legitimate
children follow the citizenship of the father and that illegitimate children are under the
parental authority of the mother and follow her nationality.

Moreover, we have also ruled that an illegitimate child of a Filipina need not perform
any act to confer upon him all the rights and privileges attached to citizens of the
Philippines; he automatically becomes a citizen himself.

However, it is our considered view that absent any evidence proving that Carlos is
indeed an illegitimate son of a Filipina, the aforestated established rule could not be
applied to him. On the other hand, the mere fact that he was able to vote does not
validate his irregular election of Philippine citizenship. At most, his registration as a
voter indicates his desire to exercise a right appertaining exclusively to Filipino citizens
but does not alter his real citizenship, which, in this jurisdiction, is determined by blood
(jus sanguinis).

The exercise of the rights and privileges granted only to Filipinos is not conclusive
proof of citizenship, because a person may misrepresent himself to be a Filipino and
thus enjoy the rights and privileges of citizens of this country. It is incumbent upon one
who claims Philippine citizenship to prove to the satisfaction of the court that he is
really a Filipino. No presumption can be indulged in favor of the claimant of Philippine
citizenship, and any doubt regarding citizenship must be resolved in favor of the state.
GONZALEZ VS. PENNISI
Digested by: Vinceromae Allysson Reveche

TOPIC: Citizenship Proceedings and Res Judicata

FACTS:

Michael Pennisi was born on March 13, 1975 in Queensland Australia to Alfio Pennisi, an
Australian National, and Anita T. Quintos, allegedly a Filipino Citizen. In March 1999,
respondent filed a petition for recognition as Filipino before the Bureau of Immigration, for
which was granted to him after he submitted all the necessary documents.

In 2003, the Senate Committee on Games, Amusement and Sports and Constitutional
Amendments jointly submitted a report recommending that the B.I conduct summary
deportation proceedings against several Filipino-foreign PBA players, and that the DOJ
secretary conduct a review of all orders of recognition. Respondent Penissi was included in the
list, the reason being the authenticity of the document that he presented, particularly his
mother’s birth certificate, was questionable.

Following this, the DOJ issued a resolution revoking the respondent's certificate of
recognition. An appeal was filed before the CA by the respondent. After the hearing, which
transpired in 2005, CA affirmed the first decision of the BI and DOJ retaining Pennisi’s
status as a Filipino. Petitioner contends that CA committed a reversible error in finding that
respondent is a Filipino citizen.

ISSUE:

1.) Whether or not Res Judicata is applicable to the decisions of respondent’s citizenship.
2.) Whether or not the respondent's mother, Anita Quintos’ birth certificate is valid.

RULING:

1. No. Res Judicata is not applicable to the decisions of respondent’s citizenship. We agree
with petitioners that the issuance of certificate of recognition to respondent has not attained
finality. In Go v. Ramos, the Court ruled that citizenship proceedings are a class of its own
and can be threshed out again and again as the occasion may demand. Res judicata may
be applied in cases of citizenship only if the following concur:

1. a person’s citizenship must be raised as a material issue in a controversy where said person
is a party;
2. the Solicitor General or his authorized representative took active part in the resolution
thereof; and
3. the finding or citizenship is affirmed by this Court.

However, the courts are not precluded from reviewing the findings of the Bureau of
Immigration. Judicial review is permitted if the courts believe that there is substantial
evidence supporting the claim of citizenship, so substantial that there are reasonable
grounds for the belief that the claim is correct.

Note:
Res Judicata → a matter that has been adjudicated by a competent court and may not be pursued further by the
same parties.

2. Yes. We agree with the Court of Appeals that while the affidavits of Soliman and Peralta
(their claim that there are no Quintoses or Tomedas that have lived or have resided in the said
barangay) might have cast doubt on the validity of Quintos’ certificate of live birth, such
certificate remains valid unless declared invalid by competent authority. The rule stands
that "(d) documents consisting of entries in public records made in the performance of a
duty by a public officer are prima facie evidence of the facts stated therein.

We further sustain the Court of Appeals that there could be reasons why the Quintoses and
Tomedas were not included in the census, such as they could have been mere transients
in the place. As for their absence in the master’s list of voters, they could have failed to
register themselves as voters. The late registration of Quintos’ certificate of live birth was
made 10 years after her birth and not anytime near the filing of respondent’s petition for
recognition as Filipino citizen. As such, it could not be presumed that the certificate’s late filing
was meant to use it fraudulently. Finally, the Australian Department of Immigration and
Multicultural Affairs itself attested that as of 14 July 1999, Quintos has not been granted
Australian citizenship. Respondent submitted a certified true copy of Quintos’ Australian
certificate of registration of alien, indicating her nationality as Filipino. These pieces of evidence
should prevail over the affidavits submitted by Soliman and Peralta to the Senate Committees.

Petition DENIED.
RE: APPLICATION FOR ADMISSION TO THE
PHILIPPINE BAR, VICENTE D. CHING

FACTS: Vicente Ching, the legitimate son of spouses Tat Ching, a Chinese citizen, and
Prescila A. Dulay, a Filipino, was born in Francia West, Tubao, La Union on April 11, 1964.
Since his birth, Ching has resided in the Philippines. Having completed his Bachelor of Laws at
St. Louis University in Baguio city, he filed an application to take the 1998 Bar Examinations.

The Court allowed him to take the Bar Examinations subject to the condition that he
must submit to the Court proof of his citizenship. Incompliance, Ching submitted 1.)
Certification showing that he is a CPA, 2.) Voter Certification, 3.) Certification that Ching
was elected as a member of the Sangguniang Bayan of Tubao, La Union during the May
12, 1992 synchronized elections.
Ching was one of the successful passers of the Bar Examination. However, because of the
questionable status of Ching’s citizenship, he was not allowed to take his oath.

The OSG filed its comment stating that Ching, being the legitimate child of a Chinese father
and a Filipino mother born under the 1935 Constitution was a Chinese citizen and continued to
do so, unless upon reaching the age of majority he elected Philippine citizenship. The OSG
adds that what Ching acquired at best was only an inchoate Philippine citizenship which he
could perfect by election upon reaching the age of majority. The OSG clarifies that 2 conditions
must concur in order that the election of Philippine citizenship may be effective, namely: a) the
mother of the person making the election must be a citizen of the Philippines; and b) said
election must be made upon reaching the age of majority.

ISSUE: Whether the delay of 14 years from Ching’s age of majority to his
election be considered a reasonable time under the 2nd condition?

HELD: The clause “upon reaching the age of majority” has been construed to mean a
reasonable time after reaching the age of majority which had been interpreted by the Secretary
of Justice to be 3 years. Said period may be extended under certain circumstances. But in
Cuenco, it was held that an election done after over 7 years was not made within a reasonable
time. Since Ching was born in 1964, the governing charter was the 1935 Constitution and
pursuant to the provisions of Article IV of the 1935 Constitution, Ching is a Chinese citizen
having a Chinese father. In the present case, Ching was already 35 years old when he
complied with the requirements of CA No 625 on June 15, 1999 or over 14 years after he had
reached the age of majority. Ching’s election was clearly beyond, by any reasonable yardstick,
the allowable period within which to exercise the privilege. The Court resolves to DENY Vicente
Ching’s application for admission to the Philippine Bar.

REPUBLIC VS. LIM


Maizelle Salialam
TOPIC: Election of Philippine Citizenship

FACTS:
This petition for review on certiorari under Rule 45 of the Rules of Court stemmed from a petition for
correction of entries under Rule 108 of the Rules of Court filed by respondent Chule Y. Lim.

In her petition, respondent claimed that she was born on October 29, 1954 in Buru-an, Iligan City. Her
birth was registered in Kauswagan, Lanao del Norte but the Municipal Civil Registrar of Kauswagan
transferred her record of birth to Iligan City. She alleged that both her Kauswagan and Iligan City records
of birth have four erroneous entries, and prays that they be corrected:
1. She claims that her surname "Yu" was misspelled as "Yo". She has been using "Yu" in all her
school records and in her marriage certificate. She presented a clearance from the National
Bureau of Investigation (NBI)3 to further show the consistency in her use of the surname "Yu".
2. She claims that her father’s name in her birth record was written as "Yo Diu To (Co Tian)" when it
should have been "Yu Dio To (Co Tian)."
3. Her nationality was entered as Chinese when it should have been Filipino considering that her
father and mother never got married. Only her deceased father was Chinese, while her mother is
Filipina. She claims that her being a registered voter attests to the fact that she is a Filipino
citizen.
4. Finally, it was erroneously indicated in her birth certificate that she was a legitimate child when
she should have been described as illegitimate considering that her parents were never married.

The RTC granted the petition and the Civil Registrar of Iligan City is directed to make the corrections in
the birth records of the petitioner.

The Republic of the Philippines appealed the decision to the Court of Appeals, which affirmed the trial
court’s decision.

ISSUE: Whether or not the Court of Appeals erred in ordering the correction of the citizenship of
respondent Chule Y. Lim from Chinese to Filipino despite the fact that respondent never demonstrated
any compliance with the legal requirements for election of citizenship.

RULING:

No, the CA did not err. Plainly, the provisions of Article IV, Section 1(3) of the 1935 Constitution regarding
the constitutional and statutory requirements of electing Filipino citizenship apply only to legitimate
children. These do not apply in the case of respondent who was concededly an illegitimate child,
considering that her Chinese father and Filipino mother were never married. As such, she was not
required to comply with said constitutional and statutory requirements to become a Filipino citizen. By
being an illegitimate child of a Filipino mother, respondent automatically became a Filipino upon birth.
Stated differently, she is a Filipino since birth without having to elect Filipino citizenship when she reached
the age of majority. This notwithstanding, the records show that respondent elected Filipino citizenship
when she reached the age of majority. She registered as a voter in Misamis Oriental when she was 18
years old. The exercise of the right of suffrage and the participation in election exercises constitute a
positive act of election of Philippine citizenship.
MA VS. FERNANDEZ, JR.
Link to full text: GR No. 183133

Digested by: Kaye Turija

TOPIC: Part 4 PH as a State > C. People > Election of Philippine Citizenship

FACTS:

● Records reveal that petitioners Felix, Jr., Balgamelo, and Valeriano (children of
Felix (Yao Kong) Ma, a Taiwanese, and Dolores Sillona Cabiling, a Filipina)
were all born under aegis of the 1935 Philippine Constitution in the years 1948,
1951, and 1957, respectively. Petitioners are children of a Taiwanese father
and a Filipino mother all of whom were born under the 1935 Philippine
Constitution.

● They were all raised in the Philippines and have resided in this country for
almost sixty (60) years; they spent their whole lives, studied and received their
primary and secondary education in the country; they do not speak nor
understand the Chinese language, have not set foot in Taiwan, and do not
know any relative of their father; they have not even traveled abroad; and they
have already raised their respective families in the Philippines.

● Immediately upon reaching the age of twenty-one, they claimed Philippine


citizenship in accordance with Section 1(4), Article IV, of the 1935
Constitution. Thus, on 15 August 1969, Felix, Jr. executed his affidavit of
election of Philippine citizenship and took his oath of allegiance. On 14
January 1972, Balgamelo did the same. In 1978, Valeriano took his oath of
allegiance.

● Thus, they secured from the Bureau of Immigration their Alien Certificates of
Registration (ACRs). Having taken their oath of allegiance as Philippine
citizens, petitioners, however, failed to have the necessary documents
registered in the civil registry as required under Section 1 of Commonwealth
Act No. 625 (An Act Providing the Manner in which the Option to Elect
Philippine Citizenship shall be Declared by a Person whose Mother is a Filipino
Citizen).

● It was only on 27 July 2005 or more than thirty (30) years after they elected
Philippine citizenship that Balgamelo and Felix, Jr. did so. On the other hand,
there is no showing that Valeriano complied with the registration requirement.
ISSUE: W/N petitioners are improperly documented aliens? (Are petitioners herein
Filipino Citizens?) → [Discrepancy: The two documents which were executed immediately within reasonable
time upon reaching the age of majority were not registered. There was a failure to register them with the nearest local
civil registry.]

RULING: The Supreme Court said that the failure to register is not a fatal defect.
What is constitutive of election is the execution of the required documents. The
registration requirement is simply for recording purposes of the election made within
reasonable time from the attainment of the age of majority. So for as long as it can be
shown that there really was, like in the case of Ma, the documents were considered
ancient documents. The failure to register was not considered.

The court ruled that under the facts peculiar to the petitioners, the right to elect
Philippine citizenship has not been lost and they should be allowed to complete the
statutory requirements for such election. Petitioners are given ninety (90) days from
notice within which to COMPLYwith the requirements of the Bureau of Immigration.

Additional but optional:

The 1935 Constitution declares as citizens of the Philippines those whose mothers are
citizens of the Philippines and elect Philippine citizenship upon reaching the age of
majority.
The mandate states:

Section 1. The following are citizens of thePhilippines:


(1)xxx;
xxxx
(4) Those whose mothers are citizens of thePhilippinesand, upon reaching the age of
majority, elect Philippine citizenship.

***

In 1941, Commonwealth Act No. 625 was enacted. It laid down the manner of electing
Philippine citizenship, to wit:

Section 1. The option to elect Philippine citizenship in accordance with subsection (4),
Section 1, Article IV, of the Constitution shall be expressed in a statement to be signed
and sworn to by the party concerned before any officer authorized to administer oaths,
and shall be filed with the nearest civil registry.The said party shall accompany the
aforesaid statement with the oath of allegiance to the Constitution and the
Government of thePhilippines.

The statutory formalities of electing Philippine citizenship are: (1) a statement of


election under oath; (2) an oath of allegiance to the Constitution and Government
of the Philippines; and (3) registration of the statement of election and of the oath
with the nearest civil registry.
REPUBLIC VS. SAGUN
Kyra Valentin
TOPIC: Election of Philippine Citizenship
FACTS: Nora Fe Sagun is the legitimate child of Albert S. Chan, a Chinese national, and Marta Borromeo, a
Filipino citizen. She was born on August 8, 1959 in Baguio City and did not elect Philippine citizenship upon
reaching the age of majority. In 1992, at the age of 33 and after getting married to Alex Sagun, she executed an
Oath of Allegiance to the Republic of the Philippines. Said document was notarized by Atty. Cristeta Leungon but
was not recorded and registered with the Local Civil Registrar of Baguio City.

Sometime in September 2005, respondent applied for a Philippine passport. Her application was denied due to the
citizenship of her father and there being no annotation on her birth certificate that she has elected Philippine
citizenship. Consequently, she sought a judicial declaration of her election of Philippine citizenship averring that
she was raised as a Filipino and she is a registered voter of Precinct No. 0419A of Barangay Manuel A. Roxas in
Baguio City and had voted in local and national elections as shown in the Voter Certification. She asserted that by
virtue of her positive acts, she has effectively elected Philippine citizenship and such fact should be annotated on
her record of birth so as to entitle her to the issuance of a Philippine passport.

After conducting a hearing, the trial court rendered the assailed Decision on April 3, 2009 granting the petition and
declaring respondent a Filipino citizen.

Upon payment of the required fees, the Local Civil Registrar of Baguio City is hereby directed to annotate [on] her
birth certificate, this judicial declaration of Filipino citizenship of said petitioner.

Petitioner, through the OSG, directly filed the instant recourse via a petition for review on certiorari before us.
Petitioner points out that while respondent executed an oath of allegiance before a notary public, there was no
affidavit of her election of Philippine citizenship. Additionally, her oath of allegiance which was not registered with
the nearest local civil registry was executed when she was already 33 years old or 12 years after she reached the
age of majority

ISSUE: Is an action or proceeding for judicial declaration of Philippine citizenship procedurally and
jurisdictionally permissible?

Is an election of Philippine citizenship, made twelve (12) years after reaching the age of majority, considered to
have been made within a reasonable time as interpreted by jurisprudence?

RULING: Yes,
After conducting a hearing, the trial court rendered the assailed Decision on April 3, 2009 granting the petition and
declaring respondent a Filipino citizen. The fallo of the decision reads:

WHEREFORE, the instant petition is hereby GRANTED. Petitioner Nora Fe Sagun y Chan is hereby DECLARED [a]
FILIPINO CITIZEN, having chosen or elected Filipino citizenship.

Upon payment of the required fees, the Local Civil Registrar of Baguio City is hereby directed to annotate [on] her
birth certificate, this judicial declaration of Filipino citizenship of said petitioner.

WHEREFORE, the petition is GRANTED. The Decision dated April 3, 2009 of the Regional Trial Court, Branch 3 of
Baguio City in Spcl. Pro. Case No. 17-R is REVERSED and SET ASIDE. The petition for judicial declaration of election
of Philippine... citizenship filed by respondent Nora Fe Sagun is hereby DISMISSED for lack of merit.

Under our laws, there can be no action or proceeding for the judicial declaration of the citizenship of an individual.
Courts of justice exist for settlement of justiciable controversies, which imply a given right, legally demandable and
enforceable, an act or omission violative of said right, and a remedy, granted or sanctioned by law, for said breach
of right. As an incident only of the adjudication of the rights of the parties to a controversy, the court may pass
upon, and make a pronouncement relative to their status. Otherwise, such a pronouncement is beyond judicial
power.

Clearly, it was erroneous for the trial court to make a specific declaration of respondents Filipino citizenship as such
pronouncement was not within the court's competence.

As to the propriety of respondent's petition seeking a judicial declaration of election of Philippine citizenship, it is
imperative that we determine whether respondent is required under the law to make an election and if so,
whether she has complied with the procedural requirements in the election of Philippine citizenship.

When respondent was born on August 8, 1959, the governing charter was the 1935 Constitution, which declares as
citizens of the Philippines those whose mothers are citizens of the Philippines and elect Philippine citizenship upon
reaching the age of majority. Sec. 1, Art. IV of the 1935 Constitution reads:

Section 1. The following are citizens of the Philippines:


xxxx

(4) Those whose mothers are citizens of the Philippines and, upon reaching the age of majority, elect Philippine
citizenship.

Under Article IV, Section 1(4) of the 1935 Constitution, the citizenship of a legitimate child born of a Filipino mother
and an alien father followed the citizenship of the father, unless, upon reaching the age of majority, the child
elected Philippine citizenship. The right to elect Philippine citizenship was recognized in the 1973 Constitution
when it provided that [t]hose who elect Philippine citizenship pursuant to the provisions of the Constitution of
nineteen hundred and thirty-five are citizens of the Philippines. Likewise, this recognition by the 1973 Constitution
was carried over to the 1987 Constitution which states that [t]hose born before January 17, 1973 of Filipino
mothers, who elect Philippine citizenship upon reaching the age of majority are Philippine citizens. It should be
noted, however, that the 1973 and 1987 Constitutional provisions on the election of Philippine citizenship should
not be understood as having a curative effect on any irregularity in the acquisition of citizenship for those covered
by the 1935 Constitution. If the citizenship of a person was subject to challenge under the old charter, it remains
subject to challenge under the new charter even if the judicial challenge had not been commenced before the
effectivity of the new Constitution.

Based on the foregoing circumstances, respondent clearly failed to comply with the procedural requirements for a
valid and effective election of Philippine citizenship. Respondent cannot assert that the exercise of suffrage and the
participation in election exercises constitutes a positive act of election of Philippine citizenship since the law
specifically lays down the requirements for acquisition of citizenship by election. The mere exercise of suffrage,
continuous and uninterrupted stay in the Philippines, and other similar acts showing exercise of Philippine
citizenship cannot take the place of election of Philippine citizenship. Hence, respondent cannot now be allowed to
seek the intervention of the court to confer upon her Philippine citizenship when clearly she has failed to validly
elect Philippine citizenship. As we held in Ching, the prescribed procedure in electing Philippine citizenship is
certainly not a tedious and painstaking process. All that is required of the elector is to execute an affidavit of
election of Philippine citizenship and, thereafter, file the same with the nearest civil registry. Having failed to
comply with the foregoing requirements, respondents petition before the trial court must be denied. (Re:
Application For Admission to the Philippine Bar. Vicente D. Ching, Bar Matter No. 914, October 1, 1999, 316 SCRA 1,
7-8.)

GRANTED
KILOSBAYAN VS. EXECUTIVE SECRETARY
Nalcot, MPAA
Full text link: https://lawphil.net/judjuris/juri2007/jul2007/gr_177721_2007.html

TOPIC: What offices under the Constitution require the official to be “Natural
Born”?

FACTS: Petitioners contend that the appointment extended to respondent Justice


Gregory Ong through respondent Executive Secretary Eduardo Ermita is patently
unconstitutional, arbitrary, whimsical and issued with grave abuse of discretion amounting
to lack of jurisdiction.

Petitioners claim that respondent Ong is a Chinese citizen, that this fact is plain and
incontestable, and that his own birth certificate indicates his Chinese citizenship.
Petitioners attached a copy of said birth certificate as Annex "H" to the petition. The birth
certificate, petitioners add, reveals that at the time of respondent Ong’s birth on May 25,
1953, his father was Chinese and his mother was also Chinese.

Petitioners invoke the Constitution:

Section 7 (1) of Article VIII of the 1987 Constitution provides that "No person shall be
appointed Member of the Supreme Court or any lower collegiate court unless he is a
natural-born citizen of the Philippines." Sec. 2 of Art. IV defines "natural-born citizens as
those who are citizens of the Philippines from birth without having to perform any act to
acquire or perfect their Philippine Citizenship."

ISSUE: WON Sandiganbayan Justice Gregory S. Ong is a natural-born citizen of the


Philippines.

RULING: No. It is clear, that from the records of this Court, respondent Ong is a
naturalized Filipino citizen. The alleged subsequent recognition of his natural-born status
by the Bureau of Immigration and the DOJ cannot amend the final decision of the trial
court stating that respondent Ong and his mother were naturalized along with his father.

Furthermore, as petitioners correctly submit, no substantial change or correction in an


entry in a civil register can be made without a judicial order, and, under the law, a change
in citizenship status is a substantial change. In Labayo-Rowe v. Republic, this Court held
that:

Changes which affect the civil status or citizenship of a party are substantial in character
and should be threshed out in a proper action depending upon the nature of the issues in
controversy, and wherein all the parties who may be affected by the entries are notified or
represented and evidence is submitted to prove the allegations of the complaint, and proof
to the contrary admitted.
Republic Act No. 9048 provides in Section 2 (3) that a summary administrative proceeding
to correct clerical or typographical errors in a birth certificate cannot apply to a change in
nationality. Substantial corrections to the nationality or citizenship of persons recorded in
the civil registry should, therefore, be effected through a petition filed in court under Rule
108 of the Rules of Court.
POE-LLAMANZARES VS. COMELEC
Angelica Wasawas

Topic: NATURAL-BORN CITIZENS

FACTS: Mary Grace Natividad Poe-Llamanzares (a.ka. Grace Poe), petitioner, wishes to run for
the Office of the President of the Republic of the Philippines. However, Grace Poe was a
foundling. It was a certain Edgardo Militar who found the infant Grace in Parish Church of
Jaro, Iloilo on September 3, 1968. Emiliano Militar (relative of Edgardo) and his wife, reported
and registered Grace Poe as a foundling with the Office of the Civil Registrar of Ilo-ilo City.

In 1991, Grace Poe married Teodoro Misael Daniel V. Llamanzares, a citizen of both the
Philippines and the U.S. The newly wed flew back to the U.S. two days after the wedding
ceremony or on 29 July 1991. She became a naturalized American citizen in 2001.

But upon the untimely demise of her adoptive father, Grace Poe and her husband decided to
move and reside permanently in the Philippines sometime in the first quarter of 2005.

On 7 July 2006, Grace Poe took her Oath of Allegiance to the Republic of the Philippines
pursuant to Republic Act (R.A.) No. 9225 or the Citizenship Retention and Re-acquisition Act
of 2003.

On 15 October 2015, petitioner filed her COC for the Presidency for the May 2016 Elections.
In her COC, the petitioner declared that she is a natural-born citizen.

A day after the Grace Poe filed her COC for President, Estrella Elamparo filed a petition to
cancel Poe’s COC before the COMELEC on the ground of material misrepresentation made by
Poe when she stated in her COC that she was a natural-born Filipino citizen and that she was
domiciled in the Philippines. Elamparo argued that petitioner cannot be considered as a
natural-born Filipino on account of the fact that she was a foundling.

And in the case docketed G.R 221698-700, where Francicso Tatad, et al. filed for certiorari
before COMELEC, seeking to disqualify Poe on the ground that she lacked requisite residency
and citizenship to qualify for Presidency. Tatad theorized that since the Philippines adheres
to jus sanguinis and that foundlings have unknown parentage, foundlings cannot be
considered natural-born Filipino citizens.

ISSUE: Whether or not Grace Poe, as a foundling, is a natural-born Filipino Citizen.

RULING: Yes. She is a natural-born Filipino citizen.


There is more than sufficient evidence that petitioner has Filipino parents and is therefore a
natural-born Filipino. Parenthetically, the burden of proof was on private respondents to
show that petitioner is not a Filipino citizen. The private respondents should have shown
that both of the petitioner's parents were aliens. Her admission that she is a foundling did
not shift the burden to her because such status did not exclude the possibility that her
parents were Filipinos, especially as in this case where there is a high probability, if not
certainty, that her parents are Filipinos.

The factual issue is not who the parents of petitioner are, as their identities are unknown, but
whether such parents are Filipinos. Under Section 4, Rule 128:

Sect. 4. Relevancy, collateral matters - Evidence must have such a relation to the fact in issue
as to induce belief in its existence or no-existence. Evidence on collateral matters shall not be
allowed, except when it tends in any reasonable degree to establish the probability of
improbability of the fact in issue.

The Solicitor General offered official statistics from the Philippine Statistics Authority (PSA)
that from 1965 to 1975, the total number of foreigners born in the Philippines was 15,986
while the total number of Filipinos born in the country was 10,558,278. The statistical
probability that any child born in the Philippines in that decade is natural-born Filipino was
99.83%. xxx

Other circumstantial evidence of the nationality of petitioner’s parents are the fact that she
was abandoned as an infant in a Roman Catholic Church in Iloilo City. She also has typical
Filipino features: height, flat nasal bridge, straight black hair, almond shaped eyes and an
oval face.

There is a disputable presumption that things have happened according to the ordinary course
of nature and the ordinary habits of life. All of the foregoing evidence, that a person with
typical Filipino features is abandoned in Catholic Church in a municipality where the
population of the Philippines is overwhelmingly Filipinos such that there would be more
than a 99% chance that a child born in the province would be a Filipino, would indicate
more than ample probability if not statistical certainty, that petitioner’s parents are
Filipinos. The Court, quotes the Solicitor General;

To deny full Filipino citizenship to all foundlings and render them stateless just because there
may be a theoretical chance that one among the thousands of these foundlings might be the
child of not just one, but two, foreigners is downright discriminatory, irrational, and unjust. It
just doesn’t make any sense. Given the statistical certainty – 99.9% – that any child born in the
Philippines would be a natural
born citizen, a decision denying foundlings such status is effectively a denial of their birthright.
There is no reason why this Honorable Court should use an improbable hypothetical to
sacrifice the fundamental political rights of an entire class of human beings.

As a matter of law, foundlings are as a class, natural-born citizens. While the 1935
Constitution’s enumeration is silent as to foundlings, there is no restrictive language which
would definitely exclude foundlings either.
REPUBLIC VS. KERRY LAO ONG
FULL TEXT: https://lawphil.net/judjuris/juri2012/jun2012/gr_175430_2012.html
YU, MARIAN ISABEL D.
TOPIC: Natural Born Citizens (Revised Naturalization Law)
FACTS: Respondent Ong filed a Petition for Naturalization in 1996 under Commonwealth
Act No. 473. He was born in Cebu to Chinese parents and is registered as a resident
alien. He was raised and educated in the Philippines, having studied in Sacred Heart
School for Boys in Cebu and later attending Ateneo de Manila University. In 1981, he married
Griselda Yap, also a Chinese Citizen and together they have 4 children all studying in the
Philippines. Ong alleged in his petition that he has been a businessman/manager since
1989 although he did not reveal the exact nature of his business. It was also stated that
Ong has a good reputation and history of taking part in Filipino practices and customs. He has
also proven that he is of sound mind and body, and has no criminal history. In 2001, the trial
court granted Ong’s petition and established the fact that
[Respondent] is a businessman/business manager engaged in lawful trade and
business since 1989 from which he derives an average annual income of more than
One Hundred Fifty Thousand Pesos (Exhibit U, V, W, and X with sub-markings)
In 2003, the Republic appealed to the CA regarding the trial court’s error in granting Ong’s
petition despite his failure to prove that he is in possession of a known lucrative trade,
profession, or lawful occupation in accordance with Section 2, Paragraph 4 of the Revised
Naturalization Law. The republic posited that his allegations regarding his occupation as a
businessman with an average annual income of ₱150,000.00 is inconsistent with the numbers
in his income tax return (1994-1997: 60k, 118k, 118k, and 128k, respectively)

ISSUE:
Whether or not respondent, Kerry Lao Ong, satisfies the qualifications stated in Section 2,
Paragraph 4 of the Revised Naturalization Law requiring the possession of lucrative trade,
profession, or legal occupation

RULING:
No, Ong did not satisfy the requirement of a lucrative trade or occupation.
Section 2, Paragraph 4 provides:
SECTION 2. Qualifications. – Subject to section four of this Act, any person having the
following qualifications may become a citizen of the Philippines by naturalization:
Fourth. He must own real estate in the Philippines worth not less than five
thousand pesos, Philippine currency, or must have some known lucrative trade,
profession, or lawful occupation;
The qualification lucrative trade, profession, or lawful occupation; means not
only does the person have enough for ordinary necessities in life but there must
be an appreciable margin in his income enough to provide adequate support in
the event of an unemployment, sickness, or disability to work and thus avoid
one’s becoming the object of charity or public charge.

The determination for an appreciable margin of income should not include spouse’s income but
in the case of Ong, his wife’s income was included by the appellate court. Ong also does not
own property and his reported average annual income of more than ₱150,000.00 is
inconsistent with the numbers in his income tax return. Considering he has 4 minor children all
studying in private schools, it is questionable whether Ong’s income had an appreciable margin
over his expenses. In addition, Ong did not provide further evidence regarding the exact nature
of his business despite his alleged occupation as businessman/business manager since 1989.

GO VS REPUBLIC
FULL TEXT: https://www.chanrobles.com/cralaw/2014julydecisions.php?id=397
Jan Joy Zamora
TOPIC: Naturalization (Disqualification)
FACTS: On October 13, 2004, petitioner filed a petition for naturalization under
Commonwealth Act (C.A.) No. 473, the Revised Naturalization Law with the RTC.
Petitioner testified to prove his compliance with all the requirements for naturalization
and presented witnesses.

After the petitioner presented his evidence and formally offered the same, the
Republic, through the OSG, posed no objection as to the relevancy and competence
of his documentary evidence.

The OSG, however, later moved for the reopening of trial for the admission of its
documentary evidence. It informed RTC that it had received a report dated November
23, 2006 issued by the National Bureau of Investigation (NBI), tending to prove
petitioner’s non-compliance with the requirements of the law on naturalization.
However, the petitioner manifested that he obtained a clearance and that he was not
the same Dennis Go, subject of the NBI investigation Report.

RTC granted the petition for naturalization, ruling that he possessed the qualifications
set forth by law. OSG moved for reconsideration and reopened the trial wherein they
admit as evidence the BOI investigation report to oppose the petition. RTC denied
OSG for lack of merit.

OSG appealed to CA. CA reversed and set aside the RTC decision and dismissed,
without prejudice, the petition for naturalization. Hence, this petition.

Issue: WON the petition for naturalization should be granted?

Ruling: No.

Jurisprudence dictates that in judicial naturalization, the application must show


substantial and formal compliance with C.A. No. 473. In other words, an applicant
must comply with the jurisdictional requirements,establish his or her possession of the
qualifications and none of the disqualifications enumerated under the law, and present
at least two (2) character witnesses to support his allegations. In Ong v. Republic of
the Philippines, the Court listed the requirements for character witnesses,
namely:

1. That they are citizens of the Philippines;


2. That they are “credible persons”;
3. That they personally know the petitioner;
4. That they personally know him to be a resident of the Philippines for the
period of time required by law;
5. That they personally know him to be a person of good repute;
6. That they personally know him to be morally irreproachable;
7. That he has, in their opinion, all the qualifications necessary to become a
citizen of the Philippines; and
8. That he “is not in any way disqualified under the provisions” of the
Naturalization Law”

The records of the case show that the joint affidavits executed by Go’s
witnesses did not establish their own qualification to stand as such in a
naturalization proceeding. In turn, Go’s did not present evidence proving that
the persons he presented were credible. In the words of the CA, “he did not
prove that his witnesses had good standing in the community, known to be
honest and upright, reputed to be trustworthy and reliable, and that their word
may be taken at face value, as a good warranty of the worthiness of Go.”

While there is no showing that Go’s witnesses were of doubtful moral inclinations,
there was likewise no indication that they were persons whose qualifications were at
par with the requirements of the law on naturalization. Simply put, no evidence was
ever proffered to prove the witnesses’ good standing in the community, honesty, moral
uprightness, and most importantly, reliability.

As a consequence, their statements about Go do not possess the measure of


credibility demanded of in naturalization cases. This lack of credibility on the
part of the witnesses, unfortunately, weakens or renders futile Go’s claim of
worthiness. An applicant for Philippine citizenship would carefully testify as to
his qualifications, placing emphasis on his good traits and character. This is
expected of a person who longs to gain benefits and advantages of Philippine
citizenship bestows. Therefore, a serious assessment of an applicant’s
witnesses, both as to the credibility of their person and their very testimony, is
an essential facet of naturalization proceedings that may not be brushed aside.

Furthermore, the background checks done on petitioner yielded negative results due
to the uncooperative behavior of the members of his household. In fact,petitioner
himself disobliged when asked for an interview by BOI agents. To the Court, this is a
display of insincerity to embrace Filipino customs, traditions and ideals.

Finally, it is noteworthy that petitioner’s failure to state his former residence in


the petition was fatal to his application for naturalization. Indeed, this omission
had deprived the trial court of jurisdiction to hear and decide the case.

WHEREFORE, the petition is DENIED. The January 18, 2012 Decision and the July
23, 2012 Resolution of the Court of Appeals in CAG. R. CV No. 95120 are
AFFIRMED. As stated in the decision of the Court of Appeals, the dismissal is without
prejudice.
REPUBLIC VS. AZUCENA
Full Text:https://lawphil.net/judjuris/juri2013/oct2013/gr_183110_2013.html
Digested by: Jannah Ajoc

TOPIC: Effect of Naturalization

FACTS: On December 2, 2002, Azucena filed a Petition for Naturalization before the
RTC of Zamboanga del Sur. Azucena alleged in her Petition that she believes in the
principles underlying the Philippine Constitution. She also stated that she intends in
good faith to become a citizen of the Philippines and to renounce absolutely and
forever all allegiance and fidelity to any foreign prince, potentate, state or sovereignty,
and particularly to China; and that she will reside continuously in the Philippines from
the time of the filing of her Petition up to the time of her naturalization. After all the
jurisdictional requirements mandated and had been complied with, the Office of the
Solicitor General filed its Motion to Dismiss on the ground that Azucena failed to allege
that she is engaged in a lawful occupation or in some known lucrative trade.
Thereafter, the hearing for the reception of Azucena’s evidence was then set on May
18, 2004.

ISSUE: Whether or not petitioner has validly complied with the citizenship requirement
as required by law to become a naturalized citizen of the Philippines.

RULING: Yes. Under existing laws, an alien may acquire Philippine citizenship
through either judicial naturalization under CA 473 or administrative naturalization
under Republic Act No. 9139 (the “Administrative Naturalization Law of 2000”). A third
option, called derivative naturalization, which is available to alien women married to
Filipino husbands is found under Section 15 of CA 473.

“Any woman who is now or may hereafter be married to a citizen of the


Philippines and who might herself be lawfully naturalized shall be deemed a
citizen of the Philippines.”

Under this provision, foreign women who are married to Philippine citizens may be
deemed ipso facto Philippine citizens and it is neither necessary for them to prove that
they possess other qualifications for naturalization at the time of their marriage nor do
they have to submit themselves to judicial naturalization. It is, therefore, not congruent
with our cherished traditions of family unity and identity that a husband should be a
citizen and the wife an alien, and that the national treatment of one should be different
from that of the other.

WHEREFORE, the Petition is DENIED. The Decision of the Court of which affirmed
the Decision of the Regional Trial Court, that granted the Petition for Naturalization, is
hereby AFFIRMED. Subject to compliance with the period and the requirements under
Republic Act No. 530 which supplements the Revised Naturalization Law, let a
Certificate of Naturalization be issued to AZUCENA SAAVEDRA BATUIGAS after
taking an oath of allegiance to the Republic of the Philippines. Thereafter, her Alien
Certificate of Registration should be cancelled.

REPUBLIC VS. LI CHING CHUNG


FULL TEXT: https://lawphil.net/judjuris/juri2013/mar2013/gr_197450_2013.html
Digested by: Jadz Felix Alicer
TOPIC: DENATURALIZATION Com. Act No. 473, Sec. 18
FACTS:

Respondent came to the Philippines on March 15, 1988 via Philippine Airlines

On August 22, 2007, respondent, otherwise known as Bernabe Luna Li or Stephen


Lee Keng, a Chinese national, filed his Declaration of Intention to Become a Citizen of
the Philippines before the OSG

That he was entitled to the benefit of Section 3 of Commonwealth Act (CA) No. 473
reducing to five (5) years the requirement under Section 2 of ten years of continuous
residence, because he knew English and Filipino having obtained his education from
St. Stephen’s High School of Manila; and that he had successfully established a
trading general merchandise business operating under the name of "VS Marketing
Corporation."

On April 5, 2008, respondent filed his Amended Petition for Naturalization

Respondent filed the Motion for Early Setting praying that the hearing be moved from
April 3, 2009 to July 31, 2008 so he could acquire real estate properties.

On June 3, 2009, the RTC granted respondent’s application for naturalization as a


Filipino citizen.

To bolster its claim for the reversal of the assailed ruling, the OSG advances this
pivotal issue of

x x x whether the respondent should be admitted as a Filipino citizen despite his


undisputed failure to comply with the requirements provided for in CA No. 473.

ISSUE: : Whether the respondent should be admitted as a Filipino citizen despite his
undisputed failure to comply with the requirements provided for in CA No. 473.

RULING:
NO. In naturalization proceedings, the burden of proof is upon the applicant to show
full and complete compliance with the requirements of the law. The opportunity of a
foreigner to become a citizen by naturalization is a mere matter of grace, favor or
privilege extended to him by the State; the applicant does not possess any natural,
inherent, existing or vested right to be admitted to Philippine citizenship. The only right
that a foreigner has, to be given the chance to become a Filipino citizen, is that which
the statute confers upon him; and to acquire such right, he must strictly comply with all
the statutory conditions and requirements. The absence of one jurisdictional
requirement is fatal to the petition as this necessarily results in the dismissal or
severance of the naturalization process.

Ultimately, respondent failed to prove full and complete compliance with the
requirements of the Naturalization Law. As such, his petition for naturalization must be
denied without prejudice to his right to re-file his application.

WHEREFORE, the petition is GRANTED. The June 30, 2011 Decision of the Court of
Appeals in CA-G.R. CV No. 93374 is REVERSED and SET ASIDE.
AASJS VS. DATUMANONG
FULL TEXT: https://www.lawphil.net/judjuris/juri2007/may2007/gr_160869_2007.html
RALF VINCENT BAJO

TOPIC: Loss of and re-acquisition of Citizenship; RA 9225

FACTS:Petitioner prays that a writ of prohibition be issued to stop respondent


(Secretary of Justice Simeon Datumanong, the official tasked to implement laws governing citizenship)
from implementing Republic Act No. 9225, otherwise known as the Citizen Retention
and Re-acquisition Act of 2003. Under the said law, natural-born citizens of the
Philippines who become citizens of a foreign country shall retain or re-acquire their
Philippine citizenship upon taking the oath of allegiance to the Republic of the
Philippines as outlined in Section 3. Petitioner avers that RA 9225 is unconstitutional
as it violates Section 5, Article IV of the 1987 Constitution that states, "Dual allegiance
of citizens is inimical to the national interest and shall be dealt with by law."

ISSUE: Whether or not RA 9225 is unconstitutional.

RULING: No. RA 9225 is NOT UNCONSTITUTIONAL.

From the excerpts of the legislative record, it is clear that the intent of the legislature in
drafting RA 9225 is to do away with the provision in Commonwealth Act No. 635 which
takes away Philippine citizenship from natural-born Filipinos who become naturalized
citizens of other countries. What RA 9225 does is allow dual citizenship to
natural-born Filipino citizens who have lost Philippine citizenship by reason of their
naturalization as citizens of a foreign country. On its face, it does not recognize dual
allegiance. By swearing to the supreme authority of the Republic, the person implicitly
renounces his foreign citizenship. Plainly, from Section 3, RA 9225 stayed clear out of
the problem of dual allegiance and shifted the burden of confronting the issue of
whether or not there is dual allegiance to the concerned foreign country. What
happens to the other citizenship was not made a concern of RA 9225.

To begin with, Section 5, Article IV of the Constitution is a declaration of a policy and it


is not a self-executing provision. The legislature still has to enact the law on dual
allegiance. In Sections 2 and 3 of RA No. 9225, the framers were not concerned with
dual citizenship per se, but with the status of naturalized citizens who maintain their
allegiance to their countries of origin even after their naturalization.9 Congress was
given a mandate to draft a law that would set specific parameters of what really
constitutes dual allegiance.10 Until this is done, it would be premature for the judicial
department, including this Court, to rule on issues pertaining to dual allegiance.

NOTES: Mercado v. Manzano differentiates dual citizenship from dual allegiance:


Dual citizenship “arises when, as a result of the concurrent application of the different laws of two or more states, a
person is simultaneously considered a national by the said states. For instance, such a situation may arise when a
person whose parents are citizens of a state which adheres to the principle of jus sanguinis is born in a state which
follows the doctrine of jus soli. Such a person, ipso facto and without any voluntary act on his part, is concurrently
considered a citizen of both states.”

“Dual allegiance, on the other hand, refers to the situation in which a person simultaneously owes, by some positive
act, loyalty to two or more states. While dual citizenship is involuntary, dual allegiance is the result of an individual's
volition.”

LOPEZ VS. COMELEC


FULL TEXT: https://lawphil.net/judjuris/juri2008/jul2008/gr_182701_2008.html
KRISTINA DANICA BALANAY

TOPIC: Loss and Re-Acquisition of Citizenship

FACTS: Petitioner Lopez, a dual citizen, was a candidate for the position of Chairman of
Barangay Bagacay, San Dionisio, Iloilo City held on October 29, 2007. He was eventually
declared the winner. On October 25, 2007, respondent Villanueva filed a petition before the
Provincial Election Supervisor of the Province of Iloilo, praying for the disqualification of Lopez
because he was ineligible from running for any public office. Lopez argued that he is a
Filipino-American, by virtue of the Citizenship Retention and Re-acquisition Act of 2003. He
said, he possessed all the qualifications to run for Barangay Chairman.

On February 6, 2008, COMELEC issued the Resolution granting the petition for disqualification
of Lopez from running as Barangay Chairman. COMELEC said, to be able to qualify as a
candidate in the elections, Lopez should have made a personal and sworn renunciation of any
and all foreign citizenship.

His motion for reconsideration having been denied, Lopez resorted to petition for certiorari,
imputing grave abuse of discretion on the part of the COMELEC for disqualifying him from
running and assuming the office of Barangay Chairman.

ISSUE: Whether or not there was grave abuse of discretion on the part of the COMELEC for
disqualifying petitioner.

RULING:
No. The Supreme Court dismissed the petition. The COMELEC committed no grave abuse of
discretion in disqualifying the petitioner as candidate for Chairman in the Barangay elections of
2007. Lopez was born a Filipino but he deliberately sought American citizenship and
renounced his Filipino citizenship. He later on became a dual citizen by re-acquiring Filipino
citizenship.

R.A. No. 9225 expressly provides for the conditions before those who re-acquired Filipino
citizenship may run for a public office in the Philippines. Section 5 of the said law states:

Section 5. Civil and Political Rights and Liabilities. – Those who retain or re-acquire Philippine
citizenship under this Act shall enjoy full civil and political rights and be subject to all attendant
liabilities and responsibilities under existing laws of the Philippines and the following conditions:

(2) Those seeking elective public office in the Philippines shall meet the qualification for holding
such public office as required by the Constitution and existing laws and, at the time of the filing
of the certificate of candidacy, make a personal and sworn renunciation of any and all foreign
citizenship before any public officer authorized to administer an oath.

Lopez was able to regain his Filipino Citizenship by virtue of the Dual Citizenship Law when he
took his oath of allegiance before the Vice Consul of the Philippine Consulate General’s Office
in Los Angeles, California; the same is not enough to allow him to run for a public office.
Lopez’s failure to renounce his American citizenship as proven by the absence of an
affidavit that will prove the contrary leads this Commission to believe that he failed to
comply with the positive mandate of law.
JACOT VS. DAL
FULL TEXT: https://lawphil.net/judjuris/juri2008/nov2008/gr_179848_2008.html
Sophia Amor C. Bersamin
TOPIC: Loss of and re-acquisition of Citizenship

Facts:

Petitioner Nestor A. Jacot seeks to assail the Resolution dated 28 September 2007 of the Commission
on Elections (COMELEC) En Banc in SPA No. 07-361, affirming the Resolution dated 12 June 2007 of
the COMELEC Second Division2 disqualifying him from running for the position of Vice-Mayor of
Catarman, Camiguin, in the 14 May 2007 National and Local Elections, on the ground that he failed to
make a personal renouncement of his United States (US) citizenship.

Petitioner was a natural-born citizen of the Philippines, who became a naturalized citizen of the US on
December 13, 1989. He sought to reacquire his Philippine citizenship under Republic Act No. 9225.
Petitioner took his Oath of Allegiance to the Republic of the Philippines before Vice Consul Edward C.
Yulo. The Bureau of Immigration issued Identification Certificate No. 06-12019 recognizing petitioner as a
citizen of the Philippines. Six months after, on March 26, 2007, petitioner filed his Certificate of Candidacy
for the Position of Vice-Mayor of the Municipality of Catarman, Camiguin. Respondent Rogen T. Dal filed
a Petition for Disqualification before the COMELEC Provincial Office in Camiguin against the petitioner,
arguing that the latter failed to renounce his US citizenship, as required under Section 5(2) of Republic
Act No. 9225. Petitioner garnered the highest number of votes for the position of Vice Mayor in the May
14, 2007 National and Local Elections

Issue:

W/N petitioner’s Oath of Allegiance to the Republic of the Philippines and his Certificate of Candidacy
substantially comply with the requirement of a personal and sworn renunciation of foreign citizenship.

Ruling:

NO. Contrary to the assertions made by petitioner, his oath of allegiance to the Republic of the Philippines
made before the Los Angeles PCG and his Certificate of Candidacy do not substantially comply with the
requirement of a personal and sworn renunciation of foreign citizenship because these are distinct
requirements to be complied with for different purposes.

Section 5(2) of Republic Act No. 9225 compels natural-born Filipinos, who have been naturalized as
citizens of a foreign country, but who reacquired or retained their Philippine citizenship (1) to take the oath
of allegiance under Section 3 of Republic Act No. 9225, and (2) for those seeking elective public offices in
the Philippines, to additionally execute a personal and sworn renunciation of any and all foreign
citizenship before an authorized public officer prior or simultaneous to the filing of their certificates of
candidacy, to qualify as candidates in Philippine elections.

In the case at bar, the oath of allegiance contained in the Certificate of Candidacy, which is substantially
similar to the one contained in Section 3 of Republic Act No. 9225, does not constitute the personal and
sworn renunciation sought under Section 5(2) of Republic Act No. 9225. It bears to emphasize that the
said oath of allegiance is a general requirement for all those who wish to run as candidates in Philippine
elections; while the renunciation of foreign citizenship is an additional requisite only for those who have
retained or reacquired Philippine citizenship under Republic Act No. 9225 and who seek elective public
posts, considering their special circumstance of having more than one citizenship.

Court ruled to dismiss the instant petition and affirmed the COMELEC resolution.
DE GUZMAN VS. COMELEC
FULL TEXT: https://lawphil.net/judjuris/juri2009/jun2009/gr_180048_2009.html
Hanna Lyka Bontilao

TOPIC – Loss of and re-acquisition of Citizenship

FACTS – Petitioner De Guzman and private respondent Angelina DG. Dela Cruz were
candidates for vice-mayor of Guimba, Nueva Ecija in the May 14, 2007 elections.
Private respondent filed a petition for disqualification against De Guzman alleging that
petitioner is not a citizen of the Philippines, but an immigrant and resident of the
United States of America.

Petitioner admitted that he was a naturalized American. However, in 2006, he applied


for dual citizenship under Republic Act No. 9225, otherwise known as the Citizenship
Retention and Re-Acquisition Act of 2003. He took his oath of allegiance to the
Republic of the Philippines and re-acquired his Philippine citizenship. He, therefore,
claims he is entitled to exercise full civil and political rights and is qualified to run as
vice-mayor of Guimba, Nueva Ecija.

Petitioner invokes the rulings in Frivaldo v. Commission on Election and Mercado v.


Manzano, that the filing by a person with dual citizenship of a certificate of candidacy,
containing an oath of allegiance, constituted as a renunciation of his foreign
citizenship.

ISSUE – Whether or not De Guzman is disqualified from running for public office for
having failed to renounce his American citizenship.

RULING – Yes. R.A. 9225 allows re-acquisition and retention of Philippine citizenship
for natural-born citizens who have lost their Philippine citizenship by reason of their
naturalization as citizens of a foreign country, as is the case with De Guzman. The law
provides that they are deemed to have re-acquired or retained their Philippine
citizenship upon taking the oath of allegiance. However, under Section 5(2) of R.A.
9225, the law imposes an additional requirement on those who wish to seek elective
public office, which is, at the time of the filing of the certificate of candidacy, to make a
personal and sworn renunciation of any and all foreign citizenship.

Contrary to petitioner’s claims, the filing of a certificate of candidacy does not ipso
facto amount to a renunciation of his foreign citizenship under R.A. No. 9225. The
Court’s rulings in the cases of Frivaldo and Mercado are not applicable to the instant
case because R.A. No. 9225 provides for more requirements.

Further, in Jacot v. Dal and COMELEC, the Court ruled that a candidate’s oath of
allegiance to the Republic of the Philippines and his Certificate of Candidacy do not
substantially comply with the requirement of a personal and sworn renunciation of
foreign citizenship sought under Section 5(2) of Republic Act No. 9225.
Hence, petitioner failed to renounce his American citizenship; as such, he is
disqualified from running for vice-mayor of Guimba, Nueva Ecija in the May 14, 2007
elections.
SOBEJANA-CONDON VS. COMELEC
FULL TEXT: https://lawphil.net/judjuris/juri2012/aug2012/gr_198742_2012.html
Kyle Ferdausi G. Cerna
Topic: On the Loss of and Renunciation of Citizenship

FACTS:
The petitioner is a natural-born Filipino citizen, born from Filipino parents. She later on
became a naturalized Australian citizen by marrying a certain Kevin Condon. When
she applied to re-acquire her Filipino citizenship, it was granted and took her oath of
allegiance as a Filipino citizen. In 2005 she renounced her Australian citizenship
through an unsworn declaration. When she ran for public office and won, her defeated
opponent contested in the COMELEC her eligibility to run for public office, on the
grounds of being a dual citizen. Petitioner claims that her renunciation of being an
Australian citizen was duly executed, and that her act of running for public office is a
clear declaration of her renunciation. Hence, the issue at hand.

ISSUE:
Whether or not the petitioner should be disqualified from running for public office.

Ruling: Yes.

According to the SC, the petitioner is disqualified from running for an elective office for
failure to renounce her Australian citizenship in accordance with Section 5, par.2 of RA
9225 (Citizenship and Re-acquisition Act of 2003).

Under the provisions of such law, the petitioner has validly re-acquired her Filipino
citizenship when she took an Oath of Allegiance to the Republic of the Philippines in
2005. A year before she ran for public office, she renounced her being an Australian
citizen, but unfortunately such was not under oath administered by proper authority,
and in contrary to the mandate stipulated by the said law.

RA 9225 states that the renunciation of foreign citizenship must be sworn before an
officer authorized to administer the oath. While the petitioner may have sworn
allegiance to the Republic of the Philippines, such was not sufficient for her to run for
public office, provided that she was not able to take her oath renouncing her
Australian citizenship before a proper authority. The high court also explained that In
fine, R.A. No. 9225 categorically demands natural-born Filipinos who re-acquire their
citizenship and seek elective office, to execute a personal and sworn renunciation of
any and all foreign citizenships before an authorized public officer prior to or
simultaneous to the filing of their certificates of candidacy, to qualify as candidates in
Philippine elections.

The SC added that since the said law does not give rise to any ambiguity, it must only
be interpreted in its plain meaning – and not to trace back from the intent of the law.
The same was also emphasized in the case of Lopez vs. COMELEC, when it declared
that, its categorical and single meaning: a Filipino American or any dual citizen cannot
run for any elective public position in the Philippines unless he or she personally
swears to a renunciation of all foreign citizenship at the time of filing the certificate of
candidacy. It also expounded on the form of the renunciation and held that to be valid,
the renunciation must be contained in an affidavit duly executed before an officer of
the law who is authorized to administer an oath stating in clear and unequivocal terms
that affiant is renouncing all foreign citizenship.
GUY VS. IGNACIO
FULL TEXT: https://lawphil.net/judjuris/juri2010/jul2010/gr_167824_2010.html
CRUZ, SOPHIA DENISSE L.
TOPIC: Loss of Citizenship

FACTS: The father of petitioners Geraldine Gaw Guy and Grace Guy Cheu became a
naturalized Filipino citizen sometime in 1959. The said petitioners, being minors at that
time, were also recognized as Filipino citizens.

Respondent Atty. Alvin Agustin T. Ignacio, filed a Complaint dated March 5, 2004 for
blacklisting and deportation against petitioners Geraldine and Grace before the
Bureau of Immigration (BI) on the basis that the latter two are Canadian citizens who
are illegally working in the Philippines, petitioners having been issued Canadian
passports.

ISSUE: WON immediate judicial intervention in deportation proceedings is allowed


where the claim of citizenship is so substantial that there are reasonable grounds to
believe that the claim is correct.

RULING: YES
Petitioners rely on Board of Commissioners (CID) v. Dela Rosa, wherein this Court
ruled that when the claim of citizenship is so substantial as to reasonably believe it to
be true, a respondent in a deportation proceeding can seek judicial relief to enjoin
respondent BOC from proceeding with the deportation case.

Thus, what if the claim to citizenship of the alleged deportee is satisfactory? Should
the deportation proceedings be allowed to continue or should the question of
citizenship be ventilated in a judicial proceeding? In Chua Hiong vs. Deportation Board
(96 Phil. 665 [1955]), this Court answered the question in the affirmative, and We
quote:

When the evidence submitted by a respondent is conclusive of his citizenship, the


right to immediate review should also be recognized and the courts should promptly
enjoin the deportation proceedings. A citizen is entitled to live in peace, without
molestation from any official or authority, and if he is disturbed by a deportation
proceeding, he has the unquestionable right to resort to the courts for his protection,
either by a writ of habeas corpus or of prohibition, on the legal ground that the Board
lacks jurisdiction. If he is a citizen and evidence thereof is satisfactory, there is no
sense nor justice in allowing the deportation proceedings to continue, granting him the
remedy only after the Board has finished its investigation of his undesirability.

Judicial intervention, however, should be granted in cases where the claim of


citizenship is so substantial that there are reasonable grounds to believe that the claim
is correct. In other words, the remedy should be allowed only on sound discretion of a
competent court in a proper proceeding (Chua Hiong v. Deportation Board, supra; Co
vs. Deportation Board, 78 SCRA 107 [1977]). It appearing from the records that
respondent's claim of citizenship is substantial, as We shall show later, judicial
intervention should be allowed.
YU VS. DEFENSOR-SANTIAGO
G.R. No. L-83882
Aimee E. Demontano

TOPIC:
Loss of citizenship by express renunciation of citizenship or expatriation

FACTS:
Yu was issued a Portuguese passport in 1971, valid for five years and renewed for the
same period upon presentment before the proper Portuguese consular officer.

Despite his naturalization as a Philippine citizen in 1978, he applied for and was
issued a Portuguese passport in 1981.

CID Board of Commissioners filed deportation against Yu on 1998.

Respondent Commissioner filed a motion to lift Temporary Restraining Order (TRO)


and give non-extendible period of 3 days from notice within which to explain and prove
why he should still be considered a citizen of the Philippines despite his acquisition
and use of a Portuguese passport.

The petitioner failed to show proof.

While still a citizen of the Philippines who had renounced, upon his naturalization,
"absolutely and forever all allegiance and fidelity to any foreign prince, potentate, state
or sovereignty" and pledged to "maintain true faith and allegiance to the Philippines,"
he declared his nationality as Portuguese in commercial documents he signed.

ISSUE:
Whether or not petitioner's claim to continued Philippine citizenship is meritorious

RULING: NO
To the mind of the court, the foregoing acts considered together constitute an express
renunciation of petitioner's Philippine citizenship acquired through naturalization. In
Board of Immigration Commissioner vs. Go Gallano, express renunciation was held to
mean a renunciation that is made known distinctly and explicitly and not left to
inference or implication.

Yu, with full knowledge and legal capacity, after having renounced Portuguese
citizenship upon naturalization as a Philippine citizen resumed or reacquired his prior
status as a Portuguese citizen, applied for a renewal of his Portuguese passport and
represented himself as such in official documents even after he had become a
naturalized Philippine citizen. Such resumption or reacquisition of Portuguese
citizenship is grossly inconsistent with his maintenance of Philippine citizenship.

Philippine citizenship, it must be stressed, is not a commodity or were to be displayed


when required and suppressed when convenient.
Jose B. Aznar vs. Emilio Mario Renner Osmeña
FULL TEXT: https://www.chanrobles.com/cralaw/1990maydecisions.php?id=528

Digested by: Rynn Judd C. Escaño

Topic: Loss of Citizenship

Facts:

On November 18, 1987, Emilio Osmeña filed his certificate of candidacy with the
COMELEC for the position of Provincial Governor of Cebu in the January 18,
1988 elections. On January 22, 1988, Jose B. Aznar in his capacity as the incumbent
Cebu PDP – Laban Provincial Council Chairman filed to the COMELEC a petition
for disqualification against the respondent on the ground that he is not a Filipino
citizen, being a citizen of the United States.

In the proceeding before the COMELEC, the petitioner presented Application for Alien
Registration Form NO. 1 of the Bureau of Immigration signed by the respondent dated
November 21, 1979; Alien Certificate of Registration No. 015356 in the name of
private respondent dated November 21, 1979; Permit to Re-enter the Philippines dated
November 21, 1979 (Exh. "D"); Immigration Certificate of Clearance dated January 3,
1980.

The respondent on the other hand argued that he is a Filipino citizen for being the
legitimate child of Dr. Emilio D. Osmeña, who is a Filipino and a son of the Late
President Sergio Osmeña Sr.; that he is a holder of a valid Philippine passport. He
also argued that he has been continuously residing in the Philippines since birth and
has not gone out of the country for more than six months; and that he has been a
registered voter in the Philippines since 1965.

On June 11, 1988, COMELEC (First Division) dismissed the petition for
disqualification for not having been timely filed and for lack of sufficient proof that
private respondent is not a Filipino citizen

Issue:

W/N Emilio Mario Renner Osmeña is a Filipino citizen.

Ruling:

YES. Being an American citizen does automatically disqualify a Filipino his or her
Filipino citizenship. According to Commonwealth Act No. 63, a Filipino may lose his
citizenship (a) by naturalization to a foreign country, (b) by express renunciation of
citizenship, (c) by subscribing an oath of allegiance to support the Constitution or laws
of a foreign country.
With the evidence presented by petitioner are not substantial to prove that the
respondent lost his citizenship due to the reason stated, the petition was dismissed.

VILANDO VS. HRET


FULL TEXT: https://lawphil.net/judjuris/juri2011/aug2011/gr_192147_2011.html
RYAN JUANITO GARCIA

FACTS:
In the May 2017 elections Jocelyn Limkaichong ran as a representative in the 1st
District of Negros Oriental. She won over the other contender, Olivia Paras. Petitions involving
the disqualification of Limkaichong and questions about her citizenship were filed. They
alleged that Limkaichong was not a natural born citizen of the Philippines because when she
was born her father was still a Chinese and that her mother lost her Filipino citizenship by
virtue of her marriage to Limkaichong’s father.

ISSUES:
(1) Whether or not the House of Representatives Electoral Tribunal already acquired
jurisdiction over the case; and ,

(2) Whether or not Limkaichong is qualified to hold an office in the Republic of the
Philippines.

RULING:
1. No. The House of Representatives Electoral Tribunal has no authority to delve into
the legality of the judgment of naturalization. To rule otherwise would operate as a collateral
attack on the citizenship of Limkalchong’s father which is not permissible. In our jurisdiction,
an attack on a person’s citizenship may only be done through a direct action for its nullity.

2. Yes. Records disclose that Limkaichong was born in Dumaguete City on November
9, 1959. The governing law is the citizenship provision of the 1935 Constitution. The HRET,
therefore, correctly relied on the presumption of validity of the July 9, 1957 and September
21, 1959 Orders of the Court of First Instance (CFI) Negros Oriental, which granted the petition
and declared Julio Sy a naturalized Filipino absent any evidence to the contrary. Respondent
Limkaichong falls under the category of those persons whose fathers are citizens of the
Philippines. (Section 1(3), Article IV, 1935 Constitution) It matters not whether the father
acquired citizenship by birth or by naturalization. Therefore, following the line of transmission
through the father under the 1935 Constitution, the respondent has satisfactorily complied
with the requirement for candidacy and for holding office, as she is a natural-born Filipino
citizen.

Respondent participated in the barangay elections as a young voter in 1976,


accomplished voter's affidavit as of 1984, and ran as a candidate and was elected as Mayor of
La Libertad, Negros Oriental in 2004. These are positive acts of election of Philippine
citizenship.
The case of In re: Florencio Mallare, elucidates how election of citizenship is
manifested in actions indubitably showing a definite choice. It was noted that respondents
had informally elected citizenship after January 17, 1973 during which time the 1973
Constitution considered as citizens of the Philippines all those who elect citizenship in
accordance with the 1935 Constitution.
PEOPLE VS. MANAYAO
FULL TEXT: https://lawphil.net/judjuris/juri1947/jul1947/gr_l-322_1947.html
Digested by Jannah Ajoc

TOPIC: Loss of Citizenship

FACTS: On January 29, 1945, Japanese soldiers and a number of Filipinos affiliated
with the Makapili, including the appellant, gathered the residents of barrio Banaban of
Angat, Province of Bulacan. In accordance with their plan, they killed the residents
excluding the children. The appellant alone killed about six women and would have
also killed the children if he had been allowed to. The appellant was then convicted
with the high crime of treason with multiple murder in the People’s Court. In his
appeal, his counsel contends that he is a member of the Armed Forces of Japan
and is therefore not subject to the jurisdiction of the People’s Court. He also
contends that the appellant has already lost his Filipino citizenship thus he is not
amenable to the Philippine law of treason.

ISSUE: Whether or not the appellant has lost his citizenship

RULING: No. The appellant did not lose his Filipino citizenship.

Repatriation results in the recovery of the original nationality. If a person was originally a
natural-born citizen before he lost his Philippine citizenship, he will be restored to his former
status as a natural-born Filipino.

In respondent Cruz's case, he lost his Filipino citizenship when he rendered service in the
Armed Forces of the United States. However, he subsequently reacquired Philippine
citizenship under R.A. No. 2630, which provides: Section 1. Any person who had lost his
Philippine citizenship by rendering service to, or accepting commission in, the Armed Forces of
the United States, or after separation from the Armed Forces of the United States, acquired
United States citizenship, may reacquire Philippine citizenship by taking an oath of allegiance
to the Republic of the Philippines and registering the same with Local Civil Registry in the place
where he resides or last resided in the Philippines. The said oath of allegiance shall contain a
renunciation of any other citizenship.

Having thus taken the required oath of allegiance to the Republic and having registered the
same in the Civil Registry of Magantarem, Pangasinan, respondent Cruz HAS RECOVERED his
original status as a natural-born citizen.
ANGAT VS. REPUBLIC
FULL TEXT: https://lawphil.net/judjuris/juri1999/sep1999/gr_132244_1999.html
Digested by : Darriel Kier Loreque

Topic : Repatriation, naturalization, governing law at the time petition is filed

Facts: On March 11, 1996 Gerardo Angat filed a petition with RTC Marikina City to be
Re-admitted as a Citizen of the Philippines under Commonwealth Act No. 63, as amended, and
R.A. 965 and 263. Angat was a natural born Filipino until he lost his citizenship by
naturalization in the United States of America, and now presently residing in Marikina.
After hearing, Angat was allowed by RTC to take his oath of allegiance to the Republic of the
Philippines pursuant to R.A. 8171. OSG opposed the petition by Manifestation and Motion that
the court a quo (RTC) ought to dismiss it for lack of jurisdiction because the proper forum for
it was the Special Committee on Naturalization consistently with Administrative Order No.
285 (AO 285), dated 22 August 1996, issued by President Fidel V. Ramos. AO 285 had tasked
the Special Committee on Naturalization to be the implementing agency of R.A 8171.

Pursuant to OSG’s motion, judge dismissed Angat’s petition. Now this present review
before SC, Angat asserts that the trial court had jurisdiction over his petition for naturalization
note: although designated as a petition for naturalization filed before RTC, it was really a
petition for repatriation filed on 11 March 1996, or months before the Special Committee on
Naturalization was constituted by the President under AO 285 on 22 August 1996, RTC thus
had the authority to take cognizance of the case.

Issue: What law should govern, AO 285 or RA 8171?

Held: AO 285

R.A. No. 8171, which has lapsed into law on 23 October 1995 is an act providing for
the repatriation (a) of Filipino women who have lost their Philippine citizenship by marriage to
aliens and (b) of natural-born Filipinos who have lost their Philippine citizenship on account or
political or economic necessity. The pertinent provisions of the law read:

Sec. 1. Filipino women who have lost their Philippine citizenship by marriage to aliens
and natural-born Filipinos who have lost their Philippine citizenship, including their minor
children, on account of political or economic necessity, may reacquire Philippine citizenship
through repatriation in the manner provided in Section 4 of Commonwealth Act No. 631, as
amended: Provided,that the applicant is not a:

(1) Person opposed to organized government or affiliated with any association or group of
persons
who uphold and teach doctrines opposing organized government;
(2) Person defending or teaching the necessity or propriety of violence, personal assault, or
association for the predominance of their ideas;
(3) Person convicted of crimes involving moral turpitude: or
(4) Person suffering from mental alienation or incurable contagious diseases.

Sec. 2. Repatriation shall be effected by taking the necessary oath of allegiance to the Republic
of
the Philippines and registration in the proper civil registry and in the Bureau of Immigration.
The
Bureau of Immigration shall thereupon cancel the pertinent alien certificate of registration and
issue the certificate of identification as Filipino citizen to the repatriated citizen.

Under Section 1 of Presidential Decree ("P.D.") No. 725, dated 05 June 1975,
amending Commonwealth Act No. 63, an application for repatriation could be filed by Filipino
women who lost their Philippine citizenship by marriage to aliens, as well as by natural born
Filipinos who lost their Philippine citizenship, with the Special Committee on Naturalization.
The committee, chaired by the Solicitor General with the Undersecretary of Foreign Affairs
and the Director of the National Intelligence Coordinating Agency as the other members, was
created pursuant to Letter of. Instruction ("LOI") No. 270, dated 11 April 1975, as amended
by LOI No. 283 and LOI No. 491 issued, respectively, on 04 June 1975 and on 29 December
1976.

Although the agency was deactivated by virtue of President Corazon C. Aquino's


Memorandum of 27 March 1987, it was not however, abrogated. In Frivaldo vs. Commission
on Elections, the Court observed that the afore dated memorandum of President Aquino had
merely directed the Special Committee on Naturalization "to cease and desist from undertaking
any and all proceedings under Letter of Instruction ("LOI") 270." Laws are repealed only by
subsequent ones and a repeal may be express or implied. It is obvious that no express repeal
was made because then President Aquino in her memorandum-based on the copy furnished us
by Lee-did not categorically and/or impliedly state that P.D. 725 was being repealed or was
being rendered without any legal effect. In fact, she did not even mention it specifically by its
number or text. On the other hand, it is a basicrule of statutory construction that repeals by
implication are not favored. An implied repeal will not be allowed "unless it is convincingly
and unambiguously demonstrated that the two laws are clear repugnant and patently
inconsistent that they cannot co-exist."

It should also be noteworthy that the petition filed with the RTC was one for
repatriation, and it was thus incorrect for petitioner to initially invoke Republic Act No. 965
and R.A. No. 2630
since these laws could only apply to persons who had lost their citizenship by rendering
service to, or accepting commission in, the armed forces of an allied foreign country or the
armed forces of the United States of America, a factual matter not alleged in the petition.
Under these statutes, the person desiring to re-acquire Philippine citizenship would not even be
required to file a petition in court, and all that he had to do was to take an oath of allegiance to
the Republic of the Philippines and to register that fact with the civil registry in the place of his
residence or where he had last resided in the Philippines.
TABASA v CA
FULL TEXT: https://lawphil.net/judjuris/juri2006/aug2006/gr_125793_2006.html

TOPIC: REACQUISITION > Legislative Act - Which is both a mode of acquiring and
reacquiring citizenship

By Eden Maagad

FACTS

When he was 7 years old, Joevanie A. Tabasa acquired American citizenship when
his father became a naturalized citizen of the US. In 1995, he arrived in the Philippines
and was admitted as "balikbayan." Thereafter, he was arrested and detained by the
agent of BIR. The Consul General of the US embassy of Manila filed a request with
the BID that his passport has been revoked and that Tabasa had a standing warrant
for several federal charges against him. Petitioner alleged that he acquired Filipino
citizenship by repatriation in accordance with the RA No. 8171, and that because he is
now a Filipino citizen, he cannot be deported or detained by the BID.

ISSUE

Whether or not he has validly reacquired Philippine citizenship under RA 8171 and
therefore, is not an undocumented alien subject to deportation.

RULING

No. Petitioner is not qualified to avail himself of repatriation under RA 8171. The only
person entitled to repatriation under RA 8171 is either a Filipino woman who lost her
Philippine citizenship by marriage to an alien, or a natural-born Filipino, including his
minor children who lost Philippine citizenship on account of political or economic
necessity. Petitioner was already 35 years old when he filed for repatriation. The act
cannot be applied in his case because he is no longer a minor at the time of his
repatriation in 1996. The privilege under RA 8171 only belongs to children who are of
minor age at the time of filing of the petition for repatriation.
MERCADO VS. MANZANO
FULL TEXT: https://lawphil.net/judjuris/juri1999/may1999/gr_135083_1999.html
Digested by: Alyana Mahilum

TOPIC: Dual Citizenship

FACTS:

Petition for disqualification was filed by Ernesto Mamaril against Edu Manzano to hold elective
office on the ground that he is both an American citizen and a Filipino citizen, having been born
in the United States of Filipino parents. COMELEC granted the petition and disqualified
Manzano for being a dual citizen pursuant to the Local Government Code RA 7160, that those
with dual citizenship are disqualified from running any public position.

ISSUE:

Whether or not dual citizenship is a ground for disqualification to hold or run office in the local
position.

RULING:

No. Dual citizenship is different from dual allegiance. What is inimical is not dual citizenship per
se, but with naturalized citizens who maintain their allegiance to their countries of origin even
after their naturalization. Hence, the phrase “dual citizenship” in RA 7160 must be understood
as referring to “dual allegiance”. Consequently, persons with mere dual citizenship do not fall
under this disqualification.
AASJS VS. DATUMANONG
FULL TEXT: https://www.lawphil.net/judjuris/juri2007/may2007/gr_160869_2007.html
RALF VINCENT BAJO

TOPIC: Loss of and re-acquisition of Citizenship; RA 9225

FACTS:Petitioner prays that a writ of prohibition be issued to stop respondent


(Secretary of Justice Simeon Datumanong, the official tasked to implement laws governing citizenship)
from implementing Republic Act No. 9225, otherwise known as the Citizen Retention
and Re-acquisition Act of 2003. Under the said law, natural-born citizens of the
Philippines who become citizens of a foreign country shall retain or re-acquire their
Philippine citizenship upon taking the oath of allegiance to the Republic of the
Philippines as outlined in Section 3. Petitioner avers that RA 9225 is unconstitutional
as it violates Section 5, Article IV of the 1987 Constitution that states, "Dual allegiance
of citizens is inimical to the national interest and shall be dealt with by law."

ISSUE: Whether or not RA 9225 is unconstitutional.

RULING: No. RA 9225 is NOT UNCONSTITUTIONAL.

From the excerpts of the legislative record, it is clear that the intent of the legislature in
drafting RA 9225 is to do away with the provision in Commonwealth Act No. 635 which
takes away Philippine citizenship from natural-born Filipinos who become naturalized
citizens of other countries. What RA 9225 does is allow dual citizenship to
natural-born Filipino citizens who have lost Philippine citizenship by reason of their
naturalization as citizens of a foreign country. On its face, it does not recognize dual
allegiance. By swearing to the supreme authority of the Republic, the person implicitly
renounces his foreign citizenship. Plainly, from Section 3, RA 9225 stayed clear out of
the problem of dual allegiance and shifted the burden of confronting the issue of
whether or not there is dual allegiance to the concerned foreign country. What
happens to the other citizenship was not made a concern of RA 9225.

To begin with, Section 5, Article IV of the Constitution is a declaration of a policy and it


is not a self-executing provision. The legislature still has to enact the law on dual
allegiance. In Sections 2 and 3 of RA No. 9225, the framers were not concerned with
dual citizenship per se, but with the status of naturalized citizens who maintain their
allegiance to their countries of origin even after their naturalization.9 Congress was
given a mandate to draft a law that would set specific parameters of what really
constitutes dual allegiance.10 Until this is done, it would be premature for the judicial
department, including this Court, to rule on issues pertaining to dual allegiance.

NOTES: Mercado v. Manzano differentiates dual citizenship from dual allegiance:


Dual citizenship “arises when, as a result of the concurrent application of the different laws of two or more states, a
person is simultaneously considered a national by the said states. For instance, such a situation may arise when a
person whose parents are citizens of a state which adheres to the principle of jus sanguinis is born in a state which
follows the doctrine of jus soli. Such a person, ipso facto and without any voluntary act on his part, is concurrently
considered a citizen of both states.”

“Dual allegiance, on the other hand, refers to the situation in which a person simultaneously owes, by some positive
act, loyalty to two or more states. While dual citizenship is involuntary, dual allegiance is the result of an individual's
volition.”

PEDRO LEE HONG HOK VS. DAVID


Moralizon, Trixia Ann P.
TOPIC: Sovereignty
FACTS:
● This is an appeal to reverse a decision of respondent Court of Appeals
affirming a lower court judgment dismissing their complaint to have the Torrens
Title of respondent Aniano David declared null and void. The CA rejected the
petitioners’ factual support of their claim to ownership of the land through
accretion leaving them to rely on an unpersuasive legal theory.
● The CA in agreement with the lower court, concluded that there is no legal
justification for nullifying the right of respondent Aniano David to the disputed
lot.
● The land in question was not a private property thus, it was auctioned prior to a
published notice. David then renewed the application of his deceased wife who
had inhabited the land since 1938.
● In response, the Director of Lands made an order of award and for issuance of
the sales patent in 1958 while the Natural Resources issued a Miscellaneous
Sales Patent No. V-1209 on August 26, 1959. This entitled David to the legal
acquisition of the title pursuant to his miscellaneous sales application.
● On October 21, 1959, the Register of Deeds of Naga City issued the OCT No.
510 to David. It covers Lot 2892 containing an area of 226 square meters,
which is a portion of Lot 2863 of the Naga Cadastre Lot 2892 containing an
area of 226 square meters, which is a portion of Lot 2863 of the Naga
Cadastre.
● The petitioners did not put up any opposition or adverse claim during all these
proceedings neither did they question the validity of the certificate of the title
based on fraud within the year after the issuance of the patent, pursuant to
Section 38 of Act 496. Thereafter the certificate of title based thereon becomes
indefeasible

ISSUE:
Whether or not the petitioners can have the respondent’s certificate declared as null
and void.

RULING:
● No. Only the Government, represented by the Director of Lands, or the
Secretary of Agriculture and Natural Resources, can bring an action to cancel
a void certificate of title issued pursuant to a void patent (Lucas vs. Durian, 102
Phil. 1157; Director of Lands vs. Heirs of Ciriaco Carlo, G.R. No. L-12485, July
31, 1959).
● In this case, it was not done by said officers but by private parties like the
plaintiffs, who cannot claim that the patent and title issued for the land involved
are void since they are not the registered owners thereof nor had they been
declared as owners in the cadastral proceedings of Naga Cadastre after
claiming it as their private property.
● The cases cited by appellants are not in point as they refer to private
registered lands or public lands over which vested rights have been acquired
but notwithstanding such fact the Land Department subsequently granted
patents to public land applicants.
● The fact that the grant was made by the government is undisputed. Whether
the grant was in conformity with the law or not is a question which the
government may raise, but until it is raised by the government and set aside,
the defendant can not question it. The legality of the grant is a question
between the grantee and the government.
PROFESSIONAL VIDEO, INC. VS. TESDA
Full Text Link: https://lawphil.net/judjuris/juri2009/jun2009/gr_155504_2009.html
Lig Andre S. Nuesca
TOPIC: Sovereign Immunity
FACTS: PROVI is an entity engaged in the sale of high technology equipment, information
technology products and broadcast devices, including the supply of plastic card printing and
security facilities.TESDA is an instrumentality of the government established under
Republic Act (R.A.) No. 7796 (the TESDA Act of 1994) and attached to the Department of
Labor and Employment (DOLE) to "develop and establish a national system of skills
standardization, testing, and certification in the country.” To fulfill this mandate, it sought
to issue security-printed certification and/or identification polyvinyl (PVC) cards to trainees who
have passed the certification process.

The PBAC recommended that TESDA enter into a negotiated contract with PROVI. On
December 29, 1999, TESDA and PROVI signed and executed their "Contract Agreement
Project: PVC ID Card Issuance" (the Contract Agreement) for the provision of goods and
services in the printing and encoding of PVC cards. Under this Contract Agreement,
PROVI was to provide TESDA with the system and equipment compliant with the
specifications defined in the Technical Proposal. In return, TESDA would pay PROVI the
amount of Thirty-Nine Million Four Hundred and Seventy-Five Thousand Pesos
(₱39,475,000) within fifteen (15) days after TESDA’s acceptance of the contracted goods
and services.

PROVI further alleged that out of TESDA’s liability of ₱39,475,000.00, TESDA paid
PROVI only ₱3,739,500.00, leaving an outstanding balance of ₱35,735,500.00, as
evidenced by PROVI’s Statement of Account. Despite the two demand letters dated March
8 and April 27, 2001 that PROVI sent TESDA, the outstanding balance remained unpaid.

ISSUE: WON TESDA cannot be sued without its consent.

RULING: Under these terms, both constitutional and statutory, we do not believe that the role
and status of TESDA can seriously be contested: it is an unincorporated instrumentality of the
government, directly attached to the DOLE through the participation of the Secretary of Labor
as its Chairman, for the performance of governmental functions – i.e., the handling of formal
and non-formal education and training, and skills development. As an unincorporated
instrumentality operating under a specific charter, it is equipped with both express and implied
powers, and all State immunities fully apply to it.

TESDA, as an agency of the State, cannot be sued without its consent.The rule
that a state may not be sued without its consent is embodied in Section 3, Article XVI of
the 1987 Constitution and has been an established principle that antedates this
Constitution. It is as well a universally recognized principle of international law that
exempts a state and its organs from the jurisdiction of another state. The principle is
based on the very essence of sovereignty, and on the practical ground that there can be no
legal right as against the authority that makes the law on which the right depends. It also rests
on reasons of public policy — that public service would be hindered, and the public
endangered, if the sovereign authority could be subjected to law suits at the instance of
every citizen and, consequently, controlled in the uses and dispositions of the means
required for the proper administration of the government.

The proscribed suit that the state immunity principle covers takes on various forms,
namely: a suit against the Republic by name; a suit against an unincorporated government
agency; a suit against a government agency covered by a charter with respect to the agency’s
performance of governmental functions; and a suit that on its face is against a government
officer, but where the ultimate liability will fall on the government.
REPUBLIC VS. VILLASOR
Louise Francis P. Ouano

TOPIC: Sovereignty (Suability of the State)

FACTS:
The case was filed by the Republic of the Philippines requesting to nullify the ruling of
The CFI in Cebu in garnishing the public funds allocated for the AFP. A decision was
rendered in Special Proceedings in favor of respondents P. J. Kiener Co., Ltd., Gavino
Unchuan, and International Construction Corporation, and against the petitioner
herein, confirming the arbitration award in the amount of P1,712,396.40, subject of
Special Proceedings.

The respondent Hon. Guillermo P. Villasor, issued an Order declaring the said decision
final and executory, directing the Sheriffs of Rizal Province, Quezon City and Manila to
execute the said decision. The corresponding Writ of Execution was issued. On the
strength of the aforementioned Alias Writ of Execution, the Provincial Sheriff of Rizal
served Notices of Garnishment with several Banks. The funds of the Armed Forces of
the Philippines on deposit with Philippine Veterans Bank and PNB are public funds
duly appropriated and allocated for the payment of pensions of retirees, pay and
allowances of military and civilian personnel and for maintenance and operations of
the AFP.

Petitioner, filed prohibition proceedings against respondent Judge Villasor for acting in
excess of jurisdiction with grave abuse of discretion amounting to lack of jurisdiction in
granting the issuance of a Writ of Execution against the properties of the AFP, hence
the notices and garnishment are null and void.

ISSUE:
W/N the state can be sued without its consent.

RULING:
No. It is a fundamental postulate of constitutionalism flowing from the juristic concept
of sovereignty that the state as well as its government is immune from suit unless it
gives its consent. It is readily understandable why it must be so. In the classic
formulation of Holmes: “A sovereign is exempt from suit, not because of any formal
conception or obsolete theory, but on the logical and practical ground that there can
be no legal right as against the authority that makes the law on which the right
depends.” This fundamental postulate underlying the 1935 Constitution is now made
explicit in the revised charter.

It is therein expressly provided: “The State may not be sued without its consent.” A
corollary, both dictated by logic and sound sense from such a basic concept is that
public funds cannot be the object of a garnishment proceeding even if the consent to
be sued had been previously granted and the state liability adjudged. The universal
rule that where the State gives its consent to be sued by private parties either by
general or special law, it may limit claimant’s action ‘only up to the completion of
proceedings anterior to the stage of execution’ and that the power of the Courts ends
when the judgment is rendered, since government funds and properties may not be
seized under writs of execution or garnishment to satisfy such judgments, is based on
obvious considerations of public policy.

Disbursements of public funds must be covered by the corresponding appropriation as


required by law. The functions and public services rendered by the State cannot be
allowed to be paralyzed or disrupted by the diversion of public funds from their
legitimate and specific objects, as appropriated by law.
JUSMAG VS. NLRC
Digested by: Vinceromae Allysson Reveche

TOPIC: Sovereign Immunity


Link to Full Text: https://lawphil.net/judjuris/juri1994/dec1994/gr_108813_1994.html

FACTS: On March 31, 1992, private respondent Florencio Sacramento, filed a complaint
with the Department of Labor and Employment on the ground that he was illegally
suspended and dismissed from service by JUSMAG. He asked for his reinstatement.

JUSMAG then filed a Motion to Dismiss invoking its immunity from suit as an agency of
the United States. It further alleged lack of employer-employee relationship and that it has no
juridical personality to sue and be sued.

The Labor Arbiter in an Order, dismissed the complaint for want of jurisdiction. Sacramento
(private respondent) appealed to the NLRC (public respondent) which reversed the decision of
the Labor Arbiter in a resolution and held that the petitioner had lost his right not to be sued.
This resolution was predicated on two grounds: (1) the principle of estoppel — that JUSMAG
failed to refute the existence of employer-employee relationship under the "control test"; and
(2) JUSMAG has waived its right to immunity from suit when it hired the services of private
respondent on December 18, 1969

Accordingly, the case was remanded to the labor arbiter for reception of evidence as to
the issue of illegal dismissal. Hence, this petition for certiorari by JUSMAG.

ISSUE: Whether or not petitioner can invoke state immunity as an agency of the United
States.

RULING:

Yes, petitioner can invoke state immunity as an agency of the United States. In this
jurisdiction, we recognize and adopt the generally accepted principles of international
law as part of the law of the land. Immunity of State from suit is one of these universally
recognized principles. In international law, "immunity" is commonly understood as an
exemption of the state and its organs from the judicial jurisdiction of another state. This is
anchored on the principle of the sovereign equality of states under which one state cannot
assert jurisdiction over another in violation of the maxim par in parem non habet imperium (an
equal has no power over an equal).

The doctrine of state immunity from suit has undergone further metamorphosis. The view
evolved that the existence of a contract does not, per se, mean that sovereign states may, at
all times, be sued in local courts. The complexity of relationships betwee`n sovereign states,
brought about by their increasing commercial activities, mothered a more restrictive application
of the doctrine. Thus, in United States of America vs. Ruiz, we clarified that our pronouncement
in Harry Lyons, supra, with respect to the waiver of State immunity, was obiter and "has no
value as an imperative authority."

We sympathize with the plight of private respondent who had served JUSMAG for more than
twenty (20) years. Considering his length of service with JUSMAG, he deserves a more
compassionate treatment. Unfortunately, JUSMAG is beyond the jurisdiction of this Court.
MOST REV. PEDRO ARIGO, et. al., Petitioners, vs. SCOTT H. SWIFT, et. al.,
Respondents. G.R. No. 206510 (September 16, 2014)

Digested by: Benjamin Misoles Jr.

TOPIC: Writ of Kalikasan, UNCLOS, Immunity from suit

FACTS:

The USS Guardian is an Avenger-class mine countermeasures ship of


the US Navy. In December 2012, the US Embassy in the Philippines requested
diplomatic clearance for the said vessel “to enter and exit the territorial waters of the
Philippines and to arrive at the port of Subic Bay for the purpose of routine ship
replenishment, maintenance, and crew liberty.” On January 6, 2013, the ship left
Sasebo, Japan for Subic Bay, arriving on January 13, 2013 after a brief stop for fuel in
Okinawa, Japan.

On January 15, 2013, the USS Guardian departed Subic Bay for its
next port of call in Makassar, Indonesia. On January 17, 2013 at 2:20 a.m. while
transiting the Sulu Sea, the ship ran aground on the northwest side of South Shoal of
the Tubbataha Reefs, about 80 miles east-southeast of Palawan. No one was injured
in the incident, and there have been no reports of leaking fuel or oil.

Petitioners claim that the grounding, salvaging and post-salvaging


operations of the USS Guardian cause and continue to cause environmental damage
of such magnitude as to affect the provinces of Palawan, Antique, Aklan, Guimaras,
Iloilo, Negros Occidental, Negros Oriental, Zamboanga del Norte, Basilan, Sulu, and
Tawi-Tawi, which events violate their constitutional rights to a balanced and healthful
ecology.

ISSUES:

1. Whether or not petitioners have legal standing.

2. Whether or not US respondents may be held liable for damages caused by USS
Guardian.

3. Whether or not the waiver of immunity from suit under VFA applies in this case.

HELD:

First issue: YES. Petitioners have legal standing. Locus standi is “a right of
appearance in a court of justice on a given question.” Specifically, it is “a party’s
personal and substantial interest in a case where he has sustained or will sustain
direct injury as a result” of the act being challenged, and “calls for more than just a
generalized grievance.” However, the rule on standing is a procedural matter which
this Court has relaxed for non-traditional plaintiffs like ordinary citizens, taxpayers and
legislators when the public interest so requires, such as when the subject matter of the
controversy is of transcendental importance, of overreaching significance to society, or
of paramount public interest.
Second issue: YES. The US respondents were sued in their official capacity as
commanding officers of the US Navy who had control and supervision over the
USS Guardian and its crew. The alleged act or omission resulting in the unfortunate
grounding of the USS Guardian on the TRNP was committed while they were
performing official military duties. Considering that the satisfaction of a judgment
against said officials will require remedial actions and appropriation of funds by the US
government, the suit is deemed to be one against the US itself. The principle of State
immunity therefore bars the exercise of jurisdiction by this Court over the persons of
respondents Swift, Rice and Robling.

Third issue: NO. The waiver of State immunity under the VF A pertains only to
criminal jurisdiction and not to special civil actions such as the present petition
for issuance of a writ of Kalikasan. In fact, it can be inferred from Section 17, Rule 7
of the Rules that a criminal case against a person charged with a violation of an
environmental law is to be filed separately. The Court also found unnecessary at this
point to determine whether such waiver of State immunity is indeed absolute. In the
same vein, we cannot grant damages which have resulted from the violation of
environmental laws. The Rules allows the recovery of damages, including the
collection of administrative fines under R.A. No. 10067, in a separate civil suit or that
deemed instituted with the criminal action charging the same violation of an
environmental law.
AMONOY VS. SPS. GUTIERREZ
Eden Maizelle Salialam

TOPIC: Part 4 PH as a State > Government > Sovereignty > Civil Law Basis
Link to full text: https://lawphil.net/judjuris/juri2001/feb2001/gr_140420_2001.html

FACTS:

● This case had its roots in Special Proceedings of the CFI of Pasig, Rizal, for
the settlement of the estate of the deceased Julio Cantolos, involving six(6)
parcels of land situated in Tanay Rizal. Amonoy was the counsel of Francisca
Catolos, Agnes Catolos, Asuncion Pasamba and Alfonso Formida.
● On 12 January 1965, the Project of Partition submitted was approved and xxx
two (2) of the said lots were adjudicated to Asuncion Pasamba and Alfonso
Formida. The Attorney's fees charged by Amonoy was P27,600.00. On 20
January 1965, Asuncion Pasamba and Alfonso Formida executed a deed of
real estate mortgage on the said two (2) lots adjudicated to them, in favor of
Amonoy to secure the payment of his attorney's fees. But it was only on 6
August 1969 after the taxes had been paid, the claims settled and the
properties adjudicated, that the estate was declared closed and terminated.
● Asuncion Pasamba died on 24 February 1969 while Alfonso Fornilda passed
away on 2 July 1969. Among the heirs of the latter was his daughter,
plaintiff-appellant Angela Gutierrez.
● The Order of respondent Trial Court, dated 25 July 1985, granting a Writ of
Possession, as well as its Ordered, dated 25 April 1986 and 16 May 1986,
directing and authorizing respondent Sheriff to demolish the houses of
petitioners Angela and Leocadia Fornilda are hereby ordered returned to
petitioners unless some of them have been conveyed to innocent third
persons.
● But by the time the Supreme Court promulgated the above mentioned
Decision, respondents' house had already been destroyed, supposedly in
accordance with a Writ of Demolition ordered by the lower court.
● In this case, the petitioner was contending that the damages claimed by the
respondent are not valid because he is just exercising his right.

ISSUE: W/N Damnum absque injuria is applicable to this case, as petitioner invokes
this legal precept in arguing that he is not liable for the demolition of respondents'
house.

RULING:

No. Damnum absque injuria finds no application to this case.


Although the acts of petitioner may have been legally justified at the outset, their
continuation after the issuance of the TRO amounted to an insidious abuse of his
right. Indubitably, his actions were tainted with bad faith. Had he not insisted on
completing the demolition, respondents would not have suffered the loss that
engendered the suit before the RTC. Verily, his acts constituted not only an abuse of a
right, but an invalid exercise of a right that had been suspended when he received the
TRO from this Court on June 4, 1986. By then he was no longer entitled to proceed
with the demolition.

Clearly then, the demolition of respondents' house by petitioner, despite his receipt of
the TRO, was not only an abuse but also an unlawful exercise of such right. In
insisting on his alleged right, he wantonly violated this Court's Order and wittingly
caused the destruction of respondent's house.
1âwphi1.nêt

Obviously, petitioner cannot invoke damnum absque injuria, a principle premised on


the valid exercise of a right.14 Anything less or beyond such exercise will not give rise
to the legal protection that the principle accords. And when damage or prejudice to
another is occasioned thereby, liability cannot be obscured, much less abated.
PROFESSIONAL VIDEO INC. VS. TESDA
Link to the full text: GR No. 155504
Digested by: Kaye Turija

TOPIC: Part 4 PH as a State > Government > Sovereignty > When is a suit against the state?

FACTS: PBAC recommended that TESDA enter into a negotiated contract with PROVI. On
December 29, 1999, TESDA and PROVI signed and executed their "Contract Agreement
Project: PVC ID Card Issuance" (the Contract Agreement) for the provision of goods and
services in the printing and encoding of PVC cards. According to PROVI, it delivered the items
subject to the agreement to TESDA.

PROVI alleged that out of TESDA’s liability of P39,475,000.00, TESDA paid PROVI only
P3,739,500.00, leaving an outstanding balance of P35,735,500.00, as evidenced by PROVI’s
Statement of Account. Despite the two demand letters dated March 8 and April 27, 2001 that
PROVI sent to TESDA, the outstanding balance remained unpaid.

PROVI filed with the RTC a complaint for a sum of money with damages against TESDA.
The RTC granted PROVI’s prayer and issued a writ of preliminary attachment against the
properties of TESDA not exempt from execution in the amount of P35,000,000.00. TESDA
responded on July 24, 2001 by filing a Motion to Discharge/Quash the Writ of Attachment,
arguing mainly that public funds cannot be the subject of garnishment. The RTC denied
TESDA’s motion. Faced with these rulings, TESDA filed a Petition for Certiorari with the CA to
question the RTC orders. The CA set aside the RTC’s orders

ISSUE:
RULING:

TESDA, as an agency of the State, cannot be sued without its consent.

TESDA is an instrumentality of the government undertaking governmental functions. Its funds


are public in character, hence exempt from attachment or garnishment. Under these terms,
both constitutional and statutory, we do not believe that the role and status of TESDA can
seriously be contested: it is an unincorporated instrumentality of the government, directly
attached to the DOLE through the participation of the Secretary of Labor as its Chairman, for
the performance of governmental functions – i.e., the handling of formal and non-formal
education and training, and skills development. As an unincorporated instrumentality operating
under a specific charter, it is equipped with both express and implied powers, and all State
immunities fully apply to it.

The rule that a state may not be sued without its consent is embodied in Section 3, Article XVI
of the 1987 Constitution and has been an established principle that antedates this
Constitution. It is as well a universally recognized principle of international law that exempts a
state and its organs from the jurisdiction of another state. The principle is based on the very
essence of sovereignty, and on the practical ground that there can be no legal right as against
the authority that makes the law on which the right depends. It also rests on reasons of public
policy — that public service would be hindered, and the public endangered, if the sovereign
authority could be subjected to law suits at the instance of every citizen and, consequently,
controlled in the uses and dispositions of the means required for the proper administration of
the government.

The proscribed suit that the state immunity principle covers takes on various forms, namely:
1. a suit against the Republic by name;
2. a suit against an unincorporated government agency;
3. a suit against a government agency covered by a charter with respect to the agency’s
performance of governmental functions;
4. and a suit that on its face is against a government officer, but where the ultimate liability will
fall on the government.

In the present case, the writ of attachment was issued against a government agency
covered by its own charter.

TESDA performs governmental functions, and the issuance of certifications is a task within its
function of developing and establishing a system of skills standardization, testing, and
certification in the country. From the perspective of this function, the core reason for the
existence of state immunity applies – i.e The public policy reason is that the performance of
governmental function cannot be hindered or delayed by suits, nor can these suits control the
use and disposition of the means for the performance of governmental functions. [Pascua
notes, pp. 84-86]

The appellate court’s conclusion was supported by the SC that no valid ground exists to
support the grant of the writ of attachment against TESDA. The CA’s annulment and setting
aside of the Orders of the RTC were therefore fully in order. WHEREFORE, the petition filed by
petitioner Professional Video, Inc. (PROVI) was DENIED.
ATO VS. SPS. RAMOS
Kyra Valentin
TOPIC: Sovereignty: When is a suit against the state?
FACTS: Respondent Spouses discovered that a portion of their registered land in Baguio City was
being used as part of the runway and running shoulder of the Loakan Airport being operated by
petitioner Air Transportation Office (ATO). The respondents agreed after negotiations to convey the
affected portion by deed of sale to the ATO in consideration of the amount of P778,150.00. However,
the ATO failed to pay despite repeated verbal and written demands.

Thus, the respondents filed an action for collection against the ATO and some of its officials in the RTC.
In their answer, the ATO and its co-defendants invoked as an affirmative defense the issuance of
Proclamation No. 1358, whereby President Marcos had reserved certain parcels of land that included
the respondents affected portion for use of the Loakan Airport. They asserted that the RTC had no
jurisdiction to entertain the action without the States consent considering that the deed of sale had
been entered into in the performance of governmental functions.

The RTC held in favor of the Spouses, ordering the ATO to pay the plaintiffs Spouses the amount of
P778,150.00 being the value of the parcel of land appropriated by the defendant ATO as embodied in
the Deed of Sale, plus an annual interest of 12% from August 11, 1995, the date of the Deed of Sale
until fully paid; (2) The amount of P150,000.00 by way of moral damages and P150,000.00 as exemplary
damages; (3) the amount of P50,000.00 by way of attorneys fees plus P15,000.00 representing the 10,
more or less, court appearances of plaintiffs counsel; (4) The costs of this suit.

On appeal, the CA affirmed the RTCs decision with modification deleting the awarded cost, and
reducing the moral and exemplary damage to P30,000.00 each, and attorneys fees is lowered to
P10,000.00.

ISSUE: Could ATO be sued without the State's consent?


RULING: An unincorporated government agency without any separate juridical personality of its
own enjoys immunity from suit because it is invested with an inherent power of sovereignty.
Accordingly, a claim for damages against the agency cannot prosper; otherwise, the doctrine of
sovereign immunity is violated. However, the need to distinguish between an unincorporated
government agency performing governmental function and one performing proprietary functions has
arisen. The immunity has been upheld in favor of the former because its function is governmental or
incidental to such function; it has not been upheld in favor of the latter whose function was not in
pursuit of a necessary function of government but was essentially a business. National Airports
Corporation v. Teodoro, Sr. and Phil. Airlines Inc., 91 Phil. 203 (1952)

Civil Aeronautics Administration vs. Court of Appeals (167 SCRA 28 [1988]),the Supreme Court,
reiterating the pronouncements laid down in Teodoro, declared that the CAA (predecessor of ATO) is an
agency not immune from suit, it being engaged in functions pertaining to a private entity.

The Civil Aeronautics Administration comes under the category of a private entity. Although not a body
corporate it was created, like the National Airports Corporation, not to maintain a necessary function of
government, but to run what is essentially a business, even if revenues be not its prime objective but
rather the promotion of travel and the convenience of the travelling public. It is engaged in an
enterprise which, far from being the exclusive prerogative of state, may, more than the construction of
public roads, be undertaken by private concerns. National Airports Corp. v. Teodoro, 91 Phil. 203 (1952)

The CA thereby correctly appreciated the juridical character of the ATO as an agency of the Government
not performing a purely governmental or sovereign function, but was instead involved in the
management and maintenance of the Loakan Airport, an activity that was not the exclusive prerogative
of the State in its sovereign capacity. Hence, the ATO had no claim to the States immunity from suit. We
uphold the CAs aforequoted holding.

The doctrine of sovereign immunity cannot be successfully invoked to defeat a valid claim for
compensation arising from the taking without just compensation and without the proper expropriation
proceedings being first resorted to of the plaintiffs property. Republic v. Sandiganbayan, G.R. No. 90478,
Nov. 2, 1991. DENIED.
CHINA NATIONAL MACHINERY & EQUIPMENT CORP.
(GROUP) VS. SANTA MARIA
NALCOT, MPAA
Full text: https://lawphil.net/judjuris/juri2012/feb2012/gr_185572_2012.html

TOPIC: When is a suit against the State?

FACTS: Petitioner China National Machinery & Equipment Corp (CNMEG) entered into a
Memorandum of Understanding(MOU)with North Luzon Railways Corporation (Northrail) for a
feasibility study on a possible railway line from Manila to San Fernando, LU.

On Aug 30, 2003, the Export Import Bank of China (EXIM) and the DOF entered into an MOU
where China agreed to extend Preferential Buyer’s Credit to the Philippine gov’t to finance the
Northrail Project. EXIM agreed to extend an amount not exceeding $400M, payable in 20 years
with a 5-year grace period at the rate of 3% per annum.

On Oct 1, 2003, the Chinese Ambassador wrote a letter to the DOF Secretary informing him of
CNMEG’s designation as the Prime Contractor.

On Dec 30, 2003, Northrail and CNMEG executed a Contract Agreement for the construction
of Phase I of the project for the price of $421.05M.The respondents filed a complaint
alleging that the Contract and Loan Agreements were void for being contrary to the
Constitution and various laws.

The CNMEG filed a Motion to Dismiss arguing that the trial court did not have jurisdiction over
its person since it was an agent of the Chinese gov’t, hence, immune from suit.

The RTC denied the motion. So did the CA when CNMEG appealed it to them.

ISSUE: WON CNMEG is entitled to immunity from suit

RULE: No. This Court explained the doctrine of sovereign immunity in Holy See v. Rosario
(G.R. No. 101949, 1 December 1994), to wit:

There are two conflicting concepts of sovereign immunity, each widely held and firmly
established. According to the classical or absolute theory, a sovereign cannot, without
its consent, be made a respondent in the courts of another sovereign. According to the
newer or restrictive theory, the immunity of the sovereign is recognized only with
regard to public acts or acts jure imperii of a state, but not with regard to private acts or
acts jure gestionis.

Since the Philippines adheres to the restrictive theory, it is crucial to ascertain the legal nature
of the act involved – whether the entity claiming immunity performs governmental, as opposed
to proprietary, functions. As held in United States of America v. Ruiz (221 Phil. 179 -1985)

The restrictive application of State immunity is proper only when the proceedings arise
out of commercial transactions of the foreign sovereign, its commercial activities or
economic affairs. Stated differently, a State may be said to have descended to the
level of an individual and can thus be deemed to have tacitly given its consent to be
sued only when it enters into business contracts. It does not apply where the contract
relates to the exercise of its sovereign functions.

A thorough examination of the basic facts of the case would show that CNMEG is
engaged in a proprietary activity.

The parties executed the Contract Agreement for the purpose of constructing the Luzon
Railways. The Memorandum of Understanding dated 14 September 2002 shows that CNMEG
sought the construction of the Luzon Railways as a proprietary venture. Also, that CNMEG,
and not the Chinese government, initiated the Northrail Project was confirmed. Thus, the desire
of CNMEG to secure the Northrail Project was in the ordinary or regular course of its business
as a global construction company. The implementation of the Northrail Project was intended to
generate profit for CNMEG.

The use of the term "state corporation" to refer to CNMEG was only descriptive of its
nature as a government-owned and/or -controlled corporation, and its assignment as
the Primary Contractor did not imply that it was acting on behalf of China in the
performance of the latter’s sovereign functions. [Ruling extracted from Atty. Pascua’s
Notes]
LANSANG VS. CA
Angelica Wasawas

Topic: Sovereignty: When is a suit against the state?

FACTS: Private respondent General Assembly of the Blind (GABI) were allegedly awarded a
verbal contract of lease in Rizal Park by the National Parks Development Committee (NPDC).
The private respondents were allegedly given office and library space, as well as kiosks area,
where to sell food and drinks- then to remit to NPDC, 40 percent of the profits derived from
operating the kiosks.

With the change of government, the new Chairman of the NPDC, Amadao J. Lansang, herein
petitioner, sought to clean up Rizal Park. In a written notice (dated February 23, 1988)
received by private respondents (February 29, 1988), petitioner terminated the so-called
verbal agreement with GABI and demanded that the latter vacate the premises and the
kiosks it ran privately within the public park. In another notice, the respondents were given
until March 8, 1988 to vacate. The latter notice was signed by the president of GABI, private
respondent Jose Iglesias, who is totally blind, allegedly to indicate his conformity to its
contents but later on claimed that he was deceived into signing the notice. He was allegedly
told by Ricardo Villanueva, then chief warden of Rizal Park, that he was merely acknowledging
receipt of the notice. Although blind, Iglesias as president was knowledgeable enough to run
GABI as well as its business.

On the day of the supposed eviction, GABI filed an action for damages and injunction in the
RTC against the petitioner but it was dismissed, ruling that the complaint was actually directed
against the state which could not be sued without its consent. On appeal, the Court of
Appeals reversed the decision of the RTC and ruled that a government official being sued in
his official capacity is not enough to protest such official from liability for acts done without or
in excess of his authority. Hence, the petition for review.

ISSUE: Whether or not private respondents' complaint against petitioner Lansang, as


Chairman of NPDC, is in effect a suit against the state which cannot be sued without its
consent.

RULING: No, the complaint is not a suit against the state. The doctrine of state immunity from suit
applies to complaints filed against public officials for acts done in the performance of their duties.
The rule is that the suit must be regarded as one against the state where satisfaction of the judgment
against the public official concerned will require the state itself to perform a positive act, such as
appropriation of the amount necessary to pay the damages awarded to the plaintiff. The rule does
not apply where the public official is charged in his official capacity for acts that are unlawful and
injurious to the rights of others. Public officials are not exempt, in their personal capacity, from
liability arising from acts committed in bad faith. Neither does it apply where the public official is
clearly being sued not in his official capacity but in his personal capacity, although the acts
complained of may have been committed while he occupied a public position.
We are convinced that petitioner is being sued not in his capacity as NPDC chairman but in his
personal capacity. The complaint filed by private respondents in the RTC merely identified petitioner
as chairman of the NPDC, but did not categorically state that he is being sued in that capacity.

CALUB VS. CA
Marian Isabel D. Yu
TOPIC:
FACTS:
On January 1992, two motor vehicles loaded with illegally sourced lumber were
apprehended by the Forest Protection and Law Enforcement Team of DENR-CENRO.
The drivers of the vehicles failed to present proper documents. Thus, the
apprehending team seized and impounded the vehicles and its load of lumber for
failing to produce requisite permits and licenses. The drivers (Abuganda and Gabon)
refused the seizure receipts. A few days later, the impounded vehicles were forcibly
taken by the drivers from the custody of DENR. Thereafter, one of the 2 vehicles was
again apprehended by a composite team of DENR-CENRO and Phil. Army elements.
The vehicle was again loaded with forest products.
Private respondents Manuela Babalcon, the vehicle owner, and Constancio
Abuganda, the driver, filed a complaint for the recovery of possession of the vehicle
with an application for replevin against petitioners DENR and DENR Officer Calub.

ISSUE:
W/N the complaint for the recovery of possession of impounded vehicles, with an
application for replevin, is a suit against the State.

RULING:
Yes. Well established is the doctrine that the State may not be sued without its
consent. And a suit against a public officer for his official acts is, in effect, a suit
against the State if its purpose is to hold the State ultimately liable. However, the
protection afforded to public officers by this doctrine generally applies only to activities
within the scope of their authority in good faith and without willfulness, malice or
corruption.
In the present case, the acts for which the petitioners are being called to account were
performed by them in the discharge of their official duties. The acts in question are
clearly official in nature. In implementing and enforcing Secs. 78-A and 89 of the
Forestry Code through the seizure carried out, petitioners were performing their duties
and functions as officers of the DENR, and did so within the limits of their authority.
There was no malice or bad faith on their part. Hence, a suit against the petitioners
who represent the DENR is a suit against the State. It cannot prosper without the
State’s consent.
PHILIPPINE AGILA SATELLITE INC. VS. DE GUZMAN
Jan Joy Zamora
TOPIC: Suit against the State
FACTS: Petitioners are Philippine Agila Satellite Inc. (PASI) and its President and
Chief Executive Officer Michael De Guzman. PASI was established by a consortium of
private telecommunications carriers which in 1994 had entered into a Memorandum of
Understanding (MOU) with the DOTC, through its then Secretary Jesus Garcia,
concerning the planned launch of a Philippine-owned satellite into outer space. Under
the MOU, the launch of the satellite was to be an endeavor of the private sector, and
the satellite itself to be owned by the Filipino-owned consortium (subsequently
organized as PASI). Petitioners filed an action against the new DOTC Secretary
Lichauco for allegedly having awarded the orbital slot to an unknown awardee.

The complaint, alleging three (3) causes of action, was for injunction,
declaration of nullity of award, and damages. The first cause of action, for
injunction, sought to establish that the award of orbital slot 153º East Longitude should
be enjoined since the DOTC had previously assigned the same orbital slot to PASI.
The second cause of action, for declaration of nullity of award, averred that the award
to the unknown bidder is null and void, as it was rendered by Lichauco beyond her
authority.

The complaint was filed before the Regional Trial Court (RTC) of Mandaluyong City,
and subsequently the RTC issued a temporary restraining order against Lichauco.
Defendant filed a Motion to admit with attached motion to Dismiss. The RTC denied
the motion to dismiss.

PASI moved to reconsider the dismissal of the complaint, but was denied by Order
dated July 17, 1998 a motion to dismiss the civil case against respondent was denied
by the trial court. On elevation of the order of denial to the Court of Appeals, said
court, by Decision dated February 21, 2000, ordered the dismissal of the case. This
Court, by Decision dated May 3, 2006, ordered the reinstatement of the case,
however.

ISSUE: WON the doctrine of non-suability of the State find application in this case?

RULING: NO. The Court rules that the defense of state immunity from suit do not
apply since said causes of action cannot be properly considered as suits against the
State in constitutional contemplation. These causes of action do not seek to impose a
charge or financial liability against the State, but merely the nullification of state action.
The prayers attached to these two causes of action are for the revocation of the Notice
of Bid and the nullification of the purported award, nothing more. Had it been so that
petitioner additionally sought damages in relation to said causes of action, the suit
would have been considered as one against the State. Had the petitioner impleaded
the DOTC itself, an unincorporated government agency, and not Lichauco herself, the
suit would have been considered as one against the State. But neither circumstance
obtains in this case.

The suit is to the mind of this court a suit against the state. At times, it would be
teasingly obvious, even from the moment of the filing of the complaint, that the suit is
one against the State. A cursory examination of the caption of the complaint can
sometimes betray such proscribed intent, as when the suit is directly initiated against
the Republic of the Philippines, any foreign government, or an unincorporated
government agency as the named respondents. In such cases, obviously there is
need for immediate caution, although if it is somehow established that those
respondents had given their consent to be sued, the suit may nonetheless prosper.

The present action was denominated against Lichauco and the unknown awardee,
Lichauco was identified in the complaint as "acting Secretary of the [DOTC]." The
hornbook rule is that a suit for acts done in the performance of official functions
against an officer of the government by a private citizen which would result in a charge
against or financial liability to the government must be regarded as a suit against the
State itself, although it has not been formally impleaded. However, government
immunity from suit will not shield the public official being sued if the government no
longer has an interest to protect in the outcome of a suit; or if the liability of the officer
is personal because it arises from a tortious act in the performance of his/her duties.

It is a different matter where the public official is made to account in his capacity as
such for acts contrary to law and injurious to the rights of the plaintiff. As was clearly
set forth by Justice Zaldivar in Director of the Bureau of Telecommunications, et al. vs.
Aligaen, etc., et al. 'Inasmuch as the State authorizes only legal acts by its officers,
unauthorized acts of government officials or officers are not acts of the State,
and an action against the officials or officers by one whose rights have been invaded
or violated by such acts, for the protection of his rights, is not a suit against the State
within the rule of immunity of the State from suit. In the same tenor, it has been said
that an action at law or suit in equity against a State officer or the director of a State
department on the ground that, while claiming to act for the State, he violates or
invades the personal and property rights or the plaintiff, under an unconstitutional act
or under an assumption of authority which he does not have, is not a suit against the
State within the constitutional provision that the State may not be sued without its
consent.' The rationale for this ruling is that the doctrine of state immunity cannot be
used as an instrument for perpetrating an injustice.

WHEREFORE, the PETITION is GRANTED. The Decision of the Court of Appeals


dated 21 February 2000 is SET ASIDE and the Order dated 14 August 1998 of the
Regional Trial Court of Mandaluyong City is REINSTATED. The Regional Trial Court is
ordered to try and decide the case on the merits with deliberate dispatch. No costs.
DOH VS. PHIL. PHARMAWEALTH, INC.
Digested by Jannah Ajoc
TOPIC: Consent to be sued

FACTS: On December 22, 1998, the Secretary of Department of Health Alberto G. Romualdez,
Jr. issued Administrative Order No. 27, Series of 1998, outlining the guidelines and procedures
on the accreditation of government suppliers for pharmaceutical products for sale or
distribution to the public, such accreditation to be valid for three years but subject to annual
review. On January 25, 2000, Secretary Romualdez issued the AO 10 series of 20006 which
amended AO 27. Under Section VII7 of AO 10, the accreditation period for government
suppliers of pharmaceutical products was reduced to two years.

Moreover, such accreditation may be recalled, suspended, or revoked after due deliberation
and proper notice by the DOH Accreditation Committee, through its chairman. Section VII of
AO 10 was later amended by the AO 66 series of 2000, which provided that the two-year
accreditation period may be recalled, suspended or revoked only after due deliberation,
hearing, and notice by the DOH Accreditation Committee, through its Chairman. The
respondent submitted to petitioner DOH a request for the inclusion of additional items in its list
of accredited drug products. Petitioners subsequently filed a Manifestation and Motion to
dismiss praying for the outright dismissal of the complaint based on the doctrine of state
immunity.

ISSUE: Whether or not a motion to dismiss based on the doctrine of state immunity will
prosper.

RULING: The suability of a government official depends on whether the official concerned was
acting within his official or jurisdictional capacity, and whether the acts done in the performance
of official functions will result in a charge or financial liability against the government. In the first
case, the Constitution itself assures the availability of judicial review, and it is the official
concerned who should be impleaded as the proper party. In its complaint, respondent
sufficiently imputes grave abuse of discretion against petitioners in their official capacity. Since
judicial review of acts alleged to have been tainted with grave abuse of discretion is
guaranteed by the Constitution, it necessarily follows that it is the official concerned who should
be impleaded as defendant or respondent in an appropriate suit.

As regards petitioner DOH, the defense of immunity from suit will not avail despite its being an
unincorporated agency of the government, for the only causes of action directed against it are
preliminary injunction and mandamus. Under Section 1, Rule 58 of the Rules of Court, a
preliminary injunction may be directed against a party or a court, agency or a person.
Moreover, the defense of state immunity from suit does not apply in causes of action which do
not seek to impose a charge or financial liability against the State.

The general rule that a state may not be sued without its consent, now embodied in Section 3, Article XVI
of the 1987 Constitution, is one of the generally accepted principles of international law, which we have
now adopted as part of the law of the land. While the doctrine of state immunity appears to prohibit only
suits against the state without its consent, it is also applicable to complaints filed against officials of the
state for acts allegedly performed by them in the discharge of their duties. The suit is regarded as one
against the state where the satisfaction of the judgment against the officials will require the state itself to
perform a positive act, such as the appropriation of the amount necessary to pay the damages awarded
against them. Hence, the rule does not apply where the public official is charged in his official capacity for
acts that are unauthorized or unlawful and injurious to the rights of others. Neither does it apply where the
public official is clearly being sued not in his official capacity but in his personal capacity, although the
acts complained of may have been committed while he occupied a public position.

REPUBLIC VS, PURISIMA


Digested by: Jadz Felix Alicer
TOPIC: Consent to be sued: Express Consent
Full Text: https://lawphil.net/judjuris/juri1977/aug1977/gr_36084_1977.html
FACTS:
The jurisdictional issued raised by Solicitor General Estelito P. Mendoza on behalf of
the Republic of the Philippines in this certiorari and prohibition proceeding arose from
the failure of respondent Judge Amante P. Purisima of the Court of First Instance of
Manila to apply the well-known and of-reiterated doctrine of the non-suability of a
State, including its offices and agencies, from suit without its consent. It was so
alleged in a motion to dismiss filed by defendant Rice and Corn Administration in a
pending civil suit in the sala of respondent Judge for the collection of a money claim
arising from an alleged breach of contract, the plaintiff being private respondent Yellow
Ball Freight Lines, Inc.

A motion to dismiss was filed on September 7, 1972.

The position of the Republic has been fortified with the explicit affirmation found in this
provision of the present Constitution: "The State may not be sued without its consent."

ISSUE: Whether or not an agreement between the Rice and Corn Administration and
Yellow Ball Freight Lines, Inc. operate as a waiver of the national government from
suit?

Ruling: NO. "The doctrine of non-suability recognized in this jurisdiction even prior to
the effectivity of the [1935] Constitution is a logical corollary of the positivist concept of
law which, to para-phrase Holmes, negates the assertion of any legal right as against
the state, in itself the source of the law on which such a right may be predicated. Nor
is this all. Even if such a principle does give rise to problems, considering the vastly
expanded role of government enabling it to engage in business pursuits to promote
the general welfare, it is not obeisance to the analytical school of thought alone that
calls for its continued applicability.

The consent, to be effective though, must come from the State acting through a duly
enacted statute as pointed out by Justice Bengzon in Mobil. Thus, whatever counsel
for defendant Rice and Corn Administration agreed to had no binding force on the
government. That was clearly beyond the scope of his authority.

WHEREFORE, the petitioner for certiorari is granted and the resolution of October 4,
1972 denying the motion to dismiss filed by the Rice and Corn Administration nullified
and set aside and the petitioner for prohibition is likewise granted restraining
respondent Judge from acting on civil Case No. 79082 pending in his sala except for
the purpose of ordering its dismissal for lack of jurisdiction. The temporary restraining
order issued on February 8, 1973 by this Court is made permanent terminating this
case. Costs against Yellow Ball Freight Lines, Inc.
DA VS. NLRC
RALF VINCENT BAJO

Full text: https://lawphil.net/judjuris/juri1993/nov1993/gr_104269_1993.html

TOPIC: Consent to be Sued; Express and Implied Consent / Non-suability of the State

FACTS: The Department of Agriculture (DA) and Sultan Security Agency (SSA)
entered into a contract for security services to be provided by the latter to the said
governmental entity. Thereafter, several guards of the SSA filed a complaint for
underpayment of wages, non-payment of 13th month pay, uniform allowances, night
shift differential pay, holiday pay and overtime pay, as well as for damages, before the
NLRC against the DA and SSA.

The Labor Arbiter rendered a decision finding the petitioner jointly and severally liable
with SSA for the payment of money claims of the complainant security guards.
Afterwards, a writ of execution was issued against certain properties of the two
respondents to be sold at a public auction to raise funds for the money claims.

In this petition for certiorari, the DA charges the NLRC with grave abuse of discretion
for refusing to quash the writ of execution. The DA claims that the NLRC has no
jurisdiction over a money claim against the Department, which falls under the
exclusive jurisdiction of the Commission on Audit; and that the NLRC has disregarded
the cardinal rule on the non-suability of the State.

The private respondents, on the other hand, argue that the petitioner has impliedly
waived its immunity from suit by concluding a service contract with SSA.

ISSUE: Whether or not the State, acting through the DA, may be sued in the case at
bar.

RULING: NO. The State’s consent to be sued may be given expressly or impliedly.

Express consent may be made through a general law or a special law. In this
jurisdiction, the general law waiving immunity is found in Act No. 3083, which gives the
consent of the State to be "sued upon any moneyed claim involving liability arising
from contract, express or implied”. Pursuant, however, to Commonwealth Act ("C.A.")
No. 327, as amended by Presidential Decree ("P.D.") No. 1145, the money claim
must first be brought to the Commission on Audit. The Labor code, in relation to
Act No. 3083, provides the legal basis for the State liability, but the prosecution,
enforcement or satisfaction thereof must still be pursued in accordance with the rules
and procedures laid down in C.A. No. 327 as amended by P.D. 1445, as a limitation to
the general law.
Implied consent, on the other hand, is conceded when the State itself commences
litigation, thus opening itself to a counterclaim or when it enters into a contract. In
this situation, the government is deemed to have descended to the level of the other
contracting party and to have divested itself of its sovereign immunity. This rule, relied
upon by the NLRC and the private respondents, is not, however, without qualification.
Not all contracts entered into by the government operate as a waiver of its
non-suability; distinction must still be made between one which is executed in the
exercise of its sovereign function and another which is done in its proprietary
capacity. In the instant case, the DA has not pretended to have assumed a capacity
apart from its being a governmental entity when it entered into the questioned
contract; nor that it could have, in fact, performed any act proprietary in character.

DECISION: The petition is GRANTED. The writ of execution against DA is nullified.


SAYSON VS. SINGSON
KRISTINA DANICA BALANAY

TOPIC: The basic doctrine of non-suability of the government without its consent is
thus decisive of the controversy

FACTS:
On January 1967, the Office of the District Engineer requisitioned various items of
spare parts for the repair of a D-8 bulldozer. The requisition was signed the District
Engineer, Adventor Fernandez, and the Requisitioning Officer, Manuel Lepatan. It was
approved by the Secretary of Public Works and Communications, Antonio Raquia. A
public bidding was conducted on May 1967 and the Committee on Award accepted
the bid of the Singkier Motor Service which was owned by respondent Felipe Singson
for the sum of Php 43, 530.00. It was approved by the Secretary of Public Works and
Communications.

Lorenzo Sayson received the purchase order and finding that the prize was
reasonable, he approved the payment of the voucher but held 2-% of the price,
equivalent to Php 8, 706.00 until supporting documents for the transaction was
submitted to the Supervising Auditor for Review. After the documents were submitted,
the General Auditing Office found the transaction to be excessive price by at least Php
40,000.00 based on the canvass made among suppliers in Manila. Malversation
charges were filed against the District Engineer and the Civil Engineer involved. A
mandamus suit was filed by the respondent, Singson as sole proprietor of Singkier
Motor Service, being adjudged as entitled to collect the balance of Php 8,706.00, the
contract in question having been upheld. Hence this appeal by certiorari.

ISSUE: Whether or not the respondent is entitled to collect through a mandamus suit
filed against the petitioners.

RULING: No. It is apparent that the respondent’s cause of action is a money claim
against the Government for the payment of the alleged balance of cost for spare parts
supplied by him. Given that the claim is valid, a mandamus suit is not the remedy to
enforce the collection of such claim against the State but an ordinary action for
specific performance. The suit disguised as a mandamus suit cannot be entertained
without the consent of the State.

The respondent should have filed his claim with the General Auditing Office, under the
provisions of Commonwealth Act 327 which prescribes the conditions under which
money claims against the government may be filed. It is provided in the said act that
all claims on settlements shall be decided within 60 days by the Auditor General.
Furthermore, appeals on decision must be submitted in writing within 30 days from the
receipt of such decision to the President of the USA or the President of the Philippines
or the SC of the Philippines if the appellant is a private person or entity. Thus, the suit
for mandamus filed against petitioners is dismissed.
MERRITT VS, GOVERNMENT OF THE PHILIPPINE
ISLANDS
Sophia Amor C. Bersamin

Full Text: https://lawphil.net/judjuris/juri1916/mar1916/gr_l-11154_1916.html

TOPIC: Consent to be Sued (Express consent)

FACTS:

Petitioner was riding a motorcycle along Padre Faura Street when he was bumped by the
ambulance of the General Hospital. Petitioner sustained severe injuries rendering him unable
to return to work.

The legislature later enacted Act No. 2457 authorizing Merritt to file a suit against the
Government in order to fix the responsibility for the collision between his motorcycle and the
ambulance of the General Hospital, and to determine the amount of the damages, if any, to
which he is entitled.

After trial, the lower court held that the collision was due to the negligence of the driver of the
ambulance. It then determined the amount of damages and ordered the government to pay the
same.

ISSUE:

W/N the Government has expressly made itself legally-liable for the amount of damages to the
petitioner under Act. No 2457.

RULING:

NO. The Government of the Philippine Islands having been "modeled after the Federal and
State Governments in the United States," may look to the decisions of the high courts of that
country for aid in determining the purpose and scope of Act No. 2457. In the United States the
rule that the state is not liable for the torts committed by its officers or agents whom it employs,
except when expressly made so by legislative enactment is well settled.

The responsibility of the state is limited by article 1903 to the case wherein it acts through a
special agent (and a special agent, in the sense in which these words are employed, is one
who receives a definite and fixed order or commission, foreign to the exercise of the duties of
his office if he is a special official) so that in representation of the state and being bound to act
as an agent thereof he executed the trust confided to him. This concept does not apply to any
executive agent who is an employee of the active administration and who in his own
responsibility performs the functions which are inherent in and naturally pertain to his office and
which are regulated by law and the regulations.”

It is, therefore, evident that the State (the Government of the Philippine Islands) is only
liable for the acts of its agents, officers and employees when they act as special agents
within the meaning of paragraph 5 of article 1903, and that the chauffeur of the
ambulance of the General Hospital was not such an agent.

The court ruled that for the foregoing reasons, the judgment appealed from must be reversed,
without costs in this instance. Whether the Government intends to make itself legally liable for
the amount of damages above set forth, which the plaintiff has sustained by reason of the
negligent acts of one of its employees, by legislative enactment and by appropriating sufficient
funds therefore, the courts are not called upon to determine. This matter rests solely with the
Legislature and not with the courts.

NHA VS. HEIRS OF GUIVELONDO


Hanna Lyka Bontilao

TOPIC – Sovereign Immunity – Consent to be Sued – Express Consent

FACTS – Petitioner NHA filed an Amended Complaint for eminent domain against the
respondents alleging that their land in Carreta, Mabolo, Cebu City is within a blighted
urban center, which petitioner intends to develop as a socialized housing project.

The Heirs of Guivelondo filed a Manifestation stating that they were waiving their
objections to petitioner’s power to expropriate their properties. Hence, the trial court issued
an Order declaring that the Petitioner has a lawful right to expropriate the properties of the
respondents.

On August 7, 2000, the trial court rendered Partial Judgment fixing the just compensation
of the lands of respondent. In response, NHA filed appeals and petitions for review saying
that it did not want the property anymore. Petitioner alleges that the intended public use
was rendered nugatory by the unreasonable just compensation fixed by the court, which is
beyond the means of the intended beneficiaries of the socialized housing project.

Respondent Sheriff Pascual Abordo served on NHA a Notice of Levy pursuant to the Writ
of Execution issued by the trial court to enforce the Partial Judgment of August 7, 2000, as
well as a Notice of Third Garnishment against the deposits, moneys and interests of
petitioner therein.

ISSUE – Whether or not the Doctrine of State Immunity applies such that the funds and
properties of the NHA are exempt from garnishment.

RULING – No. Generally, funds and properties of the government cannot be the object of
garnishment proceedings even if the consent to be sued had been previously granted and
the state liability adjudged.

The universal rule that where the State gives its consent to be sued by private parties
either by general or special law, it may limit claimant’s action "only up to the completion of
proceedings anterior to the stage of execution" and that the power of the Courts ends
when the judgment is rendered, since government funds and properties may not be seized
under writs of execution or garnishment to satisfy such judgments, is based on obvious
considerations of public policy. Disbursements of public funds must be covered by the
corresponding appropriation as required by law. The functions and public services
rendered by the State cannot be allowed to be paralyzed or disrupted by the diversion of
public funds from their legitimate and specific objects, as appropriated by law.

However, if the funds belong to a public corporation or a government-owned or


controlled corporation which is clothed with a personality of its own, separate and
distinct from that of the government, then its funds are not exempt from
garnishment. This is so because when the government enters into commercial business,
it abandons its sovereign capacity and is to be treated like any other corporation. Hence,
the funds of petitioner NHA are not exempt from garnishment or execution.

Juco v. NLRC

By Eden Maagad

Facts

Benjamin C. Juco was hired as a project engineer of National Housing Corporation (NHC)
from November 16, 1970 to May 14, 1975. On May 14, 1975, he was separated from the
service for having been implicated in a crime of theft and/or ma­lversation of public funds. On
March 25, 1977, Juco filed a complaint for illegal dismissal against the NHC with the
Department of Labor. On September 17,1977, the Labor Arbiter rendered a decision dismissing
the complaint on the ground that the NLRC had no jurisdiction over the case. Juco then
elevated the case to the NLRC which rendered a decision on December 28 1982, reversing the
decision of the Labor Arbiter. NHC the appealed the NLRC decision before the Supreme court
granted thereby setting aside the NLRC decision and reinstating the labor arbiter’s decision of
dismissing the case.

On January 6, 1989, Juco filed with the civil service commission a complaint for illegal
dismissal, with preliminary mandatory injunction. On February 6, 1989, NHC moved for the
dismissal of the complaint on the ground that the civil service commission has no jurisdiction
over the case. CSC granted the motion to dismiss on the ground of lack of jurisdiction.

On April 28, 1989, Juco filed with NLRC a complaint for illegal dismissal with preliminary
mandatory injunction against NHC. NLRC find NHC guilty of illegal dismissal. On June 1,
1990, NHC filed its appeal before the NLRC and on March 14, 1991, the NLRC promulgated a
decision which reversed the decision of labor arbiter Manuel Caday on the ground of lack of
jurisdiction.

Issue
Whether or not the NLRC committed grave abuse of discretion in holding that petitioner is not
governed by the Labor Code

Held

Yes. Under the laws then in force, employees of government-owned and/or controlled
corporations were governed by the Civil Service Law and not by the Labor Code. Although in
National Housing Corporation v. Juco, it was held that employees of government-owned and/or
controlled corporations, whether created by special law or formed as subsidiaries under the
general Corporation Law, are governed by the Civil Service Law and not by the Labor Code,
this ruling has been supplanted by the 1987 Constitution which states that the civil service
embraces all branches, subdivisions, instrumentalities, and agencies of the Government,
including government owned or controlled corporations with original charter.

UNITED STATES VS. RUIZ


CRUZ, SOPHIA DENISSE L.
TOPIC: Implied consent
FACTS: Petitioner invited the submission of bids for repair of its wharves and
shoreline in the Subic Bay Area. Eligion and Co. responded to the invitation and
submitted bids. Said company was requested by telegram to confirm its price
proposals and for the name of its bonding company, and from which it complied.

Later, the United States, through its agents, informed said company that it was not
qualified to receive an award at the project for the poorly completed projects it
awarded to third parties. The company sued petitioner for specific performance and if
no longer possible, for damages. It also asked for a writ of preliminary injunction to
restrain the defendants from entering into contracts with others.

The United States entered a special appearance for the purpose only of questioning
the jurisdiction of the court over the subject matter of the complaint and the persons of
the defendants, the subject matter of the complaint being acts and omissions of the
individual defendants as agents of the defendant United States of America, a foreign
sovereign which has not given its consent to this suit or any other suit for the cause of
action asserted in the complaint.

ISSUE: WON US may be sued

RULING: NO.
The traditional rule of State immunity exempts a State from being sued in the courts of
another State without its consent or waiver. This rule is a necessary consequence of
the principles of independence and equality of States. However, the rules of
International Law are not petrified; they are constantly developing and evolving. And
because the activities of states have multiplied, it has been necessary to distinguish
them-between sovereign and governmental acts (jure imperii) and private,
commercial and proprietary acts (jure gestionis). The result is that State immunity
now extends only to acts jure imperil The restrictive application of State immunity is
now the rule in the United States, the United Kingdom and other states in western
Europe. (See Coquia and Defensor Santiago, Public International Law, pp. 207-209
[1984].)

The restrictive application of State immunity is proper only when the proceedings arise
out of commercial transactions of the foreign sovereign, its commercial activities or
economic affairs. Stated differently, a State may be said to have descended to the
level of an individual and can thus be deemed to have tacitly given its consent to be
sued only when it enters into business contracts. It does not apply where the contract
relates to the exercise of its sovereign functions. In this case the projects are an
integral part of the naval base which is devoted to the defense of both the United
States and the Philippines, indisputably a function of the government of the highest
order; they are not utilized for nor dedicated to commercial or business purposes.

That the correct test for the application of State immunity is not the conclusion of a
contract by a State but the legal nature of the act is shown in Syquia vs. Lopez, 84
Phil. 312 (1949). In that case the plaintiffs leased three apartment buildings to the
United States of America for the use of its military officials. The plaintiffs sued to
recover possession of the premises on the ground that the term of the leases had
expired. They also asked for increased rentals until the apartments shall have been
vacated.
MALONG VS. PNR
G.R. No. L-49930
Aimee E. Demontano
TOPIC:
Consent to be sued: implied consent

FACTS:
This case is about the immunity from suit of the Philippine National Railways (PNR).

The Malong spouses alleged in their complaint that their son, Jaime Aquino, a paying
passenger, was killed when he fell from a PNR train while it was between Tarlac and
Capas.

The tragedy occurred because Jaime had to sit near the door of a coach. The train
was overloaded with passengers and baggage in view of the proximity of All Saints
Day. The Malong spouses prayed that the PNR be ordered to pay them damages
totalling P136,370.

The trial court dismissed the complaint because it has no jurisdiction because the
PNR, being a government instrumentality, the action was a suit against the State (Sec.
16, Art. XV of the Constitution). Hence, the Malong spouses appealed to this Court
pursuant to Republic Act No. 5440.

ISSUE:
Whether or not the PNR can be sued when the State divested itself of its sovereign
capacity

RULING: YES
When the government enters into a commercial business it abandons its sovereign
capacity and is to be treated like any other private corporation. It can be sued like any
other persons.

It was held that when the state organized the Philippine National Railway, it divested
itself of its sovereign capacity, and so became liable for damages that arose from the
death of one who fell from an overloaded train.

It would be unjust if the heirs of the victim of an alleged negligence of the PNR
employees could not sue the PNR for damages. Like any private common carrier, the
PNR is subject to the obligations of persons engaged in that private enterprise. It is
not performing any governmental function.
G.R. No. 76607 February 26, 1990
United States Vs. Hon. Eliodoro Guinto et al.

Philippines as a State
Implied Consent

Digested By: Rynn Judd C. Escaño

Facts:

The private respondents are suing several officers of the US Air Force in Clark Air
base in connection with a bidding conducted for a contract of barber services within the
base. Among those who submitted their bid were the respondents Roberto T. Valencia,
Emerenciana C. Tanglao, and Pablo C. del Pilar.

The winning bid was submitted by Ramon Dizon who bid for four facilities, including
the Civil Engineering Area, which was not included in the invitation to bid.

The private respondents filed a complaint to compel Philippine Area Exchange


(PHAX) and the individual petitioners to cancel the award to Dizon, to conduct a
rebidding for the barber services contract and to allow the private respondents by a writ
of preliminary injunction to continue operation the concessions pending litigation.

The petitioners filed a motion to dismiss and opposition to the petition for preliminary
injunction on the ground that the action was in effect a suit against the USA, which did
not waive its non-suability; and that the defendants, as official/employee of the U.S air
Force were also immune from suit. The trial court denied the petitioners’ motion to
dismiss.

Issue:

W/N the petitioners are immune from suit.

Ruling:

No. The state may manifest its consent to be sued, expressly or implied. Express
consent may be embodied in a general law or a special law. Consent is implied when
the state enters a contract or it itself commences litigation.

When the government enters a contract, it is deemed to have descended to the level of
the other contracting party and divested of its sovereign immunity from suit with its
implied consent. Waiver is also implied when the government files a complaint, thus
opening itself to a counterclaim.
DEPED VS. OÑATE
RYAN JUANITO GARCIA
TOPIC: Suability of State

FACTS:

Claro Oñate and Gregoria Los Baños owned Lot No. 6849 (disputed lot) with an area
of around 27,907 square meters. Sometime in 1940, Bagumbayan Elementary School of
Daraga was constructed on a portion of the disputed lot. The school was eventually renamed
Daraga North Central Elementary School. The Municipality of Daraga leveled the area while
petitioner Department of Education Culture and Sports (DECS) developed and built various
school buildings and facilities on the disputed lot. On December 15, 1992, through his
counsel, respondent sent a letter to petitioner apprising it about the facts and circumstances
affecting the elementary school and its occupancy. Respondents reported about the on-going
construction projects in the school to the district engineer. The engineer then informed
respondents counsel that petitioner DECS is the owner of the school site having acquired the
disputed lot by virtue of a Deed of Donation executed by the Municipality of Daraga, Albay in
favor of the petitioner.

On March 18, 1993, respondent instituted a Complaint for Annulment of Donation


and/or Quieting of Title with Recovery of Possession of Lot No. 6849 before the Legaspi City
RTC against petitioner DECS, Division of Albay and the Municipality of Daraga, Albay.

The Municipality of Daraga, Albay denied respondents ownership of the disputed lots
it alleged that sometime in 1940, the Municipality bought said lot from Claro Oate,
respondents’ grandfather, and since then it had continually occupied said lot openly and
publicly in the concept of an owner until 1988 when the Municipality donated the school site
to petitioner DECS; thus asserting that it could also claim ownership also through adverse
possession. Moreover, it claimed that the disputed lot had been declared in the name of
defendant municipality in the Municipal Assessors Office under Tax Declaration No. 31954
from 1940 until 1988 for purposes of exemption from real estate taxes. Further, defendant
Municipality contended that respondent was guilty of laches and was estopped from assailing
ownership over the disputed lot.

* DECS; now Department of Education [DepEd]

ISSUE:
Whether or not petitioner may be sued in violation of the state's immunity from suit.

RULING:
Yes, DECS can be sued without its permission.

That the petitioner DECS, as a result of its being privy to the Deed of Donation executed by
the Municipality of Daraga, Albay over the disputed property. When it voluntarily gave its
consent to the donation, any dispute that may arise from it would necessarily bring petitioner
DECS down to the level of an ordinary citizen of the State vulnerable to a suit by an interested
or affected party. It has shed off its mantle of immunity and relinquished and forfeited its
armor of non-suability of the State.
PTA VS. PGDEI
Digested by: Jadz Felix Alicer
TOPIC: Consent to be sued: Implied Consent
Full Text: https://lawphil.net/judjuris/juri2012/mar2012/gr_176628_2012.html
FACTS:
On April 3, 1996, Philippine Tourism Authority (PTA), an agency of the Department
of Tourism, whose main function is to bolster and promote tourism, entered into a
contract with Atlantic Erectors, Inc. (AEI) for the construction of the Intramuros Golf
Course Expansion Projects for a contract price of about P57M.

The civil works of the project commenced. Since AEI was incapable of constructing
the golf course aspect of the project, it entered into a sub-contract agreement with
PHILGOLF, a duly organized domestic corporation, to build the golf course amounting
to P27M. The sub-contract agreement also provides that PHILGOLF shall submit its
progress billings directly to PTA and, in turn, PTA shall directly pay PHILGOLF.

On October 2, 2003, PHILGOLF filed a collection suit against PTA amounting to


Eleven Million Eight Hundred Twenty Thousand Five Hundred Fifty and 53 centavos
(₱11,820,550.53) for the construction of the golf course.

Since PTA failed to file its answer, RTC rendered a judgment of default, ordering PTA
to pay PHILGOLF. PHILGOLF then filed a motion for execution.

PTA then invoked the defense of state immunity since it is a government entity

ISSUE: Whether or not the Philippine Tourism Authority (PTA) is immune from suit.

RULING:

NO. PTA also erred in invoking state immunity simply because it is a government
entity. The application of state immunity is proper only when the proceedings arise out
of sovereign transactions and not in cases of commercial activities or economic affairs.
The State, in entering into a business contract, descends to the level of an individual
and is deemed to have tacitly given its consent to be sued.

Since the Intramuros Golf Course Expansion Projects partakes of a proprietary


character entered into between PTA and PHILGOLF, PTA cannot avoid its financial
liability by merely invoking immunity from suit.
EPG CONSTRUCTION CO. VS. VIGILAR
NAME OF DIGESTER
TOPIC: SOVEREIGNTY - CONSENT - IMPLIED CONSENT
Facts:

● In 1983, the Ministry of Human Settlement (MHS), through the BLISS Development
Corporation, intiated a housing project on a government property along the east bank of
Manggahan Floodway in Pasig
● The MHS entered into a Memorandum of Agreement (MOA) with Ministry of Public Works and
Highways (MPWH) where the latter undertook to develop the housing site and construct
thereon 145 housing units
● By virtue of the MOA, MPWH forged individual contracts with petitioners EPG, Ciper, Septa,
Phil. Plumbing, Home Construction, World Builders, Glass World, Performance Builders, and De
Leon Araneta Construction for the construction of the housing units
● Under the contracts, the scope of construction and funding covered only around "2/3 of each
housing unit" Petitioners agreed to undertake and perform "additional constructions" for the
completion of the housing units despite the fact that there was only a verbal promise, and not
a written contract, by the MPWH Undersecretary Aber Canlas that additional funds will be
available and forthcoming
● Unpaid balance for the additional constructions amounted to P5,918,315.63
● Upon a demand letter from the petitioners, on November 14, 1988, DPWH Asst. Secretary
Madamba opined that payment of petitioners' money claims should be based on quantum
meruit (what one has earned) and should be forwarded to the Commission on Audit (COA)
● In a Letter of the Undersecretary of Budget and Management dated December 20, 1994, the
amount of P5,819,316.00 was then released for the payment of the petitioners' money claims
under Advise of Allotment No. A4-1303-04-41-303
● In an indorsement dated December 27, 1995, the COA referred anew the money claims to the
DPWH
● In a letter dated August 26, 1996, respondent Secretary Gregorio Vigilar denied the subject
money claims
Issue:
1. Whether or not the implied, verbal contracts between the petitioners and then
Undersecretary Canlas should be upheld
2. Whether or not the State is immune from suit
Ratio:
1. YES. While the court agrees with the respondent that the implied contracts are void, in view of
violation of applicable laws, auditing rules, and lack of legal requirements, it still finds merit in
the instant petition
● The illegality of the implied contracts proceeds from an express declaration or prohibition by
law, not from any intrinsic illegality
● "in the interest of substantial justice," petitioners-contractors' right to be compensated is
upheld, applying the principle of quantum meruit
● Even the DPWH Asst. Sec. for Legal Affairs recommends their compensation; even the DPWH
Auditor did not object to the payment of the money claims
2. NO. The respondent may not conveniently hide under the State's cloak of invincibility against suit,
considering that this principle yields to certain settled exceptions.

● The State's immunity cannot serve as an instrument perpetrating injustice


Petition granted. RTC decision reversed and set aside.
REPUBLIC VS. UNIMEX
LIM, MA. THERESA
TOPIC: Implied Consent
FACTS:
-April 1985, respondent Unimex Micro-Electronics GmBH (Unimex) shipped a 40-foot container and 171
cartons of Atari game computer cartridges, duplicators, expanders, remote controllers, parts and
accessories to Handyware Phils., Inc. (Handyware). Don Tim Shipping Corporation transported the goods
with Evergreen Marine Corporation as shipping agent.
-After the shipment arrived in the Port of Manila on July 9, 1985, the Bureau of Customs (BOC) agents
discovered that it did not tally with the description appearing on the cargo manifest. As a result, BOC
instituted seizure proceedings against Handyware and later issued a warrant of seizure and detention
against the shipment.
-On June 5, 1987, the Collector of Customs issued a default order against Handyware for failing to
appear in the seizure proceedings. After an ex parte hearing, the Collector of Customs forfeited the goods
in favor of the government.
-Subsequently, on June 15, 1987, respondent Unimex (as shipper and owner of the goods) filed a motion
to intervene in the seizure proceedings. The Collector of Customs granted the motion but later on
declared the June 5, 1987 default order against Handyware as final and executory, thus affirming the
goods’ forfeiture in favor of the government.
-June 15, 1992, the Court of Tax Appeals reversed the forfeiture decree and ordered the release of the
subject shipment to respondent subject to the payment of customs duties. The CTA decision became final
and executory on July 20, 1992.
-Unfortunately, respondent’s counsel failed to secure a writ of execution to enforce the CTA decision. -On
September 5, 2001, respondent filed in the CTA a petition for the revival of its June 15, 1992 decision. It
prayed for the immediate release by BOC of its shipment or, in the alternative, payment of the shipment’s
value plus damages. The BOC Commissioner failed to file his answer, hence, he was declared in default.
-During the ex parte presentation of respondent’s evidence, BOC informed the court that the subject
shipment could no longer be found at its warehouses.
-In its decision of September 19, 2002, the CTA declared that its June 15, 1992 decision could no longer
be executed due to the loss of respondent’s shipment so it ordered the BOC Commissioner to pay
respondent the commercial value of the goods based on the prevailing exchange rate at the time of their
importation.
-Petitioner argues that a money judgment or any charge against the government requires a
corresponding appropriation and cannot be decreed by mere judicial order.
ISSUE: Whether or not petitioner, the State, may be compelled to comply with the money judgement.
RULING:
The general rule that once a decision becomes final and executory, it cannot be altered or modified is not
absolute. In some cases, we held that where facts or events transpire after a decision has become
executory, which facts constitute a supervening cause rendering the final judgment unenforceable, said
judgment may be modified. Also, a final judgment may be altered when its execution becomes impossible
or unjust. In the case at bar, parties do not dispute the fact that after the June 15, 1992 CTA decision
became final and executory, respondent’s goods were inexplicably lost while under the BOC’s custody.
Certainly, this fact presented a supervening event warranting the modification of the CTA decision. Even if
the CTA had maintained its original decision, still petitioner would have been unable to comply with it for
the obvious reason that there was nothing more to deliver to respondent.

Although it may be said that the satisfaction of respondent’s demand will ultimately fall on the
government, and that, under the political doctrine of "state immunity," it cannot be held liable for
governmental acts (jus imperii), we still hold that petitioner cannot escape its liability. The circumstances
of this case warrant its exclusion from the purview of the state immunity doctrine.
As previously discussed, the Court cannot turn a blind eye to BOC’s ineptitude and gross negligence in
the safekeeping of respondent’s goods. We are not likewise unaware of its lackadaisical attitude in failing
to provide a cogent explanation on the goods’ disappearance, considering that they were in its custody
and that they were in fact the subject of litigation. The situation does not allow us to reject respondent’s
claim on the mere invocation of the doctrine of state immunity. Succinctly, the doctrine must be fairly
observed and the State should not avail itself of this prerogative to take undue advantage of parties that
may have legitimate claims against it.

In Department of Health v. C.V. Canchela & Associates, we enunciated that this Court, as the staunch
guardian of the people’s rights and welfare, cannot sanction an injustice so patent in its face, and allow
itself to be an instrument in the perpetration thereof. Over time, courts have recognized with almost
pedantic adherence that what is inconvenient and contrary to reason is not allowed in law. Justice and
equity now demand that the State’s cloak of invincibility against suit and liability be shredded.
FROILAN VS. ORIENTAL PAN SHIPPING: GR L-6060
DIGESTER: ADRIAN WILLIAM J. LO
TOPIC: SOVEREIGNTY - Consent to be sued - implied consent

FACTS:

- Fernando Froilan buys a ship (FD-197) from the Shipping Commission worth P200K.
He gives P50K as down.

- For various reasons, including non-payment of installments, the Commission takes


possession of the vessel.

- Considering the contract of sale canceled, the Commission charters and delivers the
vessel to Oriental Pan Shipping. Froilan wants it back.

- Lower court issues a writ of replevin prayed for by Froilan. Oriental Pan Shipping is
divested of its possession of the ship.

- Government of the Republic of the Philippines, files a complaint in intervention


alleging that Froilan had failed to pay to the Shipping Commission the balance due on
the purchase price of the vessel in question, that intervenor was entitled to the
possession of the said vessel either under the terms of the original contract or in order
that it may cause the extrajudicial sale thereof under the Chattel Mortgage Law.

ISSUE: WoN the Government of the Philippines is immune from suit in this case.

RULING: No, it is not immune. It gave its IMPLIED CONSENT to be sued when it filed
a complaint in intervention.

By filing its complaint in intervention the Government in effect waived its right of
non-suability.

FULLTEXT:
https://lawphil.net/judjuris/juri1954/sep1954/gr_l-6060_1954.html#:~:text=Froilan
%2C%20filed%20a%20complaint%20against,the%20purchase%20price%2C%20
he%20executed
THE COMMISSIONER OF PUBLIC HIGHWAYS VS. SAN
DIEGO
NAME OF DIGESTER: Darriel Kier Loreque

Facts: Before WWII, the Philippine Government filed an action for the
expropriation of a parcel of land owned by Hashim for the construction of a
public road. Government took possession over the property after the deposit of
the amount of P23, 413.64. Records of the case were destroyed during the
WWII. After the war, Hashim filed an action for money claims before the CFI
against Bureau of Public Highways.

The parties entered into a compromise agreement wherein the Bureau shall
pay almost half of the amount claimed. The bureau failed to pay so Hashim
filed a motion for the issuance of a writ of execution. Respondent judge
granted the motion. The sheriff served the writ with a Notice of Garnishment to
PNB against the Bureau's funds. Hashim further filed a motion for issuance of
an order ordering the release of the amount. It was granted. PNB released the
amount. Petitioner filed this petition for certiorari

Issue: W/N the Bureau’s funds can be subject of garnishment.

Held: No.

Although the government, as plaintiff in expropriation proceedings, submits


itself to the jurisdiction of the court and thereby waives its immunity from suit,
the judgment that is thus rendered requiring its payment of the award
determined as just compensation for the condemned property as a condition
precedent to the transfer to the title thereto in its favor, cannot be realized upon
execution. The legislature must first appropriate the amount, over and above
the provisional deposit, that may be necessary to pay the award determined in
the judgment, since the government cannot keep the land and dishonor the
judgment.

All government funds deposited with PNB by any agency or instrumentality of


the government, whether by way of general or special deposit, remain
government funds, since such government agencies or instrumentalities do not
have any nonpublic or private funds of their own. They are not subject to
garnishment or levy; even assuming that the funds become commingled with
other funds of the bank, this does not remove the character of the funds as a
credit representing government funds thus deposited.
The universal rule that where the State gives its consent to be sued by
private parties either by general or special law, it may limit claimant’s action
“only up to the completion of proceedings anterior to the stage of execution”
and that the power of the Courts ends when the judgment is rendered, since
government funds and properties may not be seized under writs of
execution or garnishment to satisfy such judgments, is based on obvious
considerations of public policy.

Disbursements of public funds must be covered by the corresponding


appropriation as required by law. The functions and public services rendered
by the State cannot be allowed to be paralyzed or disrupted by the diversion of
public funds from their legitimate and specific objects, as appropriated by law.
Juco v. NLRC
By Eden Maagad

Facts

Benjamin C. Juco was hired as a project engineer of National Housing Corporation (NHC)
from November 16, 1970 to May 14, 1975. On May 14, 1975, he was separated from the
service for having been implicated in a crime of theft and/or ma­lversation of public funds. On
March 25, 1977, Juco filed a complaint for illegal dismissal against the NHC with the
Department of Labor. On September 17,1977, the Labor Arbiter rendered a decision dismissing
the complaint on the ground that the NLRC had no jurisdiction over the case. Juco then
elevated the case to the NLRC which rendered a decision on December 28 1982, reversing the
decision of the Labor Arbiter. NHC the appealed the NLRC decision before the Supreme court
granted thereby setting aside the NLRC decision and reinstating the labor arbiter’s decision of
dismissing the case.

On January 6, 1989, Juco filed with the civil service commission a complaint for illegal
dismissal, with preliminary mandatory injunction. On February 6, 1989, NHC moved for the
dismissal of the complaint on the ground that the civil service commission has no jurisdiction
over the case. CSC granted the motion to dismiss on the ground of lack of jurisdiction.

On April 28, 1989, Juco filed with NLRC a complaint for illegal dismissal with preliminary
mandatory injunction against NHC. NLRC find NHC guilty of illegal dismissal. On June 1,
1990, NHC filed its appeal before the NLRC and on March 14, 1991, the NLRC promulgated a
decision which reversed the decision of labor arbiter Manuel Caday on the ground of lack of
jurisdiction.

Issue

Whether or not the NLRC committed grave abuse of discretion in holding that petitioner is not
governed by the Labor Code

Held

Yes. Under the laws then in force, employees of government-owned and/or controlled
corporations were governed by the Civil Service Law and not by the Labor Code. Although in
National Housing Corporation v. Juco, it was held that employees of government-owned and/or
controlled corporations, whether created by special law or formed as subsidiaries under the
general Corporation Law, are governed by the Civil Service Law and not by the Labor Code,
this ruling has been supplanted by the 1987 Constitution which states that the civil service
embraces all branches, subdivisions, instrumentalities, and agencies of the Government,
including government owned or controlled corporations with original charter.
PNB VS. CIR
Digested By: Alyana Mahilum
TOPIC: Scope of consent

FACTS:

What was sought to be garnished was the money of the People’s Homesite and Housing
Corporation deposited at petitioner’s branch in Quezon City, to satisfy a decision of respondent
Court which had become final and executory. A writ of execution in favor of private respondent
Gabriel V. Manansala had previously been issued. He was the counsel of the prevailing party,
the United Homesite Employees and Laborers Association, in the aforementioned case. The
validity of the order assailed is challenged on the ground that the funds subject of the
garnishment “may be public in character.” In thus denying the motion to quash, petitioner
contended that there was on the part of respondent Court a failure to abide by authoritative
doctrines amounting to a grave abuse of discretion.

ISSUE:

Are the funds of People’s Homesite and Housing Corporation exempt from garnishment?

RULING:

No. The premise that the funds could be spoken of as public in character may be accepted in
the sense that the People’s Homesite and Housing Corporation was a government-owned
entity. It does not follow though that they were exempt from garnishment.

It is worth noting that the decision referred to, the Bank of the United States v. Planters’ Bank,
was promulgated by the American Supreme Court as early as 1824, the opinion being penned
by the great Chief Justice Marshall. As was pointed out by him: “It is, we think, a sound
principle, that when a government becomes a partner in any trading company, it divests itself,
so far as concerns the transactions of that company, of its sovereign character, and takes that
of a private citizen. Instead of communicating to the company its privileges and its
prerogatives, it descends to a level with those with whom it associates itself, and takes the
character which belongs to its associates, and to the business which is to be transacted. Thus,
many states of this Union who have an interest in banks, are not suable even in their own
courts; yet they never exempt the corporation from being sued. The state of Georgia, by giving
to the bank the capacity to sue and be sued, voluntarily strips itself of its sovereign character,
so far as respects the transactions of the bank, and waives the privileges of that character. As
a member of a corporation, a government never exercises its sovereignty. It acts merely as a
corporator, and exercises no other power in the management of the affairs of the corporation,
that are expressly given by the incorporating act
REPUBLIC VS. HIDALGO
Mondano, Aleric

TOPIC: Execution of the Scope of Consent to be Sued

FACTS:
Tarcila Laperal Mendoza filed an action for the annulment or declaration of nullity of
the title and deed of sale, reconveyance and/or recovery of ownership and possession
of a four thousand nine hundred twenty-four-square meter (4,924.60 sq. m. to be
exact) property against the Republic of the Philippines. It is also known as the Arlegui
Residence which housed two (2) Philippine presidents and which now holds the Office
of the Press Secretary and the News Information Bureau.

In an Order dated 07 July 2003, Judge Hidalgo declared the Republic in default for
failure of Solicitor Gabriel Francisco Ramirez, the handling solicitor, to file the required
Answer within the period prayed for in his motion for extension dated 21 May 2003.

Ordering the Republic of the Philippines to pay ONE BILLION FOUR HUNDRED EIGHTY
MILLION SIX HUNDRED TWENTY SEVEN THOUSAND SIX HUNDRED EIGHTY EIGHT
(₱1,480,627,688.00) PESOS.

ISSUE: Whether Judge Hidalgo violated the Constitution and the fundamental rule
that government funds are exempt

from execution or garnishment.

RULING: YES, he violated the fundamental rule. In the case at bar, respondent Judge
not only failed to perform his duties in accordance with the Rules, but he also acted
wilfully and in gross disregard of the law and controlling jurisprudence. He was
ignorant of the basic and simple procedural rules by issuing the writ of execution and
pronouncing the costs of suit against the government. Verily, respondent Judge’s
actions visibly indicate his lack of sufficient grasp of the law.

For issuing the writ of execution and pronouncing the costs of the suit against the
government, we deem that the respondent Judge is liable for gross ignorance of the
law or procedure under Rule 140 of the Rules of Court.

DOCTRINE:
It is settled that when the State gives its consent to be sued, it does not thereby
necessarily consent to an unrestrained execution against it. Tersely put, when the
State waives its immunity, all it does, in effect, is to give the other party an opportunity
to prove, if it can, that the state has a liability. In Republic v. Villasor22 this Court, in
nullifying the issuance of an alias writ of execution directed against the funds of the
Armed Forces of the Philippines to satisfy a final and executory judgment, has
explained, thus—

. . . The universal rule that where the State gives its consent to be sued by private
parties either by general or special law, it may limit claimant’s action "only up to the
completion of proceedings anterior to the stage of execution" and that the power of the
Courts ends when the judgment is rendered, since government funds and properties
may not be seized under writs of execution or garnishment to satisfy such judgments,
is based on obvious considerations of public policy. Disbursements of public funds
must be covered by the correspondent appropriation as required by law. The functions
and public services rendered by the State cannot be allowed to paralyzed or disrupted
by the diversion of public funds from their legitimate and specific objects, as
appropriated by law
THE MUNICIPALITY OF SAN FERNANDO, LA UNION VS.
FIRME
Moralizon, Trixia Ann P.

TOPIC: Sovereignty-> Consent to be sued-> Scope of consent

FACTS:

● This is a petition seeking the nullification or modification of the proceedings and the orders
issued by the respondent Judge Romeo N. Firme, ordering defendants Municipality of San
Fernando, La Union and Alfredo Bislig to pay, jointly and severally, the plaintiffs for funeral
expenses, actual damages consisting of the loss of earning capacity of the deceased, attorney's
fees and costs of suit and dismissing the complaint against the Estate of Macario Nieveras and
Bernardo Balagot in relation to the case of “Juana Rimando Baniña, et al. vs. Macario Nieveras,
et al.” Firme is impleaded in his official capacity as the presiding judge of the Court of First
Instance of La Union, Branch IV, Bauang, La Union.
● In the case of Juana Rimando Baniña, et al. vs. Macario Nieveras, et al., a passenger jeepney
owned by the Estate of Macario Nieveras and driven by Bernardo Balagot, a gravel and sand
truck driven by Jose Manandeg and owned by Tanquilino Velasquez and a dump truck of the
Municipality of San Fernando, La Union and driven by Alfredo Bislig.
● Four passengers sustained varying degrees of physical injuries while Laureano Baniña Sr. along
with other passengers died in the accident.
● Private respondents filed a complaint for damages against the owner and driver of the
passenger jeepney. In response, defendants also instituted a Third Party Complaint against the
petitioner, Municipality of San Fernando, La Union and Bislig.
● The case was subsequently transferred to Branch IV, presided over by the respondent judge and
was subsequently docketed as Civil Case No. 107-Bg. Subsequently, the private respondents
amended the complaint wherein the petitioner and Bislig were sued for the first time as
defendants.
● Petitioner filed its answer and raised affirmative defenses such as lack of cause of action,
non-suability of the State, prescription of cause of action and the negligence of the owner and
driver of the passenger jeepney as the proximate cause of the collision.
● Firme ordered defendants Municipality of San Fernando, La Union and Alfredo Bislig to pay,
jointly and severally, the plaintiffs. When the petitioner filed for a motion for reconsideration,
Firme issued an order providing that if defendants municipality and Bislig further wish to pursue
the matter disposed of in the order of July 26, 1979, such should be elevated to a higher court in
accordance with the Rules of Court.

ISSUE:

(1) Whether or not the Municipality of San Fernando, La Union may be sued

(2) Whether or not the Municipality of San Fernando, La Union is liable to pay damages

RULING:

(1) No. The State may not be sued except when it gives consent to be sued. Consent takes the form
of express or implied consent. The doctrine of non-suability of the State is expressly provided
for in Article XVI, Section 3 of the Constitution, to wit: "the State may not be sued without its
consent."

(2) No. The municipality cannot be held liable for the torts committed by its regular employee, who
was then engaged in the discharge of governmental functions. Hence, the death of the
passenger –– tragic and deplorable though it may be –– imposed on the municipality no duty to
pay monetary compensation.

● All premises considered, the Court is convinced that the respondent judge's dereliction in failing
to resolve the issue of non-suability did not amount to grave abuse of discretion. But said judge
exceeded his jurisdiction when it ruled on the issue of liability.
● ACCORDINGLY, the petition is GRANTED and the decision of the respondent court is hereby
modified, absolving the petitioner municipality of any liability in favor of private respondents.
BACANI VS. NACOCO-GR. L-9657

FACTS: Herein petitioners are stenographers in Branch VI of the CIF Manila.

In a pending civil case where the public respondents are involved, they requested for the services of the
stenographers and thereby paid them for the said transcript at the rate of P1 per page, amounting to
P714 in total.

However, upon inspecting the books of the corporation, the Auditor General disallowed the payment of
such fees and sought for the recovery of the amounts paid. Consequently, the AG required the petitioners
to reimburse the amounts invoking that the National Coconut Corporation is a government entity within
the purview of section 2 of the Revised Administrative Code of 1917 which states that: “‘The Government
of the Philippine Islands’ is a term which refers to the corporate governmental entity through which the
functions of government are exercised throughout the Philippine Islands, including, save as the contrary
appears from the context, the various arms through which political authority is made effective in said
Islands, whether pertaining to the central Government or to the provincial or municipal branches or other
form of local government.”, hence, exempted from the payment of the fees in question.

ISSUE: Whether the NCC is a government entity and is exempted from the payments in question?

RULING: The Court held No. Discussing, there are two-fold functions of the government namely:
constituent and ministrant. The constituent function refers to the bonds of society and are compulsory in
nature, while ministrant is more on public welfare like public works, education, charity, health and safety.
From such, we may infer that there are functions which our government is required to exercise to promote
its objectives as expressed in our Constitution and which are exercised by it as an attribute of
sovereignty, and those which it may exercise to promote merely the welfare, progress and prosperity of
the people.

The NCC has that function because the corporation promotes certain aspects of the economic life of the
people. In short, NCC belongs to what we call the government-owned and controlled corporation which is
governed by Corporation Law.

Albeit the NCC performs governmental functions for the people’s welfare, however, it was given a
corporate power separate and distinct from our government, for it was made subject to the provisions of
our Corporation Law in so far as its corporate existence and the powers that it may exercise are
concerned.

To recapitulate, we may mention that the term “Government of the Republic of the Philippines” used in
section 2 of the Revised Administrative Code refers only to that government entity through which the
functions of the government are exercised as an attribute of sovereignty, and in this are included those
arms through which political authority is made effective whether they be provincial, municipal or other
form of local government.

Therefore, NCC is not a government entity and is not exempted from the payment of fees in question;
petitioners are not subject to reimbursement.

Petition GRANTED.
NPC VS. CITY OF CABANATUAN
FACTS:
Pursuant to Sec. 37 of Ordinance No. 165-92, the city government of Cabanatuan
assessed NPC, a (Gov’t owned and controlled corp) GOCC tasked to undertake the
development of power plants, a franchise tax amounting to P808,606.41, representing
75 percent of 1 percent of the latter's gross receipts for the preceding year.

NPC refused to pay, arguing that the city government did not have the authority to
impose tax on government entities, and that as a non-profit organization, it is
exempted from the payment of all forms of taxes, charges, duties or fees in
accordance with Sec. 13 of RA No. 6395.

The city government, on the other hand, claimed that NPC's exemption from payment
of tax was already repealed by Sec. 193 of the Local Government Code.

ISSUE: WON The City of Cabanatuan has the authority to impose taxes on the NPC.

RULING: Yes. The Court ruled in favor of Cabanatuan City.

Although as a general rule, LGUs cannot impose taxes, fees, or charges of any kind
on the National Government, its agencies and instrumentalities, this rule now admits
an exception, i.e., when specific provisions of the LGC authorize the LGUs to impose
taxes, fees or charges on the aforementioned entities. In this case, Section 151 in
relation to Section 137 of the LGC clearly authorizes Cabanatuan government to
impose on NPC the franchise tax in question.

To determine whether the NPC is covered by the franchise tax in question, the
following requisites should concur: (1) that petitioner has a “franchise” in the sense of
a secondary or special franchise; and (2) that it is exercising its rights or privileges
under this franchise within the territory of the respondent city government. NPC fulfills
both requisites.

Petitioner was created to “undertake the development of hydroelectric generation of


power and the production of electricity from nuclear, geothermal and other sources, as
well as the transmission of electric power on a nationwide basis.”[66] Pursuant to this
mandate, petitioner generates power and sells electricity in bulk. Certainly, these
activities do not partake of the sovereign functions of the government. They are purely
private and commercial undertakings, albeit imbued with public interest. The public
interest involved in its activities, however, does not distract from the true nature of the
petitioner as a commercial enterprise, in the same league with similar public utilities
like telephone and telegraph companies, railroad companies, water supply and
irrigation companies, gas, coal or light companies, power plants, ice plant among
others; all of which are declared by this Court as ministrant or proprietary functions of
government aimed at advancing the general interest of society.

MALTO VS. PEOPLE as stated in CABALLO VS. PEOPLE


NAME OF DIGESTER: Benjamin Misoles Jr,

FACTS:
Between November 1997 to 1998, Michael John. Z. Malto, seduced his
student, AAA, a minor, to indulge in sexual intercourse several times with him.
Prior to the incident, petitioner and AAA had a “mutual understanding” and
became sweethearts. Pressured and afraid of the petitioner’s threat to end
their relationship, AAA succumbed and both had sexual intercourse.
AAA’s mother lodged a complaint through the prosecutor and a violation of
Section 5(a), Article III, RA 7610 was filed against Malto. The trial court found
the evidence in finding petitioner guilty beyond reasonable doubt for violation of
said law and sentenced him to reclusion temporal. The appellate court affirmed
the conviction and thereby sentenced to an indeterminate penalty prision
mayor because the acts are not covert by Sec. 5 of RA 7610.

ISSUE:
Whether or not the CA erred in sustaining petitioner’s conviction on the
grounds that there was no rape committed since their sexual intercourse was
consensual by reason of their “sweetheart” relationship.
HELD:
NEGATIVE. Petitioner is wrong.
Petitioner violated Section 5(b), Article III of RA 7610, as amended. The first
element of Section 5(b), Article III of RA 7610 pertains to the act or acts
committed by the accused. The second element refers to the state or condition
of the offended party. The third element corresponds to the minority or age of
the offended party. Since all three elements of the crime were present, the
conviction of the petitioner was proper.
Consent of the child is immaterial in criminal cases involving violation of Sec. 5,
Art. III of RA 7610. A child cannot give consent to a contract under our civil
laws. This is on the rationale that she can easily be the victim of fraud as she is
not capable of fully understanding or knowing the nature or import of her
actions.
“A child cannot give consent to a contract under our civil laws.This is on the
rationale that she can easily be the victim of fraud as she is not capable of fully
understanding or knowing the nature or import of her actions. The State, as
parens patriae, is under the obligation to minimize the risk of harm to those
who, because of their minority, are as yet unable to take care of themselves
fully. Those of tender years deserve its protection.”
PEOPLE VS. BAYLON
Louise Francis P. Ouano

TOPIC: Doctrine of Parens Patriae

FACTS:
The complainant, Sugana Aspili, only thirteen years of age at the time of the alleged
crime, testified that as a first year high school student, she used to commute from the
barrio where she lived to the poblacion, about four kilometers away from where she
lived, where the Batac Institute was located.1 She was on March 15, 1965, at about
5:00 o'clock in the morning, on her way to school, her classes starting at 7:30.2 As she
was nearing the barrio school of Colo, appellant, suddenly emerged from the thicket
on the left side of the road, embraced her, and at the same time pulled her towards
him.3 She shouted and cried for help; she was silenced, appellant covering her mouth
with his right palm, and pointing a knife on her right chest, saying, "You shout and I will
kill you, if you cry, I will kill you."4 Her bandanna was then wrapped around her mouth
by appellant, who dragged her in the direction of the canal on the left side of the
road.5

Upon reaching the spot, he tried to force her to the ground, but she strongly resisted.
She was then hit by him on her abdomen, the painful blow resulting in her weakened
condition. Appellant was quick to take advantage. Forthwith, he pushed her to the
ground, raised her chemise, forcibly removed her panties, separated her legs and
knelt between her thighs. He then unbuttoned his pants, went on top of her and tried
to insert his penis into her private
parts.6 At first, he was unable to do so, as she continued her resistance, but ultimately
he was able to pin her shoulders down with his elbows at which instance, he finally
succeeded in putting his penis into her organ.7 While thus on top of her, appellant
moved his hips up and down, at the same time kissing her, after removing the
bandana tied over her mouth. Shortly thereafter, she felt something hot and slippery
emitting from the penis of the appellant. Soon, he released her from his grip and left
the place.

With her father and mother present, the complainant described to the barrio captain
how she was raped by the appellant Domiciano Baylon.15 Thereafter, the barrio
captain brought the three of them to the house of Fidel Ramos, a barrio councillor, as
this house was the center of the barrio.16 It was the councilor who had the appellant
fetched for investigation. Appellant, as he had a right to, refused confirm the
accusation against him, maintaining his silence, preferring to speak only if so advised
by counsel.17

ISSUE: w/n the defense of Baylon’s alibi would prosper and have the decision
reversed.
RULING: NO.

Nor is this all. As was noted in a recent case, People v. Molina,32 it is manifest
in the decisions of this Court that where the offended parties are young and immature
girls like the victim in this case,33 there is a marked receptivity on its part to lend
credence to their version of what transpired. It is not to be wondered at. The state, as
parens patriae, is under the obligation to minimize the risk of harm to those, who,
because of their minority, are as yet unable to take care of themselves fully. Those of
tender years deserve its utmost protection. Moreover, the injury in cases of rape is not
inflicted on the unfortunate victim alone. The consternation it causes her family must
also be taken into account. It may reflect a failure to abide by the announced concern
in the fundamental law for such institution .34 There is all the more reason then for the
rigorous application of the penal law with its severe penalty for this offense, whenever
warranted. It has been aptly remarked that with the advance in civilization, the
disruption in public peace and order it represents defies explanation, much more so in
view of what currently appears to be a tendency for sexual permissiveness. Where the
prospects of relationship based on consent are hardly minimal, self-restraint should
even be more marked.
VELASCO, JR. OPINION in GATUS VS. QUALITY HOUSE,
INC.
Link to full text: G.R. No. 156766
Digested by: Kaye Turija

TOPIC: Part 4 PH as a State > Government > Definition, Classifications, Functions, Parens
Patriae

FACTS: Petitioner Rosario A. Gatus (petitioner) started her employment as an


assembler with respondent Quality House, Inc. (respondent company) on July 14,
1987.

The respondent company placed her on preventive suspension on July 1, 1997 to


end on 08 July 1997 in view of the incident that occurred 30 June 1997 at Mapa
Avenue, Sta. Mesa, Manila involving her husband, Ferdinand Gatus, Rosario, and
her co-employee, Leonilo Echavez.

**The assailed decision narrated the June 30, 1997 incident as follows:

It appears that on June 30, 1997, Mr. Echavez [petitioner] and her husband and other
employees of [respondent] corporation, namely, Nelia Burabo and Reynaldo Padayao,
were in a waiting shed when [petitioner's] husband suddenly turned towards
Echavez and mauled the latter. Echavez fell to the ground and sustained several
bruises, soft tissue swelling and musculoskeletal pain, as shown by a medico-legal
report (Rollo, p. 65) and Echavez' affidavit.

The petitioner promptly submitted on the same date her explanation in response to the
respondent company's July 1, 1997 notice. She complained in Filipino that she was
experiencing difficulties in her work, caused by her co-employees Shelly, Rene and
Nilo Echavez, due to her trade union activities. She claimed that she was being
harassed by the three, especially Nilo Echavez, because she did not join the
Philippine Association of Free Labor Unions (PAFLU). She said she preferred to be an
independent unionist. She narrated that the harassment and humiliation persisted
to the point of becoming unbearable; she was left with no recourse but to tell her
husband about her workplace problems. This made her husband mad.

Petitioner filed a complaint for illegal suspension and damages against and
subsequently after her dismissal, charges of unfair labor practice and illegal dismissal,
with claims for moral and exemplary damages.

The respondents' reply narrated the infractions the petitioner committed during her
employment that showed her continuing poor work attitude, and for which she
received the penalties of reprimand and two suspensions.
Labor Arbiter Potenciano S. Cañizares, Jr. dismissed the complaint for lack of merit.
The arbiter
found no substantial evidence that showed that the respondents committed unfair
labor practice,
and that her dismissal was for a just cause under the Labor Code. The NLRC
(National Labor Relations Commission) affirmed the labor arbiter's ruling.

Petitioner secured a motion for reconsideration from NLRC, which referred the case to
Labor Arbiter Luis D. Flores. Arbiter Flores, who submitted a report recommending the
petitioner's reinstatement, with full backwages and without loss of seniority rights. The
NLRC found the report to be supported by the facts and the law and, on this basis,
reversed its earlier decision. The respondents sought relief from the CA by way of a
petition for certiorari and prohibition. The CA ruled that the NLRC committed grave
abuse of discretion amounting to lack of jurisdiction when it reinstated the petitioner
and awarded her monetary benefits.

ISSUE: W/N her suspension was illegal or the Due Process Issue

RULING: Similarly, the CA was correct when it concluded that the petitioner was not
denied due process in the consideration of her dismissal. The petitioner insinuated in
this regard that due process requires a formal hearing as an absolute requirement in
employee dismissals.

It was noted and stressed once more for everyone's guidance that the law itself only
requires "ample opportunity to be heard." The essence of this requirement as an
element of due process in administrative proceedings is the chance to explain one's
side. Jurisprudence has amply clarified that administrative due process cannot be fully
equated with due process in the strict judicial sense, and that there is no violation of
due process even if no formal or actual hearing was conducted, provided a party is
given a chance to explain his side. What is frowned upon is the denial of the
opportunity to be heard.

Court DENIED the petition for lack of merit.

In relation to Labor - Social justice commands that the State, as parens patriae, and
guardian of the general welfare of the people, afford protection to the needy and the
less fortunate members of society, meaning the working class. This command
becomes all the more urgent in labor cases where security of tenure is an integral
issue. (From Pascua notes, p. 111)
PEOPLE VS. BABASA
Kyra Valentin

TOPIC: Government: Definition, Classification, Functions, Parents Patriae

FACTS: This is a review en consulta of the judgment of the Court of First


Instance of Albay, convicting Edmundo Babasa of forcible abduction with rape,
sentencing him to death and ordering him to pay to Magdalena Bermas moral
damages amounting to three thousand pesos (Criminal Case No. 158).

Magdalena claimed that she was raped by 3 men including Babasa and that
when another man entered the scene, Babasa asked the man if he wanted to
have intercourse with her too. The 4th man declined his offer. This claim by
Magdalena was defended by Babasa saying that he was only a mere driver of
the two men and that he was the one being invited to have intercourse with
Magdalena but he declined.

Rape committed by two or more persons is punished by reclusion perpetua to


death while forcible abduction is punished by reclusion temporal. The penalty
for rape, as the more serious offense, should be imposed in its maximum
period, meaning that the greater penalty, which is death, should be imposed
(Arts. 48, 63[1], 335 and 342, Revised Penal Code).

ISSUE: whether Babasa’s guilt as a co-principal in the complex crime of forcible


abduction with rape was established beyond reasonable doubt.

RULING: WHEREFORE, the trial court’s judgment is affirmed with the


modification that the indemnity of three thousand pesos is increased to twelve
thousand pesos. Costs de oficio.
CABANAS VS. PILAPIL
Nalcot, MPAA
FT: https://lawphil.net/judjuris/juri1974/jul1974/gr_l_25843_1974.html

TOPIC: Definition, Classifications, Functions, Parens Patriae

FACTS: The insured, Florentino Pilapil had a child, Millian Pilapil, with a married
woman, the plaintiff, Melchora Cabanas. She was ten years old at the time the
complaint was filed on October 10, 1964. The defendant, Francisco Pilapil, is the
brother of the deceased. The deceased insured himself and instituted as beneficiary,
his child, with his brother to act as trustee during her minority. Upon his death, the
proceeds were paid to him. Hence this complaint by the mother, with whom the child is
living, seeking the delivery of such sum. She filed the bond required by the Civil Code.
Defendant would justify his claim to the retention of the amount in question by invoking
the terms of the insurance policy.

ISSUE: WON the mother is the rightful trustee for the minor beneficiary.

RULING: Yes. After trial duly had, the lower court in a decision of May 10, 1965,
rendered judgment ordering the defendant to deliver the proceeds of the policy in
question to plaintiff. Its main reliance was on Articles 320 and 321 of the Civil Code.
The former provides: "The father, or in his absence the mother, is the legal
administrator of the property pertaining to the child under parental authority. If the
property is worth more than two thousand pesos, the father or mother shall give a
bond subject to the approval of the Court of First Instance." The latter states: "The
property which the unemancipated child has acquired or may acquire with his work or
industry, or by any lucrative title, belongs to the child in ownership, and in usufruct to
the father or mother under whom he is under parental authority and whose company
he lives;

It is very clear, therefore, considering the above, that unless the applicability of the two
cited Civil Code provisions can be disputed, the decision must stand. There is no
ambiguity in the language employed. The words are rather clear. Their meaning is
unequivocal. Time and time again, this Court has left no doubt that where codal or
statutory norms are cast in categorical language, the task before it is not one of
interpretation but of application. So it must be in this case. So it was in the appealed
decision.

There is recognition in the law of the deep ties that bind parent and child. In the event
that there is less than full measure of concern for the offspring, the protection is
supplied by the bond required. With the added circumstance that the child stays with
the mother, not the uncle, without any evidence of lack of maternal care, the decision
arrived at can stand the test of the strictest scrutiny. It is further fortified by the
assumption, both logical and natural, that infidelity to the trust imposed by the
deceased is much less in the case of a mother than in the case of an uncle.
FERNANDO VS. CA
Angelica Wasawas
TOPIC: State as Parens Patriae

FACTS: Acting on reports of sale and distribution of pornographic materials, officers of the
Philippine National Police Criminal Investigation and Detection Group in the National Capital
Region (PNP-CIDG NCR) conducted police surveillance on the store bearing the name of
Gaudencio E. Fernando Music Fair (Music Fair). Judge Perfecto Laguio of the RTC of Manila,
issued a Search Warrant for violation of Article 201 of the Revised Penal Code against
petitioner Gaudencio E. Fernando and a certain Warren Tingchuy.

Police officers of the PNP-CIDG NCR served the warrant on Rudy Estorninos, who, according to
the prosecution, introduced himself as the store attendant of Music Fair.

The police searched the premises and confiscated twenty-five (25) VHS tapes and ten (10)
different magazines, which they deemed pornographic.

The RTC convicted Gaudencio E. Fernando and Rudy Estorninos for violation of Article 201 of
the Revised Penal Code, but acquitted Tingchuy for lack of evidence to prove his guilt.

On appeal, the Court of Appeals affirmed the decision of RTC in toto. Hence the petition.

ISSUE: Whether or not the Court of Appeals erred in affirming the petitioners’ conviction.

Ruling: No. As obscenity is an unprotected speech which the State has the right to regulate,
the State in pursuing its mandate to protect, as parens patriae, the public from obscene,
immoral and indecent materials must justify the regulation or limitation.

One such regulation is Article 201 of the Revised Penal Code. To be held liable, the
prosecution must prove that (a) the materials, publication, picture or literature are obscene;
and (b) the offender sold, exhibited, published or gave away such materials. Necessarily, that
the confiscated materials are obscene must be proved.

In this case, the trial court found the confiscated materials obscene and the Court of Appeals
affirmed such findings. The trial court in ruling that the confiscated materials are obscene,
reasoned as follows:
Are the magazines and VHS tapes confiscated by the raiding team obscene or
offensive to morals? . . .
Pictures of men and women in the nude doing the sexual act appearing in the nine (9)
confiscated magazines namely Dalaga, Penthouse, Swank, Erotic, Rave, Playhouse,
Gallery and two (2) issues of QUI are offensive to morals and are made and shown not
for the sake of art but rather for commercial purposes, that is gain and profit as the
exclusive consideration in their exhibition. The pictures in the magazine exhibited
indecent and immoral scenes and acts…The exhibition of the sexual act in their
magazines is but a clear and unmitigated obscenity, indecency and an offense to
public morals, inspiring…lust and lewdness, exerting a corrupting influence especially
on the youth. (Citations omitted)
The VHS tapes also [exhibit] nude men and women doing the sexual intercourse. The
tape entitled "Kahit sa Pangarap Lang" with Myra Manibog as the actress shows the
naked body of the actress. The tape exhibited indecent and immoral scenes and acts.
Her dancing movements excited the sexual instinct of her male audience. The motive
may be innocent, but the performance was revolting and shocking to good minds...

Findings of fact of the Court of Appeals affirming that of the trial court are accorded great
respect, even by this Court, unless such findings are patently unsupported by the evidence on
record or the judgment itself is based on misapprehension of facts. In this case, petitioners
neither presented contrary evidence nor questioned the trial court’s findings. There is also no
showing that the trial court, in finding the materials obscene, was arbitrary.

You might also like